20 ĐỀ, ĐÁP ÁN THI CHUYÊN THPT MÔN TIẾNG ANH

219 725 7
20 ĐỀ, ĐÁP ÁN THI CHUYÊN THPT MÔN TIẾNG ANH

Đang tải... (xem toàn văn)

Tài liệu hạn chế xem trước, để xem đầy đủ mời bạn chọn Tải xuống

Thông tin tài liệu

Part 8: Read the following passage and then put a circle round the letter of the correct words to complete each sentence which follows:.. Keeping our teeth healthy.[r]

(1)

20 ĐỀ, ĐÁP ÁN THI CHUYÊN THPT CÁC TỈNH MÔN TIẾNG ANH Sưu tầm tổng hợp: info@123doc.org info@123doc.org

Facebook: https://www.facebook.com/Toeic-4U-923591884393234/ ĐỀ SỐ 1

SỞ GIÁO DỤC VÀ ĐÀO TẠO BẮC GIANG

ĐỀ THI CHÍNH THỨC

ĐỀ THI TUYỂN SINH LỚP 10 THPT CHUYÊN BẮC GIANG NĂM HỌC 2015-2016

Môn thi: TIẾNG ANH

(Dành cho học sinh thi vào chuyên tiếng Anh, tiếng Pháp, tiếng Trung)

Ngày thi: 10/6/2015

Thời gian làm bài: 150 phút, không kể thời gian giao đề

ĐIỂM BÀI THI

Bằng số: Bằng

chữ:

Chữ ký giám khảo

1: Chữ ký giám khảo

2:

LƯU Ý : - Đề thi gồm 06 trang, thí sinh làm trực tiếp vào đề thi. - Thí sinh khơng sử dụng tài liệu kể từ điển.

I  LISTENING (20 pts)

Part 1: Listen to invitations left on Nick’s voicemail What phrase completes each statement? Write the correct letter (5 pts)

1 Denise is inviting Nick to 1 _ a dinner Ray is inviting Nick to come over

and 2 _ b the beach

3 Nick’s mother is inviting him for 3 _ c listen to a band Lauren is inviting Nick to 4 _ d watch the game on

TV

5 Steve is inviting Nick to 5 _ e a movie

(2)

Part 2: You will hear a conversation between a secretary and a student The secretary is asking the student for information in order to complete an application form for a course As you listen, fill in the appropriate information on the form below (5 pts)

Surname: (1)

First name: (2)

Country: (3)

Age: (4)

Telephone No: (5)

Part 3: You are going to hear an interview on transportation As you listen, fill in the form below with NOmore than THREE WORDSor NUMBERS (10 pts)

Questions

Names

How you get to school?

How far is it from your

home to school?

How long does it take you to get to

school?

Are you ever late because

of transportatio

n problems?

Suggestions for improving

the transportatio

n? Mike By bus ………… ………… ………… …………

Liz ………… 20 miles ………… No 10 Better Tom ………… A ………… 15 minutes …………

 THE END OF THE LISTENING

II Choose the correct answer by circling A, B, C or D to complete the sentences (5 pts)

(3)

A Shall you make some coffee, please B Would you mind making some coffee

C Why don’t we cook some coffee D Shall I make you like some coffee Welcome to the interview Please, take a

A seat B look C cup of coffee D bow

3 We went to the restaurant there was no food in the house

A although B however C because D but

4 Barcelona football club Champions League history by defeating Juventus 3-1 in the final match last Sunday to have the cup for the 5th time.

A marked B done C won D made

5 Some designers have inspiration from Vietnam's ethnic minorities to change the traditional ao dai

A made B brought C taken D done She is one of the few people

A I look up to them B to who I look up C to that I look up D to whom I look up

7 According to a recent survey, most people are on good with their neighbors

A relations B acquaintance C terms D relationships

8 Bien Cuong, a commentator, came a lot of criticism for his speech in a match of the 2015 SEA Games

A in for B over C out of D off

9 I have bought a car

A red small French B small French red C French small red D small red French

10 The bomb in the street; fortunately no one hurt

A went up B went off C went by D went out

III Give the correct tense or form of the verbs in brackets to complete the following sentences (5 pts)

(4)

2 I not understand why Hue (always forget) her umbrella

2 _ His mother, together with her friends, (just arrive) at

the station

3 _ Not only Tom (visit) us but he also brought us many

presents _

5 There is no point in (complain) about your son if you can nothing (help) him

5 _ _ It was my fault to keep you waiting so long I (inform)

you in advance

7 The virus (know) as MERS has spread enough for the

WHO to say the situation is now more serious and urgent Taylor Swift (name) among Forbes' 2015 list of the

100 Most Powerful Women in the World recently _ They suggest that the heater (repair) before winter

comes _

IV Fill in each blank in the following sentences with a proper preposition (5 pts) Hundreds people died although scientists had warned them

the eruption of Mount Pinatubo

2 The ao dai consists of a long silk tunic that is slit on the sides and worn loose pants

3 Before Tet holiday, I like to clean my house and decorate the living room flowers

4 I’m afraid I will be debt soon because I’m extremely pressed money these days

5 My teacher divided the class three groups ten students Paula applied for the post but she was turned

V Give the correct form of the given words to complete the text below (5 pts)

1 All necessary for the 28th SEA Games in Singapore have been completed so far (PREPARE)

2 Unfortunately, the film got reviews (FAVOR)

(5)

4 Teenagers always try to be of their parents (DEPEND) He wished the young couple a life of happiness and

(PROSPER)

6 The mayor of Hanoi ordered the suspension of officials responsible for a tree-cutting (MASS)

7 In my factory, women tend to men by six to one (NUMBER)

8 Are there any rivers left in the world? (POLLUTE)

9 Young children are very and shouldn’t be allowed to watch violent movies (IMPRESSION)

VI The following conversation is between a shop assistant and a customer who is buying a camera Complete the conversation with the words or phrases given in the box Write your answer in the space provided (5 pts)

A What I now?

B How I use this camera? C What happens if I

D it’s pulled up

E if it’s the wrong way F I’m ready to take my first picture

G first take the film

H Now, the camera is open I close the cover

J faces upward

Customer :

(1) _?

Assistant: Well, can you see this black button here? If (2) _, the back cover opens The button unlocks the back cover (3) _

(6)

:

Assistant: Yes, (4) _ out of the small can Make sure this part of the film (5) _ - like this

Customer :

(6) _ put it in the other way round? Assistant: Well, it won’t fit, (7) _

Customer :

(8) _?

Assistant: Now put the end of it in the spool here and wind the film on Turn this handle and then press the red button Now (9) _ of the camera The film must then be wound on once again

Customer :

The number “1” appears in the window, and (10) _!

VII The passage below contains 10 errors UNDERLINE and CORRECT them Write your answers in the numbered blanks provided There is an example (10 pts)

1 10 11 12

People began to keep animals in zoos over 3,000 years before, when the rulers of China opened an enormous zoo called the Gardens of Intelligence In much of the early zoos, animals taught to perform for the visitors This no longer happens and it is accepted that the purpose of zoos are for people to see animals behaving natural

Today, most cities have a zoo or wildlife park However, not everybody approves of zoos Person who think that zoos are a good idea say they provide us for the opportunity to learn about the natural world and be close to wild animals Both of these would not be possible with zoos On the other hand, some people disapprove of zoos because they believe it is wrong putting animals in cages, and argue that in zoos where are not managed properly, animals live in dirty conditions and eat suitable food

(7)

4 Line _ → Line _ → Line _ → Line _ → _ Line _ → _ Line _ → 10.Line _ → _

VIII Fill in each space in the following passage with one suitable word (10 pts) The 2015 Nepal earthquake,which (1) _ more than 8,000 people and injured more than 18,000, occurred at 11:56 on 25thApril The earthquake (2) _ about twenty seconds Its epicenter was the village of Barpak, Gorkha district, and its hypocenter was at a depth (3) _ approximately 15km It was the worst (4) _ disaster to strike Nepal since the 1934 Nepal-Bihar earthquake

Hundreds of thousands of people became (5) _ when their houses collapsed, entire villages were flattened Many old buildings were completely (6) _ The country also had a continued risk of landslides

Two other powerful earthquakes struck Nepal at 06:11 and 06:45 The (7) _ earthquake measured 7.9 Mw and its epicenter was identified at a distance of 80km to the northwest of Kathmandu, the capital of Nepal Bharatpur was (8) _ nearest major city to the main earthquake, 53km from the epicenter The second one was somewhat less powerful (9) the first one It occurred 65km east of Kathmandu These (10) were really terrible

IX Circle the correct answer A, B, C or D that best fits each of the blank spaces (10 pts)

MARKETS

In practically any country in the world, you are (1) to find a market somewhere Markets have been with us since (2) times, and arose wherever people need to exchange the goods they produced For example, a farmer might have exchanged a cow for tools But just as times have (3) , so have markets practices So, (4) in early times the main activity (5) with markets would have been ‘bartering’- in (6) words exchanging goods – today most stall holders wouldn’t be too (7) on accepting potatoes as payment, for instance, instead of cash

(8)

behavior which is expected in a market in one country may not be acceptable in another Even within one country, there may be some markets where you could haggle quite (10) and others where it would be advisable not to try

1 A inevitable B confident C sure D definite A ancient B antique C old D past A changed B turned C developed D differed A however B despite C nevertheless D whereas A associated B relating C connecting D attached A different B other C new D alternative

7 A fond B keen C eager D pleased

8 A look B vision C sight D view

9 A confirm B consent C approve D agree 10 A simply B plainly C easily D clearly

X Choose from the sentences (A-F) the one which fits each gap (1-5) Write your answer in the space provided Number is an example.(5 pts)

A It was the river, the Ryburn, which normally flowed so gently, that threatened us most

B It was the year when the storms came early,before the calendar even hinted at winter, even before November was out.

C They twisted and turned, rising eastwards and upwards, warning of what was to come

D It was far deeper than we’d ever seen it so near our home, lunging furiously at its banks

E There in the heights it was like the Niagara Falls, as the water surged over the edge of the dam and poured into the stream below

F It almost completely blocked our lane and made the streamside path slippery and dangerous

(9)

We had been living in our valley for sixteen months when we first realized the dangers that could exist in the surrounding hills and threaten our very survival

(0) _B _ Until that time, we had felt safe and sheltered in our valley below the protecting hills

Soon snow began to fall Within a day it lay some 15cm deep (1) _ But on the neighboring heights the snow was much deeper and stayed for longer Up there the wind blasted fiercely Deep in our valley, we felt only sudden gusts of wind; trees swayed but the branches held firm

And yet we knew that there was reason for us to worry The snow and wind were certainly inconvenient but they did not really trouble us great (2) _ It reminded us of what could have occurred if circumstances had been different, if the flow of water from the hills had not, many years before, been controlled, held back by a series of dams

In a short time, the snow started to melt Day after day, we watched furious clouds pile up high over the hills to the west Sinister grey clouds extended over the valleys (3) _ We had seen enough of the sky; now we began to watch the river, which every day was becoming fuller and wilder

The snow was gradually washed away as more and more rain streamed from the clouds, but high up in the hills the reservoir was filling and was fast approaching danger level And then it happened – for the 1st time in years the reservoir overflowed (4) _

The river seemed maddened as the waters poured almost horizontally down to its lower stretches Just a couple of meters from our cottage, the stream seemed wild beneath the bridge (5) _ For three days we prayed that it would stay below its wall Fortunately, our prayers were answered as the dam held and the waters began to subside.Thanks to this protection, we can feel our home in the valley is still secure and safe

XI Rewrite the sentences, beginning with the words given so that the meanings stay the same as the first ones (5 pts)

1 I listened to the news last night It was very informative

→ The news A lot of children and old people have to go to hospitals these days because of the hot weather

(10)

→ His brother plays He listens to the radio almost every day

→ Hardly They did not decide to move to a bigger house until after the birth of their second son

→ Only when _

XII Complete the second sentence so that it has similar meaning to the first sentence, using the word given DO NOT change the word given You must use between TWO and FIVE words, including the word given (5 pts)

1 The firemen managed to put the fire out after five hours succeeded → The firemen _ out after five hours It takes five hours by car to reach the nearest hotel drive

→ It is to the nearest hotel Are you familiar with his teaching style yet? used

→ Have you _ his teaching style yet? David impressed his new boss by settling down to work good

→ David _ his new boss by settling down to work

5 “Is Peter likely to change his mind?” Rob asked chance

→ “Is there _ changing his mind?” Rob asked

XIII Write a paragraph (10 pts)

Write a paragraph of about 80 to 100 words on how to protect our environment

(11)

HẾT -ĐÁP ÁN ĐỀ SỐ 1

SỞ GIÁO DỤC VÀ ĐÀO TẠO BẮC GIANG

HƯỚNG DẪN CHẤM

(12)

ĐỀ THI CHÍNH THỨC (Bản hướng dẫn có 02 trang)

Môn thi: TIẾNG ANH

(Dành cho học sinh thi vào chuyên tiếng Anh, tiếng Pháp, tiếng Trung)

Ngày thi: 10/6/2015

(Tổng điểm: 100 điểm)

I LISTENING (20 pts) (Mỗi câu điểm)

Part b the beach d watch the game on TV

3 a dinner e a movie c listen to a

band

Part Williams Peter

3 Canada 25 2342965

Part 3. By car

2 By bicycle/ by bike few blocks

4 (about) miles 15-20 minutes

6 It depends Yes

8 No

9 Need more buses 10 subway system

II (5 pts) (Mỗi câu 0,5 điểm)

1 B Would you mind making some coffee

2 A seat C because D made C taken

6 D to whom I look up C terms

8 A in for

9 D small red French 10.B went off

III (5 pts) (Mỗi động từ chia 0,5 điểm) would stay

2 is always forgetting

(13)

3 has just arrived did Tom visit

5 complaining/ to help

8 has been named (should) be repaired

IV (5 pts) (Mỗi giới từ 0,5 điểm) of / about

2 over up / with

4 in / for into / of down

V (5 pts) (Mỗi câu 0,5 điểm) preparations

2 unfavorable

3 surprisingly/ weaken independent

5 prosperity

6 massive outnumber unpolluted impressionable

VI (5 pts) (Mỗi câu 0,5 điểm)

1 B D H G J

6 C E A I 10 F

VII (10 pts) (Mỗi lỗi tìm được 0,5 điểm, sửa 0,5 điểm Các lỗi mà học sinh tìm khơng theo thứ tự đây)

1 Line 2: much → many

2 Line 3: taught → were taught Line 4: are → is

(14)

1 killed lasted of natural homeless

6 destroyed/ ruined first

8 the than

10 earthquakes/ quakes IX.(10 pts) (Mỗi câu điểm)

1 C sure A ancient A changed D whereas A associated

6 B other B keen C sight D agree 10 C easily X (5 pts) (Mỗi câu điểm)

1 F A C E D

XI (5 pts) (Mỗi câu điểm)

1 The news (which/ that)/ to which I listened/ I listened tolast night was very informative

2 Becauseit/the weather is (very) hot, a lot of children and old people have to go to hospitals

3 His brother plays tennis professionally, doesn’t he?

4 Hardly a day goes by/ passes without his listening to the radio Or: Hardly a day goes by/ passes when he doesn’t listen to the radio

5 Only when they had their second son/ their second son was born did they decide to move to a bigger house

XII (5 pts) (Mỗi câu điểm)

1 The firemen succeeded in putting the fireout after five hours It is a five-hour drive to the nearest hotel

3 Have you got used tohis teaching style yet?

4 David made a good impression on his new boss by settling down to work “Is thereany chance of Peterchanging his mind?” Rob asked

(15)

Các tiêu chí chấm viết đoạn văn.

Tiêu chí Mơ tả chi tiết tiêu chí Điểm

Cấu trúc

Đúng cấu trúc đoạn văn, phù hợp với chủ đề: Có câu chủ đề (Topic sentence); ý minh họa (Supporting sentences/ ideas, từ supporting sentences/ ideas trở lên); câu kết luận (Concluding sentence)

3

Nội dung

- Viết chủ đề, ý minh họa rõ ràng, có tính thuyết

phục

- Viết logic, hợp lí; mạch lạc, rõ ràng

Ngôn ngữ

- Viết cấu trúc ngữ pháp; sử dụng liên từ hợp lý để liên kết ý đoạn

- Sử dụng từ vựng phù hợp, phong phú, hạn chế lặp lại từ vựng

2

Trình bày - Viết đủ số từ theo quy định

- Khơng xuống dịng

Tổng điểm 10/100

Cách tính lỗi (trừ điểm)

- Mỗi lỗi ngữ pháp, cấu trúc câu,….: 0,2

- Mỗi lỗi từ vựng 0,2

- Số lượng từ (+ - 30% số từ theo quy định)

- HẾT -

(16)(17)(18)(19)(20)(21)(22)(23)

ĐỀ SỐ 3

ĐẠI HỌC QUỐC GIA HÀ NỘI TRƯỜNG ĐẠI HỌC NGOẠI NGỮ

KỲ THI TUYỂN SINH HỆ THPT CHUYÊN NGOẠI NGỮ NĂM 2015 ĐỀ THI MÔN: TIẾNG ANH

Ngày thi: 07/06/2015

Thời gian làm 120 phút (không kể thời gian phát đề) (Đề thi gồm trang Thí sinh làm PHIẾU TRẢ LỜI.)

I Câu 01-05: Chọn từ (ứng với A, B, C D) có phần gạch phát âm khác với từ lại câu.

01 A eliminate B equal C elaborate D ejection 02 A carriage B dosage C massage D voyage 03 A suit B bruise C suite D fruit

04 A calculate B populate C contemplate D fortunate 05 A apprehension B division C precision D measure

II Câu 06-10:Chọn từ (ứng với A, B, C D) có trọng âm nhấn vào âm tiết có vị trí khác với từ cịn lại câu.

06 A ignorant B decisive C horizon D museum

07 A suffice B product C nuclear D province 08 A neurosis B nocturnal C nominate D nostalgia 09 A psychological B contributory C argumentative D hypersensitive 10 A crescendo B attorney C compromise D endeavor

III Câu 11-20:Đọc đoạn văn sau chọn phướng án (ứng với A, B, C hoặc D) cho câu hỏi.

(1) On the night of September 2, 1666, a fire broke out in a baker's shop near Fish Street Hill in London Before the flames were finally extinguished, nearty the entreaty had been reduced to ashes Over thirteen thousand homes, fifty churches and numerous public buildings and hospitals were lost in the blaze For all practical purposes, London was destroyed

(5) The Great Fire was not seen as a total tragedy, however The deplorable conditions of the city had been attacked by physicians and humanitarians for years before the fire; thus, with the opportunity dearly presented to create a shining new dty, artists and craftsmen from all over England hurried to submit their designs for the rebuilding of London

Among those who submitted plans was Sir Christopher Wren, one of England's leading architects and the

(24)

realized that the Great Fire would not have been so damaging if the city had been better laid out: broader streets were needed to replace the crooked, narrow alleys overhung with dilapidated wooden houses and shops He also felt that redesigning the main thoroughfares of London would result in increased and more effective transportation within the city

Shortly after Wren began working on his first drafts for the rebuilding King Charles I issued a proclamation (15) prohibiting the construction of any house or shop within the city limits until after the plans were completed When the plans were unveiled to the citizens of London, however, they were overwhelmingly rejected The most vocal leaders of the opposition were the landlords, who feared that such a drastic widening of the streets would reduce the amount of land available for development

(20) Winter was approaching; consequently, it was necessary for the rebuilding to proceed at once Permission was, therefore, granted for the townspeople and landlords to commence reconstruction of their houses and shops at the sites where they had been before the fire Had the need for immediate action not been so pressing, some kind of compromise could likely have been reached This was not to be, however, and the ideas that could (25) have made London one of the world's most beautiful cities never came to pass

11 The probable meaning of “reduced to ashes” (line 2) is

A made unbearably hot B discolored by the fire and smote C covered with ashes D destroyed by fire

12 Which of the followings was probably among the deplorable conditions of the city?

A lack of adequate lightning B the rough streets and alleys

C the run-down condition of many houses and shops D all of the above

13 What did Sir Christopher Wren describe as the reason for the damage of the city? A the configuration B the construction C the landscape D the pathway

14 The word thoroughfares (line 13) refers to

A public buildings B streets C alleys D houses and shops 15 The word proclamation (line 14) refers to

A promulgation B speech C request D law 16 The word unveiled (line 16) refers to

A concealed B disclosed C obscured D edipsed 17 What became of the plans for the rebuilding of London?

A Nothing - they were never finished B They were adopted in modified form

C They were followed and London was rebuilt at once

D Nothing - London was rebuilt as it had been before the fire 18 To whom or what does they (line 23) refer to ?

(25)

C the burned-down houses and shops D the landlords

19 Which of the following describes the author's probable attitude toward Sir Christopher Wren's plans?

A He feels it would have been a mistake to rebuild London according to Wren's designs

B He feels it was a mistake for London not to have been rebuilt according to Wren's designs

C He feels that someone rather than Wren should have been chosen to plan the rebuilding

D He feels that Wren's ideas had much more influence than commonly acknowledged

20 Which of the following would be an appropriate title for the passage? A "The Life and Times of Sir Christopher Wren"

B "The Rebuilding of London"

C "The Holocaust of Fish Street Hill" D "The London That Might Have Been"

IV Câu 21-30: Đọc đoạn văn sau chọn phương án (ứng với A, B, C hoặc D) cho chỗ trống.

Smart Shoes

Smart shoes that adjust their size throughout the day could soon be available A prototype has already been produced and a commercial (21) may be in production within a few years The shoe contains sensors that constantly check the amount of room left in it If the foot has become too large, a tiny valve opens and the shoe (22) slightly The entire control system is about 5mm square and is located inside the shoe This radical shoe (23) a need because the volume of the (24) foot can change by as much as 8% during the course of the day The system is able to learn about the wearer's feet and (25) up a picture of the size of his or her feet throughout the day It will allow the shoes to change in size by up to 8% so that they always fit (26) They are obviously more comfortable and less likely to cause blisters From an athlete's point of view, they can help improve (27) a little, and that is why the first (28) for the system is likely to be in a sports shoe Eventually, this system will find a (29) In many other household items, from beds that automatically change to fit the person sleeping in them, to power tools that (30) themselves to the user's hand for better grip

21 A assortment B version C style D variety 22 A amplifies B develops C expands D increases

23 A detects B finds C meets D faces

24 A average B general C usual D medium

25 A build B pick C grow D set

(26)

29 A function B part C way D place 30 A shape B change C respond D convert

V Câu 31-40: Chọn từ/ cụm từ thích hợp (ứng với A, B, C D) để hoàn thành câu sau.

31 Save your money Don't it too quickly

A go through B die down C hold on D touch on 32 They all have to follow the rules, and none of them is the law

A over B beyond C above D onto

33 They the aid of the United States but did not receive it

A asked B found C sought D obtained

34 The police spokesman said he was to believe that the arrested man was the serial killer they had been looking for

A inclined B seemed C suspected D supposed 35 I'm my brother is

A nowhere near as ambitious as B nothing near as ambitious as C nowhere like so ambitious as D nothing as ambitious as 36 He says he might come, but I

A doubt B doubt it C doubt so D doubt about it 37 I'm sure your bank manager will lend you a ear when you explain the situation to him

A merciful B sympathetic C pitiful D compassionate 38 Have you seen the girl ?

A that I told B I told you of C whom I told you D I told you of her

39 Children usually a flu much more quickly than adults

A pick up B pick at C pick on D pick out 40 The accident was the mistake of the driver

A causing many people to die B caused great human loss

C to cause many people die D which causes many people to die

VI Câu 41-50: Sử dụng từ ngoặc dạng thích hợp để điền vào chỗ trống

(27)

screen is among great (46 essential) One unique challenge for the director is that the presentation is two-dimensional The screen is flat, so puppets can only move forwards and backwards Having chosen a screen and designed the set, the next step is to determine the light that will be used There are several factors to be considered: (47 intense) , spread, and angle Therefore, finding the optimal (48 combine) of light, shadow involves careful (49 plan) and scrupulous design. Every detail must be controlled in relation to others, making shadow puppetry an art of (50 precise)

VII Câu 51-60: Tìm từ thích hợp để điền vào chỗ trống đoạn văn Mỗi chỗ trống điền MỘT từ.

Son Doong Cave is in the heart of Phong Nha Ke Bang National Park in Quang Binh province of Central Vietnam Only recently (51) in 2009-2010 by the British Cave Research Association, the cave has only been open to the (52) since 2013

Fewer people have seen the inside of Son Doong Cave than have stood on the summit of Mount Everest Join us on this otherworldly expedition and become one of the lucky few (53) have had the life changing experience of exploring the world's largest cave

Imagine trekking straight into the (54) of the world's largest cave on an expedition unlike any other A cave is (55) massive that a Boeing 747 could fly through its largest cavern Foreign landscapes found (56) else, enormous stalagmites rising from the ground and statuesque stalactites hanging from the celling like an alien species Jungles emerge (57) the cave itself, which is so surreal that it's worth seeing once Misty clouds envelop the whole scene, a result of the cave's own localised weather system Passages adorned (58) ancient fossils offer evidence of the millions of years that have passed on this Earth

As you approach the Jungle just outside the entrance, the (59) of cool wind that cascades out brings to life everything inside of you Hazy, cold and exhilarating, it is apparent that there's (60) magical waiting just beyond the opening to the cave

VIII Câu 61-70: Sử dụng từ cho sẵn ngoặc từ khác để hoàn thành câu thứ hai cho ý nghĩa tương tự câu thứ Chỉ viết từ đến 5 từ vào Phiếu Trả Lời không thay đổi dạng thức từ cho sẵn.

61 Mary cried her eyes out just after she was told she'd been rejected for the Job (broke)

Mary as soon as she heard she'd been rejected for the job 62 I never thought that I would win the lottery

(occurred)

It never that I would win the lottery

63 I failed to make him understand that I didnt want to see him anymore (across)

(28)

64 Would you give us your answer as soon as possible? (convenience)

Please reply 65 I'm so sorry, I didn't realise it was so late

(track)

Sorry, I time 66 Everyone who spoke to the victim is a suspect

(under)

Everyone who spoke to the victim 67 She will probably be chosen for the beauty contest

(stands)

She chosen for the beauty contest 68 "Can you tell me what happened yesterday, John?" She asked

(account)

She asked John what had happened the day before 69 Those two makes of car are practically the same

(hardly)

There are those two makes of car 70 There is no way that I can meet you tomorrow

(question)

Meeting you tomorrow

IX Câu 71-80: Viết lại câu theo gợi ý (kể từ bắt đầu câu cho sẵn) vào Phiếu Trả Lời cho ý nghĩa ban đầu câu không thay đổi.

71 It's sad, but the crime rate is unlikely to go down this year Sad as 72 You must concentrate on your study more

You must apply

73 It has been nearly one year since they stopped subscribing to that magazine They cancelled

74 I don't feel like going to the party

I am not in 75 "Please don't run so fast!" Suzy begged her friend

Suzy pleaded 76 You should not only balance your diet but also sports In addition 77 Immediately after their arrival, the meeting was delayed Barely 78 If you don’t pay on time, your booking will be cancelled Failure 79 David is proud of the fact that he is never late

David prides 80 It’s nobody's fault that the meeting was cancelled

(29)

X Câu 81: Viết ĐOẠN VĂN khoảng 120 - 150 từ, sử dụng lập luận dẫn chứng cụ thể để bình luận chủ đề sau.

To be admitted to university, using the result of study during high school years is better than that of the university entrance examination Do you agree or disagree?

HẾT ĐÁP ÁN ĐỀ SỐ 2

ANSWER KEY

I Câu 01-05: Chọn từ (ứng với A, B, C D) có phần gạch phát âm khác với từ lại câu.

01 A eliminate B equal C elaborate D ejection 02 A carriage B dosage C massage D voyage 03 A suit B bruise C suite D fruit 04 A calculate B populate C contemplate D fortunate 05 A apprehension B division C precision D measure

II Câu 06-10:Chọn từ (ứng với A, B, C D) có trọng âm nhấn vào âm tiết có vị trí khác với từ cịn lại câu.

06 A ignorant B decisive C horizon D museum

07 A suffice B product C nuclear D province 08 A neurosis B nocturnal C nominate D nostalgia 09 A psychological B contributory C argumentative D hypersensitive 10 A crescendo B attorney C compromise D endeavor

III Câu 11-20:Đọc đoạn văn sau chọn phướng án (ứng với A, B, C hoặc D) cho câu hỏi.

(1) On the night of September 2, 1666, a fire broke out in a baker's shop near Fish Street Hill in London Before the flames were finally extinguished, nearty the entreaty had been reduced to ashes Over thirteen thousand homes, fifty churches and numerous public buildings and hospitals were lost in the blaze For all practical purposes, London was destroyed

(30)

Among those who submitted plans was Sir Christopher Wren, one of England's leading architects and the

(10) Surveyor of London The task of rebuilding the city was given to him Wren realized that the Great Fire would not have been so damaging if the city had been better laid out: broader streets were needed to replace the crooked, narrow alleys overhung with dilapidated wooden houses and shops He also felt that redesigning the main thoroughfares of London would result in increased and more effective transportation within the city

Shortly after Wren began working on his first drafts for the rebuilding King Charles I issued a proclamation (15) prohibiting the construction of any house or shop within the city limits until after the plans were completed When the plans were unveiled to the citizens of London, however, they were overwhelmingly rejected The most vocal leaders of the opposition were the landlords, who feared that such a drastic widening of the streets would reduce the amount of land available for development

(20) Winter was approaching; consequently, it was necessary for the rebuilding to proceed at once Permission was, therefore, granted for the townspeople and landlords to commence reconstruction of their houses and shops at the sites where they had been before the fire Had the need for immediate action not been so pressing, some kind of compromise could likely have been reached This was not to be, however, and the ideas that could (25) have made London one of the world's most beautiful cities never came to pass

11 The probable meaning of “reduced to ashes” (line 2) is

A made unbearably hot B discolored by the fire and smote C covered with ashes D destroyed by fire

12 Which of the followings was probably among the deplorable conditions of the city?

A lack of adequate lightning B the rough streets and alleys

C the run-down condition of many houses and shops D all of the above

13 What did Sir Christopher Wren describe as the reason for the damage of the city? A the configuration B the construction C the landscape D the pathway

14 The word thoroughfares (line 13) refers to

A public buildings B streets C alleys D houses and shops

15 The word proclamation (line 14) refers to

A promulgation B speech C request D law 16 The word unveiled (line 16) refers to

A concealed B disclosed C obscured D

edipsed

17 What became of the plans for the rebuilding of London? A Nothing - they were never finished

(31)

C They were followed and London was rebuilt at once

D Nothing - London was rebuilt as it had been before the fire 18 To whom or what does they (line 23) refer to ?

A the original building sites B the townspeople C the burned-down houses and shops D the landlords

19 Which of the following describes the author's probable attitude toward Sir Christopher Wren's plans?

A He feels it would have been a mistake to rebuild London according to Wren's designs

B He feels it was a mistake for London not to have been rebuilt according to Wren's designs

C He feels that someone rather than Wren should have been chosen to plan the rebuilding

D He feels that Wren's ideas had much more influence than commonly acknowledged

20 Which of the following would be an appropriate title for the passage? A "The Life and Times of Sir Christopher Wren"

B "The Rebuilding of London"

C "The Holocaust of Fish Street Hill" D "The London That Might Have Been"

IV Câu 21-30: Đọc đoạn văn sau chọn phương án (ứng với A, B, C hoặc D) cho chỗ trống.

Smart Shoes

Smart shoes that adjust their size throughout the day could soon be available A prototype has already been produced and a commercial (21) may be in production within a few years The shoe contains sensors that constantly check the amount of room left in it If the foot has become too large, a tiny valve opens and the shoe (22) slightly The entire control system is about 5mm square and is located inside the shoe This radical shoe (23) a need because the volume of the (24) foot can change by as much as 8% during the course of the day The system is able to learn about the wearer's feet and (25) up a picture of the size of his or her feet throughout the day It will allow the shoes to change in size by up to 8% so that they always fit (26) They are obviously more comfortable and less likely to cause blisters From an athlete's point of view, they can help improve (27) a little, and that is why the first (28) for the system is likely to be in a sports shoe Eventually, this system will find a (29) In many other household items, from beds that automatically change to fit the person sleeping in them, to power tools that (30) themselves to the user's hand for better grip

21 A assortment B version C style D variety 22 A amplifies B develops C expands D increases

23 A detects B finds C meets D faces

(32)

25 A build B pick C grow D set

26 A exactly B absolutely C completely D totally 27 A achievement B performance C success D winning 28 A purpose B exercise C use D operation

29 A function B part C way D place

30 A shape B change C respond D convert

V Câu 31-40: Chọn từ/ cụm từ thích hợp (ứng với A, B, C D) để hoàn thành câu sau.

31 Save your money Don't it too quickly

A go through B die down C hold on D touch on

32 They all have to follow the rules, and none of them is the law

A over B beyond C above D onto

33 They the aid of the United States but did not receive it

A asked B found C sought D obtained

34 The police spokesman said he was to believe that the arrested man was the serial killer they had been looking for

A inclined B seemed C suspected D

supposed

35 I'm my brother is

A nowhere near as ambitious as B nothing near as ambitious as

C nowhere like so ambitious as D nothing as ambitious as 36 He says he might come, but I

A doubt B doubt it C doubt so D doubt about it

37 I'm sure your bank manager will lend you a ear when you explain the situation to him

A merciful B sympathetic C pitiful D compassionate

38 Have you seen the girl ?

A that I told B I told you of C whom I told you D I told you of her

39 Children usually a flu much more quickly than adults

A pick up B pick at C pick on D pick out 40 The accident was the mistake of the driver

A causing many people to die B caused great human loss

C to cause many people die D which causes many people to die

(33)

trống

Shadow puppetry is a traditional art form that often goes (41 appreciate) in modem times A large part of the (42 appealable) of puppet shows is the (43 craft) behind the creation of the actual puppets In shadow puppetry, on the other hand, the puppets remain (44 see) , so the real artistry is in the presentation The combination of the puppets’ shape, the background screen, and the light itself creates the overall effect of the shadow puppet show The task of the director is to ensure these elements are working together (45 harmony) in order to produce the optimal experience for the audience The screen is the medium through which the audience experiences the performance, so selecting the best screen is among great (46 essential) One unique challenge for the director is that the presentation is two-dimensional The screen is flat, so puppets can only move forwards and backwards Having chosen a screen and designed the set, the next step is to determine the light that will be used There are several factors to be considered: (47 intense) , spread, and angle Therefore, finding the optimal (48 combine) of light, shadow involves careful (49 plan) and scrupulous design. Every detail must be controlled in relation to others, making shadow puppetry an art of (50 precise)

41 appreciated 42 appeal 43 craftiness 44 unseen 45 harmoniously 46 essentials 47 intensity 48 combination 49 planning 50 precision VII Câu 51-60: Tìm từ thích hợp để điền vào chỗ trống đoạn văn Mỗi chỗ trống điền MỘT từ.

Son Doong Cave is in the heart of Phong Nha Ke Bang National Park in Quang Binh province of Central Vietnam Only recently (51) in 2009-2010 by the British Cave Research Association, the cave has only been open to the (52) since 2013

Fewer people have seen the inside of Son Doong Cave than have stood on the summit of Mount Everest Join us on this otherworldly expedition and become one of the lucky few (53) have had the life changing experience of exploring the world's largest cave

Imagine trekking straight into the (54) of the world's largest cave on an expedition unlike any other A cave is (55) massive that a Boeing 747 could fly through its largest cavern Foreign landscapes found (56) else, enormous stalagmites rising from the ground and statuesque stalactites hanging from the celling like an alien species Jungles emerge (57) the cave itself, which is so surreal that it's worth seeing once Misty clouds envelop the whole scene, a result of the cave's own localised weather system Passages adorned (58) ancient fossils offer evidence of the millions of years that have passed on this Earth

As you approach the Jungle just outside the entrance, the (59) of cool wind that cascades out brings to life everything inside of you Hazy, cold and exhilarating, it is apparent that there's (60) magical waiting just beyond the opening to the cave

(34)

56 Nowhere 57 outside 58 with 59 current / blowing 60 more VIII Câu 61-70: Sử dụng từ cho sẵn ngoặc từ khác để hoàn thành câu thứ hai cho ý nghĩa tương tự câu thứ Chỉ viết từ đến 5 từ vào Phiếu Trả Lời không thay đổi dạng thức từ cho sẵn.

61 Mary cried her eyes out just after she was told she'd been rejected for the Job (broke)

Mary as soon as she heard she'd been rejected for the job broke down

62 I never thought that I would win the lottery (occurred)

It never that I would win the lottery occurred to me

63 I failed to make him understand that I didnt want to see him anymore (across)

I failed that I didnt want to see him anymore put it across

64 Would you give us your answer as soon as possible? (convenience)

Please reply at your convenience

65 I'm so sorry, I didn't realise it was so late (track)

Sorry, I time didn’t follow the track of (time)

66 Everyone who spoke to the victim is a suspect (under)

Everyone who spoke to the victim is under suspicion

67 She will probably be chosen for the beauty contest (stands)

She chosen for the beauty contest.stands a chance of being

68 "Can you tell me what happened yesterday, John?" She asked (account) She asked John what had happened the day before to account for

69 Those two makes of car are practically the same (hardly)

There are those two makes of car hardly any differences between

70 There is no way that I can meet you tomorrow (question)

Meeting you tomorrow is out of the question

(35)

Phiếu Trả Lời cho ý nghĩa ban đầu câu không thay đổi. 71 It's sad, but the crime rate is unlikely to go down this year Sad as it is, the crime rate is unlikely to go down this year.

72 You must concentrate on your study more You must apply yourself with your study.

73 It has been nearly one year since they stopped subscribing to that magazine They cancelled subscribing to that magazine nearly a year ago.

74 I don't feel like going to the party

I am not in the mood for going to the party.

75 "Please don't run so fast!" Suzy begged her friend Suzy pleaded with her friend not to run so fast.

76 You should not only balance your diet but also sports In addition to balancing your diet, you should sports.

77 Immediately after their arrival, the meeting was delayed Barely had they arrived when the meeting was delayed.

78 If you don’t pay on time, your booking will be cancelled Failure to pay on time will cancel your booking

79 David is proud of the fact that he is never late David prides himself on being never late.

80 It’s nobody's fault that the meeting was cancelled

Nobody is to blame for the meeting being cancelled / for the cancel of the meeting X Câu 81: Viết ĐOẠN VĂN khoảng 120 - 150 từ, sử dụng lập luận dẫn chứng cụ thể để bình luận chủ đề sau.

To be admitted to university, using the result of study during high school years is better than that of the university entrance examination Do you agree or disagree?

The End ĐỀ SỐ + ĐÁP ÁN ĐỀ SỐ 4

BỘ GlÁO DỤC VÀ ĐÀO TẠO CỘNG HÒA XÃ HỘI CHỦ NGHĨA VIỆT NAM TRƯỜNG ĐẠI HỌC SƯ PHẠM HÀ NỘl Độc lập – Tự – Hạnh phúc ĐỀ THI TUYỂN SINH

VÀO TRƯỜNG TRUNG HỌC PHỔ THÔNG CHUYÊN NĂM 2015 Môn thi: TIẾNG ANH

(Dùng cho thí sinh thi vào chuyên Anh) Thời gian làm bài: 120 phút

Mã đề số: 210

(36)

Question 1: A laughter B caught C daughter D augment Question 2: A incline B eradicate C exacerbate D enclosure Question 3: A friends B opinions C picnics D computers

Question 4: A chimpanzee B interviewee C refugee D committee Question 5: A politeness B conversation C resolution D introduction Choose the best answer to complete each of the following sentences.

Question 6: I'd love to have lived in the old days, when people to market by horse and carriage

A have been travelling B got used to travelling

C would travel D had been travelling

Question 7: I'll give you the phone number of my hotel so that you can reach me if anything happens .anything happen I want you lo look after my children

A Can B Might C Will D Should

Question 8: We've been together through in our friendship, and we won't desert each other now

A bad and good B thick and thin C odds and ends D spick and span Question 9: a scholarship, I entered the frightening and unknown territory of private education

A To award B Having awarded

C To be awarded D Having been awarded

Question 10: Millions of people say Coke tastes best from a bottle, and whether this is scientifically provable or not These millions know they like the look of the bottle and the way it fits so into the hand

A neatly B orderly C tidily D finitely

Question 11: Language is so woven into human experience that it is scarcely possible to imagine life without it

A tightly B tautly C rigidly D stiffly

Question 12: I'd like to help you out, I'm afraid I just haven't got any spare money at the moment

A Much as B Try as C Even D Despite

Question 13: She was very badly depressed after the car accident Now she is beginning to think that there could be light at the end of the

A tunnel B subway C passage D journey

Question 14: The government must strong measures against crime.

A be seen be taking B see to be taking

C be seen to be taking D seen to take

Question 15: Athough she would have preferred to carry on working, my mum her career in order to have children

(37)

Choose the word whose primary stress is placed differently from that of the others Question 17: A compliment B excellent C nominate D distinguish Question 18: A remember B quality C occasion D terrific Question 19: A worldclass B wheelchair C firewood D blackmail Question 20: A volunteer B referee C spiritual D recommend Question 21: A influential B accessible C rudimentary D incidental Read the passage and choose the best answer to each of the questions that follow

GETTING THE BEST OUT OF OUR CHILDREN

There is a strange paradox to the success of the Asian education model On the one hand, class sizes are huge by western standards with on average between 30 and 40 students per class in countries like Japan and Korea On the other hand, school children in developed Asian economies rank among the highest in the world for academic achievement in the areas of science and mathematics, especially on standardised tests Meanwhile, British secondary school students fail to shine in conditions most educational researchers would say are far more likely to help them succeed

Why Asian students seem to perform so well then? Is it their legendary discipline? Certainly, classroom management seems to be a whole lot easier in places like Korea, and perhaps lessons are more effective as a direct consequence After all, we are only too aware of the decline in discipline standards in our own schools; belligerent and disrespectful students appear to be the norm these days Teachers in Britain seem powerless to control what happens any more Surely this situation cannot create a very effective learning environment, so perhaps the number of students is far less relevant than is the manner in which they conduct themselves But there are other factors to consider, too Korean students spend a lot more time with their teachers It seems logical to suggest, therefore, that they might form stronger bonds and greater trust, and that Korean teachers, in understanding their pupils better, might be able to offer them a more effective learning programme Of course, trust and understanding leads to greater respect as well, so Korean students are probably less likely to ignore their teachers’ advice

Then there is the home environment The traditional family unit still remains relatively intact in Korea Few children come from broken homes, so there is a sense of security, safety and trust both at home and at school In Britain meanwhile, one in every two marriages fails and divorce rates are sky-high Perhaps children struggle to cope with unstable family conditions and their only way to express their frustration is by misbehaving at school Maybe all this delinquent behaviour we are complaining about is just a cry for help and a plea for attention

(38)

question: is all that extra effort justified for a few extra percentage points in some meaningless international student performance survey? So Asian students are on average 3-5% better at maths than Britons – big deal! What is their qualily of life like? Remember, school days are supposed to be the best, are they not?

There has been a lot of attention and praise given to these Asian models and their impressive statistics of late And without question, some of this praise is justified, but it seems to be a case of two extremes in operation here At one end, there is the discipline and unbelievably hard work ethic of the Asian students success in education before all else At the other end, Brilish students at times appear careless and extremely undisciplined by comparison, but at least they DO have the free time to enjoy their youth and explore their interests Is either system better outright? Or is it perhaps about time we stopped comparing and started trying to combine the best bits of both, so that we can finally offer our students a balanced, worthwhile education? We are not just dealing with statistics; never forget that every statistic is a little human being somewhere who desperately needs our help and guidance - who deserves it

Question 22: What does the writer mean when he says there is a paradox in the Asian education model?

A There are too many students in each class

B Larger classes are expected to lead to poorer results but they not C Asian students outperform their peers in other countries

D Class sizes in Asia are much smaller in other parts of the world Question 23: British secondary school students

A better on standardised tests B have larger class sizes

C fail at school more than they succeed D enjoy better classroom conditions Question 24: What does the writer suggest might make lessons in Korean schools more successful than in Britain?

A stricter classroom discipline B better school Boards of Management C more effective lesson planning D better teachers

Question 25: What can be inferred from the utterance perhaps the number of students is far less relevant than is the manner in which they conduct themselves? A Class size does not affect student performance

B Class size is important to maintaining control

C How students behave might be more important than class size D How teachers conduct classes affects student performance Question 26: The traditional family unit

A is unstable in Korea due to conditions in the home B is bad for children that come from broken homes

C is disappearing in Korea due to high divorce rates D is more common in Korea than in Britain

Question 27: Look at the following sentence.

You see, behind those great maths and science scores, there is a quite remarkable work ethic.

(39)

But while the Japanese, Korean and Asian models generally seem to produce excellent results, the statistics don’t tell the truth [1] Asian sludents tend to put their education before literally everything else They very few extracurricular activities and devote far effort more time to their studies than their British peers [2] And this begs the question: is all that extra effort justified for a few extra percentage points in some meaningless international student performance survey? So Asian students are on average 3-5% better at maths than Britons – big deal! [3] What is their qualily of life like? Remember, school days are supposed to be the best, are they not?

[4]

A [4] B [3] C [2] D [1]

Question 28: According to the wriler, Asian students A don't allow themselves much time to relax and have fun

C don't have as good a work ethic as British ones D make a big deal of their good results

Question 29: What are the 'too exlremes’ mentioned in the last paragraph? A good discipline and a hard work ethic B success and failure

C carelessness and indiscipline D neglecting school and neglecting free time

Question 30: Which conclusion about the two educational systems discussed would the author most probably agree with?

A Neither system is perfect B Both systems are quite satisfactory

C The Asian system is obviously better D The British system is too strict Read the text below and decide which answer best fits each gap The first one has been done as an example (0).

Example: (0) A beginning B first C primary D basic VOLUNTEERING

When Pamela Janett left university to become a (0) school teacher, it was by no (31) easy to find a job She therefore decided to go abroad as a volunteer teacher for a year When she realized she would be teaching deaf and blind children, she was a (32) taken aback But after a month's training she felt more confident that she would be able to cope with the situation The basic living conditions also (33) as something of a shock

(40)

Question 31: A means B extent C ways D account Question 32: A rather B quite C little D somewhat Question 33: A appeared B came C proved D arrived Question 34: A combined B consisted C contained D composed

Question 35: A down B away C out D off

Question 36: A worthy B valuable C priceless D precious Question 37: A contrast B difference C change D transformation Question 38: A furthermore B nonetheless C otherwise D however Question 39: A growing B widening C expanding D stretching Question 40: A greater B larger C steeper D taller

Put each verb in brackets into a suitabie tense or form The first one has been done as an example (0).

Example: (0) took

LEARNING TO SURVIVE

Last summer I (0 take) a three-week survival course The purpose of the course was (41 teach) us how to survive outdoors, where there are no shops, no houses and no electricity I had never slept outdoors before the course, and here I was (42 learn) to make a fire, navigate and find food in the forest The part that I (43. like) the most was catching our own fish and cooking them over a fire While (44. do) .the course, I realised how much people (45 depend) .on modern technology They think that they can't survive without it but they can if they (46 learn) how I did! I (47 never forget) .that course - it was the greatest experience I (48 ever have) so far Now I think if I (49 not take) the survival course, I (50 not be) able to deal with so many difficult situations in life

Use the word given in brackets to form a word that fits in the space The first one has been done as an example (0) Example: (0.) likely

MODERN CULTURE?

When people talk about contemporary culture they are just as (0 LIKE) to be talking about fast cars, trainers or high heels as they are to be talking about Shostakovich or Shakespeare

Goods have become as (51 MEAN) a measure and marker of culture as the Great and the Good The word "culture" can now cover just about anything Culture is no longer merely the beautiful and (52 SINGLE) … until the late twentieth century that a (53 SCHOLAR) interest in objects began to (54 PLACE)… the traditional interest in -isms, with historians, (55 LITERATE) critics and philosophers all suddenly becoming fascinated by the meaning of objects, large and small Is this a sign, perhaps, of a society cracking under the strain of too many things?

(41)

Read the text below and think of the word which best fits each space Use only ONE word in each space.

The first one (0) has been done as an example.Example: (0) with

When presented (0) the idea of international boarding school, parents are (61) to baulk of emptying their bank (62) to send their darlings to live a life permeated by blackboards, bunk beds and Bovril (63) conversely, will feel overjoyed at the prospect of signing (64) for a literally fantastic life at Hogwarts Such misconceptions neatly miss the point of internalional boarding education, sidestepping its capacity to (65) solid foundations for students' academic and professional success in today’s increasingly global society

Parents, understandably, require justification for (66) a considerate amount of money in their offspring's education Their offspring, on the other hand, should be made (67) of the huge number of opportunities provided by boarding school life Most of the world's leading boarding schools offer stimulating, progressive (68) which prioritizes their students' needs, thus consistently produce graduates (69) are confident learners, critical thinkers and self starters

Therefore, the time has come for parents and students (70) to discover the unique, intellectually challenging experiences offered by internalional boarding schools worldwide

Complete each of the following sentences with one of the phrasal verbs given in the box Each phrasal verb is used only ONCE Make any necessary changes.

put up go up come off make up

for call for come up drop out of set up take up grow out

of

Question71 The hotels were all full so we offered to Carla… for the night Question72 Early that morning, we set off on our journey as the sun was Question73 An enquiry was into the use of chemicals in farming.

Question74 You're such a good singer that you, should it professionally. Question75 How are they going to the time they wasted playing cards in the barracks?

Question76 Stop wasting your time The whole situation an immediate response that could bring more

decisive effects

Question77 I like this photograph so much that I am going to have it…

Question78 As far as I know, the idea of the party does not quite appeal to him and that's why he's thinking

(42)

Question79 I can't stop thinking there's something more that the scheme needs

Question80 Peter used to be fond of collecting mascots, but after his military service, he the hobby

Complete the second sentence using the word given so that it has a similar meaning to the following sentence Write between two and five words in the space provided on the answer sheet Do NOT change the word given in brackets in any way.

Question 81 That historian is famous for his vast knowledge of primitive life (AUTHORITY)

 He is

Question82 Nobody could possibly believe the story about her achievements (BEYOND)

 The story about her achievements

Question83 I couldn't make sense of the radio message because of the interference. (IMPOSSIBLE)

The interference on the radio to make sense of the message

Question84 They arrived at the station with only a minute to spare (NICK)  They arrived at the station time

Question85 David praised her exceptionally good choice of venue for the party (CONGRATULATED)

 David a good venue for the party

Complete the unfinished sentence in such a way that it means nearly the same as the sentence printed before it.

Question86 Were Jack not so affluent a man, she would not be dating with him.  But

Question87 It is likely that they forgot about the extra class.  They

Question88 Do you have any scarves? I'm looking for one that's woolen, green and fairly long

 I'm looking for a

Question89 You should not lock this door for any reason when the building is open to the public

 Under no

Question90 A bee sting is more likely to cause death these days than a snake bite. Death

(43)

THE END ĐÁP ÁN ĐỀ SỐ 3

ANSWER KEY

Question 1: A laughter Question 2: C exacerbate Question 3: C picnics Question 4: D committee Question 5: B conversation

Question 6: C would travel Question 7: B Might Question 8: B. thick and thin

Question 9: D Having been awarded Question 10: A neatly Question 11: A. tightly

Question 12: C Even Question 13: A tunnelQuestion 14: C be seen to be taking

Question 15: D sacrificed

Question 16: A stroke

Question 17: D distínguish Question 18: B quálity

Question 19: A worldcláss Question 20: C spíritual Question 21: B. accéssible

Question 22: B Larger classes are expected to lead to poorer results but they not. Question 23: A better on standardised tests

Question 24: C more effective lesson planning

Question 25: A Class size does not affect student performance. Question 26: D is more common in Korea than in Britain

Question 27: D [1]

Question 28: A don't allow themselves much time to relax and have fun Question 29: A good discipline and a hard work ethic

Question 30: C The Asian system is obviously better

Question 31: A means [by no means = not at all] Question 32: C little

Question 33: B came Question 34: B consisted

Question 35: D off Question 36: B valuable

Question 37: D transformation Question 38: B nonetheless Question 39: C expanding Question 40: A greater

41 teaching – 42 learning – 43 liked - 44 doing – 45 depended – 46 learn

47 will never forget – 48 have ever had – 49 had not taken – 50 would not have been

(44)

61 ready – 62 and – 63 Children – 64 up – 65 lay – 66 save – 67 aware – 68 opportunities – 69 who – 70 Themselves

Question71 put Carla up Question72 coming up Question73 set up

Question74 take it up Question75 make up for Question76 calls for Question77 coming off Question78 dropped out of itQuestion79 will go up Question80 grew out of

Question 81 He is an authority on [the history of] primitive life. Question82 The story about her achievementswas beyond belief.

Question83 The interference on the radio made it impossible[for me] to make sense of the message

Question84 They arrived at the station in the nick of time.

Question85 David congratulated her on a good venue for the party.

Question86 But forJack’s being so affluent, she would not be dating with him. Question87 They probably forgot about the extra class.

Question88 I’m looking for a fairly long, green and woolen scarf.

Question89 Under no circumstances should you open this door when the building is open to the public

Question90.Death is more likely to be caused by a bee sting than by a snake bite these days

BỘ GlÁO DỤC VÀ ĐÀO TẠO CỘNG HÒA XÃ HỘI CHỦ NGHĨA VIỆT NAM TRƯỜNG ĐẠI HỌC SƯ PHẠM HÀ NỘl Độc lập – Tự – Hạnh phúc

ĐỀ THITUYỂNS1NH

VÀO TRƯỜNG TRUNG HỌC PHỔ THÔNG CHUYÊN NĂM 2015 Mơn thi: TIẾNG ANH

(Dùng cho thí sinh thi vào chuyên Anh) Thời gian làm bài: 120 phút

Mã đề số: 210

Choose the word whose underlined part is pronounced differently from that of the others.

Question 1: A laughter B caught C daughter D augment Question 2: A incline B eradicate C exacerbate D enclosure Question 3: A friends B opinions C picnics D computers

(45)

Question 5: A politeness B conversation C resolution D introduction

Choose the best answer to complete each of the following sentences.

Question 6: I'd love to have lived in the old days, when people to market by horse and carriage

A have been travelling B got used to travelling

C would travel D had been travelling

Question 7: I'll give you the phone number of my hotel so that you can reach me if anything happens .anything happen I want you lo look after my children

A Can B Might C Will D Should

Question 8: We've been together through in our friendship, and we won't desert each other now

A bad and good B thick and thin C odds and ends D spick and span Question 9: a scholarship, I entered the frightening and unknown territory of private education

A To award B Having awarded

C To be awarded D Having been awarded

Question 10: Millions of people say Coke tastes best from a bottle, and whether this is scientifically provable or not These millions know they like the look of the bottle and the way it fits so into the hand

A neatly B orderly C tidily D finitely

Question 11: Language is so woven into human experience that it is scarcely possible to imagine life without it

A tightly B tautly C rigidly D stiffly

Question 12: I'd like to help you out, I'm afraid I just haven't got any spare money at the moment

A Much as B Try as C Even D Despite

Question 13: She was very badly depressed after the car accident Now she is beginning to think that there could be light at the end of the

A tunnel B subway C passage D journey

Question 14: The government must strong measures against crime

A be seen be taking B see to be taking

(46)

Question 15: Athough she would have preferred to carry on working, my mum hercareer in order to have children

A devoted B aholishcd C repealed D sacrificed

Question 16: A number of landslides have Nepal since the 7.8-magnitude earthquake on 25thApril which killed more than 8,000 people and injured many more. A stroke B caused C hit D blown

Choose the word whose primary stress is placed differently from that of the others Question 17: A compliment B excellent C nominate D distinguish Question 18: A remember B quality C occasion D terrific

Question 19: A worldclass B wheelchair C firewood D blackmail Question 20: A volunteer B referee C spiritual D recommend Question 21: A influential B accessible C rudimentary D

incidental

Read the passage and choose the best answer to each of the questions that follow GETTING THE BEST OUT OF OUR CHILDREN

There is a strange paradox to the success of the Asian education model On the one hand, class sizes are huge by western standards with on average between 30 and 40 students per class in countries like Japan and Korea On the other hand, school children in developed Asian economies rank among the highest in the world for academic achievement in the areas of science and mathematics, especially on standardised tests Meanwhile, British secondary school students fail to shine in conditions most educational researchers would say are far more likely to help them succeed

Why Asian students seem to perform so well then? Is it their legendary discipline? Certainly, classroom management seems to be a whole lot easier in places like Korea, and perhaps lessons are more effective as a direct consequence After all, we are only too aware of the decline in discipline standards in our own schools; belligerent and disrespectful students appear to be the norm these days Teachers in Britain seem powerless to control what happens any more Surely this situation cannot create a very effective learning environment, so perhaps the number of students is far less relevant than is the manner in which they conduct themselves

(47)

stronger bonds and greater trust, and that Korean teachers, in understanding their pupils better, might be able to offer them a more effective learning programme Of course, trust and understanding leads to greater respect as well, so Korean students are probably less likely to ignore their teachers’ advice

Then there is the home environment The traditional family unit still remains relatively intact in Korea Few children come from broken homes, so there is a sense of security, safety and trust both at home and at school In Britain meanwhile, one in every two marriages fails and divorce rates are sky-high Perhaps children struggle to cope with unstable family conditions and their only way to express their frustration is by misbehaving at school Maybe all this delinquent behaviour we are complaining about is just a cry for help and a plea for attention

Butwhilethe Japanese, Korean and Asian models generally seem to produce excellent results, the statisticsdon’t tell the truth Asian sludents tend to put their education before literally everything else They very few extracurricular activities and devote farmore time to their studies than their British peers And this begs the question: is all that extra effort justified fora few extra percentagepoints in some meaninglessinternational student performance survey? So Asian students are on average 3-5% better at maths than Britons – big deal! What is their qualily of life like? Remember, school days are supposed to be the best, are they not?

There has been a lot of attention and praise given to these Asian models and their impressive statistics of late And without question, some of this praise is justified, but it seems to be a case of two extremes in operation here At one end, there is the discipline and unbelievably hard work ethic of the Asian students success in education before all else At the other end, Brilish students at times appear careless and extremely undisciplined by comparison, but at least they DO have the free time to enjoy their youth and explore their interests Is either system better outright? Or is it perhaps about time we stopped comparing and started trying to combine the best bits of both, so that we can finally offer our students a balanced, worthwhile education? We are not just dealing with statistics; never forget that every statistic is a little human being somewhere who desperately needs our help and guidance - who deserves it

Question 22: What does the writer mean when he says there is a paradox in the Asian education model?

(48)

B Larger classes are expected to lead to poorer results but they not C Asian students outperform their peers in other countries

D Class sizes in Asia are much smaller in other parts of the world Question 23: British secondary school students

A better on standardised tests B have larger class sizes

C fail at school more than they succeed D enjoy better classroom conditions Question 24: What does the writer suggest might make lessons in Korean schools more successful than in Britain?

A stricter classroom discipline B better school Boards of Management C more effective lesson planning D better teachers

Question 25: What can be inferred from the utterance perhaps the number of students is far less relevant than is the manner in which they conduct themselves? A Class size does not affect student performance

B Class size is important to maintaining control

C How students behave might be more important than class size D How teachers conduct classes affects student performance Question 26: The traditional family unit

A is unstable in Korea due to conditions in the home B is bad for children that come from broken homes

C is disappearing in Korea due to high divorce rates D is more common in Korea than in Britain

Question 27: Look at the following sentence

You see, behind those great maths and science scores, there is a quite remarkable work ethic.

Where does the sentence best fit in the paragraph?

Butwhilethe Japanese, Korean and Asian models generally seem to produce excellent results, the statisticsdon’t tell the truth [1] Asian sludents tend to put their education before literally everything else They very few extracurricular activities and devote far effortmore time to their studies than their British peers [2] And this begs the question: is all that extra effort justified fora few extra percentagepoints in some meaninglessinternational student performance survey? So Asian students are on average 3-5% better at maths than Britons – big deal! [3] What is their qualily of life like? Remember, school days are supposed to be the best, are they not?

(49)

A.[4] B [3] C [2] D [1]

Question 28: According to the wriler, Asian students A don't allow themselves much time to relax and have fun C don't have as good a work ethic as British ones

D make a big deal of their good results

Question 29: What are the 'too exlremes’ mentioned in the last paragraph? A good discipline and a hard work ethic B success and failure

C carelessness and indiscipline D neglecting school and neglecting free time

Question 30: Which conclusion about the two educational systems discussed would the author most probably agree with?

A Neither system is perfect B Both systems are quite satisfactory

C The Asian system is obviously better D The British system is too strict Read the text below and decide which answer best fits each gap The first one has been done as an example (0).

Example: (0) A beginning B first C primary D basic VOLUNTEERING

When Pamela Janett left university to become a (0) school teacher, it was by no (31) easy to find a job She therefore decided to go abroad as a volunteer teacher for a year When she realized she would be teaching deaf and blind children, she was a (32) taken aback But after a month's training she felt more confident that she would be able to cope with the situation The basic living conditions also (33) as something of a shock

(50)

Question 31: A means B extent C ways D.account Question 32: A rather B quite C little D.somewhat Question 33: A appeared B came C proved D arrived Question 34: A combined B consisted C contained D

composed

Question 35: A down B away C out D.off

Question 36: A worthy B valuable C priceless D precious Question 37: A contrast B difference C change D

transformation

Question 38: A furthermore B nonetheless C otherwise D however

Question 39: A growing B widening C expanding D stretching Question 40: A greater B larger C steeper D taller

Put each verb in brackets into a suitabie tense or form The first one has been done as an example (0).

Example: (0) took

LEARNING TO SURVIVE

Last summer I (0 take) a three-week survival course The purpose of the course was (41 teach) us how to survive outdoors, where there are no shops, no houses and no electricity I had never slept outdoors before the course, and here I was (42 learn) to make a fire, navigate and find food in the forest The part that I (43. like) the most was catching our own fish and cooking them over a fire While (44. do) .the course, I realised how much people (45 depend) .on modern technology They think that they can't survive without it but they can if they (46 learn) how I did! I (47 never forget) .that course - it was the greatest experience I (48 ever have) so far Now I think if I (49 not take) the survival course, I (50 not be) able to deal with so many difficult situations in life

41 teaching – 42 learning – 43 liked - 44 doing – 45 depended – 46 Learn - 47. will never forget –48 have ever had – 49 had not taken – 50 would not have been

Use the word given in brackets to form a word that fits in the space The first one has been done as an example (0) Example: (0.) likely

(51)

When people talk about contemporary culture they are just as (0 LIKE) to be talking about fast cars, trainers or high heels as they are to be talking about Shostakovich or Shakespeare

Goods have become as (51 MEAN) a measure and marker of culture as the Great and the Good The word "culture" can now cover just about anything Culture is no longer merely the beautiful and (52 SINGLE) … until the late twentieth century that a (53 SCHOLAR) interest in objects began to (54 PLACE)… the traditional interest in -isms, with historians, (55 LITERATE) critics and philosophers all suddenly becoming fascinated by the meaning of objects, large and small Is this a sign, perhaps, of a society cracking under the strain of too many things?

Our current (56 OBSESSIVE) with material culture, one might argue, is simply a (57 RESPOND.) to the Western crisis of abundance There are obvious problems with this (58 MATERIAL) conception of culture If our experience of everyday life is so (59 SATISFY) ., then how much more so is the (60 SPECTATOR) of our everyday things under scrutiny

51 MEANINGFUL – 52 SINGLENESS – 53 SCHOLASTIC – 54 PLACE – 55 LITERARY – 56 OBSESSION – 57 RESPONSE – 58 MATERIALISTIC – 59 SATISFACTORY – 60. SPECTATOR

Read the text below and think of the word which best fits each space Use only ONE word in each space.

The first one (0) has been done as an example.Example: (0) with

When presented (0) the idea of international boarding school, parents are (61) to baulk of emptying their bank (62) to send their darlings to live a life permeated by blackboards, bunk beds and Bovril (63) conversely, will feel overjoyed at the prospect of signing(64) for a literally fantastic life at Hogwarts Such misconceptionsneatly miss the point of internalional boarding education, sidestepping its capacity to (65) solid foundations for students' academic and professional success in today’s increasingly global society

(52)

Therefore, the time has come for parents and students (70) to discover the unique, intellectually challenging experiences offered by internalional boarding schools worldwide

61 ready – 62 and – 63 They – 64 up – 65 lay – 66 save – 67 aware – 68. opportunities – 69 who – 70 themselves

Complete each of the following sentences with one of the phrasal verbs given in the box Each phrasal verb is used only ONCE Make any necessary changes.

put up go up come off make up

for call for come up drop out of set up take up grow out

of

Question71 The hotels were all full so we offered to Carla… for the night

put Carla up

Question72 Early that morning, we set off on our journey as the sun was

coming up

Question73 An enquiry was into the use of chemicals in farming set up

Question74 You're such a good singer that you, should it professionally

take it up

Question75 How are they going to the time they wasted playing cards in the barracks?make up for

Question76 Stop wasting your time The whole situation an immediate response that could bring more

decisive effects.calls for

Question77.I like this photograph so much that I am going to have it… coming of Question78 As far as I know, the idea of the party does not quite appeal to him and that's why he's thinking

of dropped out of it

Question79 I can't stop thinking there's something more that the scheme needs will go up

Question80 Peter used to be fond of collecting mascots, but after his military service, he the hobby

(53)

Complete the second sentence using the word given so that it has a similar meaning to the following sentence Write between two and five words in the space provided on the answer sheet Do NOT change the word given in brackets in any way.

Question 81 That historian is famous for his vast knowledge ofprimitivelife (AUTHORITY)

 He is He isan authority on (the history of) primitive life

Question82 Nobody could possibly believe the story about her achievements (BEYOND)

 The story about her achievements The story about her achievementswas beyond belief.

Question83 I couldn't make sense of the radio message because of the interference (IMPOSSIBLE)

The interference on the radio to make sense of the message The interference on the radio made it impossible to make sense of the message Question84 They arrived at the station with only a minute to spare (NICK)  They arrived at the station time

They arrived at the stationin the nick of time

Question85 David praised her exceptionally good choice of venue for the party (CONGRATULATED)

 David a good venue for the party Davidcongratulated her on a good venue for the party

Complete the unfinished sentence in such a way that it means nearly the same as the sentence printed before it.

Question86 Were Jack not so affluent a man, she would not be dating with him But

But forJack’s being so affluent, she would not be dating with him Question87 It is likely that they forgot about the extra class  They

They probably forgot about the extra class

Question88 Do you have any scarves? I'm looking for one that's woolen, green and fairly long

(54)

Question89 You should not lock this door for any reason when the building is open to the public

 Under no

Under no circumstance should you open this doorwhen the building is open to the public

Question90 A bee sting is more likely to cause death these days than a snake bite Death

Death is more likely to be caused by a bee sting than by a snake bite these days Write a paragraph of about 150 words about the benefits of studying at a gifted school.

A Suggested Writing Sample

A gifted school is really good environment to study and culivate the mind We all can see that studying in a gifted school bring us many benefits.

Firstly, if you’re a student at a gifted school, you will have good conditions to develop abilities in your favorite subjects, help you to build dream and orient your future career.

Secondly, most of the teacher in the gifted school are good and enthusiatic They always create comfortable atmosphere in studying for students to receive knowledge easily.

Besides, there are many valuable scholarships for good students, especially, for students that have difficult circumstances

Finally, the gifted school always has many different facilities for students to relax after studying hard

There are many advantages of studying in a gifted school Therefore, we need to make the best use of these benefits to have the most best results. [143 words]

THE END

ĐỀ SỐ 5

UBND TỈNH QUẢNG NGÃI SỞ GIÁO DỤC VÀ ĐÀO TẠO

(55)

Môn thi: Tiếng Anh (Cho thí sinh thi vào chuyên Anh) Thời gian: 120 phút ( không kể thời gian giao đề )

Ngày thi: 10/ 7/ 2008

A pronunciation (10 points).

I Choose the word (A, B, C or D) that has the underlined part pronunced differently from the others in each group (5 points)

1 A crop B concern C conserve D protect

2 A conserve B advertise C perfect D reverse

3 A cities B workers C series D satellites

4 A food B good C tooth D tool

5 A although B without C thank D these

II Choose the word (A, B, C or D) whose main stress pattern is not the same as that of the others (5 points)

1 A cartoon B open C paper D answer

2 A manage B shortage C village D teacher

3 A remain B convenient C volunteer D memorable A maintain B marine C challenge D device A important B community C organize D diseases B VOCABULARY AND GRAMMAR (40 points).

I Choose the best answers to completes each sentence (10 points) The stolen jewels were a lot of money

A valued B cost C priced D worth

2 , after trying three times, he passed the examination

A Lastly B Last of all C Last D At last I don’t see any in arriving early at the theatre

A cause B point C reason D aim

4 The children loved the old castle

A discovering B exploring C hunting D detecting This is the oldest building the village

A of B by C in D to

6 These figures show a in the number of unemployed people in England and Wales

(56)

7 scientists have observed increased pollution in the water supply

A late B later C Latter D Lately

8 After he had broken his leg, Henry could only go up and down stairs A hardly B in difficulties C with difficulty D hard

9 You will have to your holiday if you are too ill to travel

A put off B cut down C put out D put up 10.At four o’clock Mr Hutchinson still had some to in the garden

A work B job C effort D task

II Give the correct form of the verbs in brackets (10 points)

1 This is the first time I (read) a novel (write) _ by an American novelist

2 The stereo (play) since pm I wish someone (turn) it off The new students hope (include) in many of the school’s social activities He wore a false beard (avoid) (recognize)

5 (Write) the letter, she put it carefully in an envelope

6 If you don’t hurry, all the tickets (sell) by the time we (get) there

III Fill in the blank with a / an / the Put an  where none is required (5 points) My father has just bought me _ LG computer

2 _ whale is _ mammal, not _fish We are going to _ tea with _Smiths today

4 _Titanic, one of the most famous ocean liners, sank on its first voyage across _ Atlantic

5 For centuries, _ buildings were made of _ stones

IV Give the correct form of the verbs to complete the passage (10 points) Ask any adult over forty to make a (1) _ between the

past and the present and most will tell you that things have been getting steadily worse for as long as they can remember Take the weather for example Everyone remembers that in their (2) _ the summers were considerably hotter, and that winter always included (3) _ falls of snow just when the school holidays had started Of course, the food in those days was far superior too, as nothing was imported and everything was fresh (4) _ was negligible, the money in your pocket really was worth something, and you could buy a (5) _ house even if your means were limited And above all, people were somehow nicer in those days, and spent their free time on innocent (6) _ making model boats and tending their stamp (7) _ rather than gazing at the

Compare

Young Abound

(57)

television screen for hours on end As we know, this figure of the past simply cannot be true, and there are plenty of statistics dealing with heath and (8) _ which prove that it is not true So, why is it that we all have a (9) _ to idealize the past and to be so (10) _ of the presents?

Collect Prosper Tend Critisize V Fill in each blank with a suitable prepostion (5 points)

1 It’s a small island, four miles _ the Thai coast I know her _ sight, but I’ve never talked to her

3 _ be half of everyone here tonight, I’d like to thank you _ your advice

4 You can refer _ these documents _ more information How much did he change you _ doing _ the room? She began her talk _ an apology _ the audience C READING (30 points).

I Read the passage below and choose the best answer for each question (10 points)

Over the past 600 years, English has grown from a language of few speakers to become the dominant language of international communication English as we know it today emerged around 1350, after having incorporated many elements of French that were introduced following the Norman invasion of 1066 Until the 1600s, English was, for the most part, spoken only in England and had not extended even as far as Wales, Scotland, or Ireland However, during the course of the next two centuries, English began to spread around the globe as a result of exploration, trade (including slave trade), colonization, and missionary work Thus, small enclaves of English speakers became established and grew in various parts of the world As these communities proliferated, English gradually became the primary language of international business, banking, and diplomacy

Currently, about 80 percent of the information stored on computer systems worldwide is in English Two-thirds of the world's science writing is in English, and English is the main language of technology, advertising, media, international airports, and air traffic controllers Today there are more than 700 million English users in the world, and over half of these are nonnative speakers, constituting the largest number of nonnative users than any other language in the world

1 What is the main topic of this passage ? A The number of non-native users of English B The French influence on the English language

(58)

D The use of English for science and technology

2 The word “emerged” in line could best be replaced by which of the follwing ? A appeared B hailed C frequented D engaged

3 As used in line , the word “ elements ” is not similar to which of the following : A declaration B features C curiousities D customs

4 Approximately when did English begin to be used beyond England? A in 1066 B around 1350 C before 1600 D after 1600

5 According to the passage, all of the following contributed to the spread of English around the world except

A The slave trade B Colonization C Missionaries D The Norman invasion

6 As used in line 6, which of the follwing is closet in meaning to the word “course”?

A Subject B Policy C Time D Track

7 The word “enclaves” in line could be best replaced by which of the following? A communities B organizations C regions D countries

8 The word “proliferated” in line is closest in meaning to which of the following? A prospered B organized C disbanded D expanded

9 Which of the following is closet in meaning to the word “ constituting ” in line 15 ? A looking over B sitting down C doing in D making up

10.According to the passage, approximately how many nonnative users of English are there in the world to day?

A a quarter million

B half a million C 350 million D 700 million

II These paragraphs or sentences ( A – H ) are not in a good order Rearrange them into a good order (1 – 8) to make it a logical meaningful passage (8 points)

A Anyone can try to make a drink which tastes similar to Coca-Cola Chemists say that it consists almost entirely of carbonated water and sugar

B In only a few years Coca-Cola became the national drink of the United States The shape of the famous bottle was invented in 1961 During the Second World War, American soldiers drank three million bottles of Coca-Cola a day

C Coca-Cola was originally invented by John Pemberton, a pharmacist in Atlanta, Georgia In 1886, he created a new kind of medicine to cure tiredness and headaches

(59)

E Then a businessman bought Pemberton’s recipe for his creation He did something to the recipe and made a really refreshing drink that appealed to many people Demand for the drink grew

F But no one outside the Coca-Cola company has yet discovered what the ‘magic’ combination of ingredients is, and a very few people- only seven, have ever known the secrete formula of Coca-Cola Only two of them are still alive today, so they never travel on the same aeroplane

G Today, nearly a hundred million bottles or cans are drunk every day, in one hundred and fifty-five different countries

H It tasted so revolting that just about no one wanted to buy it In the first year, Pemberton only made $50 from his creation

III Fill in each blank with one suitable word to complete this passage (12 points) LOOKING FOR A JOB

I finished university six month (1) _, I’ve got a degree in business administration I enjoyed the course very much (2) _ I realize I should have studied a lot harder! A few of my friends have (3) _ got full-time jobs but most, me, are still waiting (4) _ something suitable to turn up Meanwhile, I make sure that I keep myself busy I look through the job advertisements (5) _ the newspapers every day and I also ask all the people I know to tell me if they hear of any vacancies (6) _ they work What I am looking for is something challenging and I would certainly be happy to move to another city or even work abroad for a while The (7) _ is not so important at this stage , provided I earn enough to live on , because I don’t want to continue (8) _ to depend on my parents , although they are (9) _ generous to me At the moment, I’m working in a nearby restaurant two evenings (10) _ week, washing up and generally helping out, which brings a little money The other people working there are very friendly, and many of them are in the same (11) _ as me, so we have lots of to talk (12) _

D WRITING (20 points).

I Rewrite the sentences in such a way that they mean almost the same as those printed before them (10 points)

1 Martin may not be very well but he still manages to enjoy life

Martin’s Thanks to his aunt’s legacy of £10.000 he was able to buy the house he wanted

Had his Mary was the only student absent from class

Apart Jane hasn’t made much progress at school

(60)

5 My parents find fault with everything I

No matter It’s the cheapest watch but it’s the nicest

Not only The police didn’t at all suspect that the judge was the murder

Little “ What you have cooked is the best in the world!”, John said to his girlfriend

John congratulated Now that his mother was being there, they said nothing about it

On 10.The last storm destroyed Lan’s house

It‘s high time II Use the suggest words and phrases to write complete sentences of a passage (10 points)

1 Jack London/ famous/ American writer

2 He/ bear/ January 13th/1876/ San Francisco/ California family/ poor/ he/ have/ leave/ school/ make money He/ work/ hard/ different jobs

5 Later/ he/ return/ school/ he/ not/ stay/ long 1897/he/ go/ Alaska/ find/ gold

7 There/ he/ find/ ideas/ books / stories He/ come/ home/ start/ write

9 writings/ successful/ he/ become/ rich/ famous/ twenties 10.poor health/ he/ die/1916/ age/ 40

The end -ĐÁP ÁN ĐỀ SỐ 5

UBND tỉnh quảng ngãi Sở giáo dục - đào tạo

đáp án đề thi tuyển sinh vào lớp 10 thpt chuyên Năm học 2007 - 2008

M«n thi : TiÕng Anh

Thời gian: 120 phút ( không kể thời gian giaođề ) Ngày thi: 10/ 7/ 2008

A pronunciation (10 points).

I Choose the word (A, B, C or D) that has the underlined part pronunced differently from the others in each group (5 points) - 1/ each.

(61)

1 A B D B C

II Choose the word (A, B, C or D) whose main stress pattern is not the same as that of the others (5 points) - 1/ each.

1 A A C C C

B VOCABULARY AND GRAMMAR (35 points).

I Choose the best answers to completes each sentence (10 points) - 1/ each.

1 D D B B C

6 C D C A 10 A

II Give the correct form of the verbs in brackets (10 points) - 1/ each. have read – written

2 has been playing – would turn to be included

4 to avoid - being recognized Having written

6 will have been sold – get

III Fill in the blank with a / an / the Put an  where none is required (5 points)-0.5/ each.

1 an the a a 

6 the the the  10 

IV Give the correct form of the verbs to complete the passage (10 points) - 1/ each. 1. comparison

2. youth 3. abundant

4. Unemployment 5. sizeable

6. pursuits 7. collection 8. prosperity 9. tendency 10.critical

V Fill in each blank with a suitable prepostion (5 points) - 0.5/ each.

1 off by on for to

6 for for up with 10 to

C READING (30 points).

I Read the passage and choose the best answer to each question (5 points) - 1/ each.

1 C A B D D

6 C A D D 10 C

II These paragraphs or sentences ( A – H ) are not in a good order Rearrange them into a good order (1 – 8) to make it a logical meaningful passage (8 points)

(62)

III Fill in each blank with one suitable word to complete this passage (12 points) -1/ each.

1 ago although already for in where salary having a/per 10 very 11 situation 12 about IV WRITING (20 points).

I Rewrite the sentences in such a way that they mean almost the same as those printed before them (10 points)- 1/ each.

1 Martin’s poor health doesn’t prevent him from enjoying life

2 Had his aunt not died and left him her legacy of £10.000, he wouldn’t have been able to buy the house he wanted

3 Apart from Mary every students was present in class Jane has only made a little progress at school

5 No matter what I do, my parents find fault with it

6 Not only is it the cheapest watch but it’s also the nicest Little did the police suspect that the judge was the murder John congratulated his girlfriend on her cooking

9 On account of his mother’s being there, they said nothing about it

On account of the fact that his mother was being there, they said nothing about it On account of his mother’s presence , they said nothing about it

10.It’s high time Lan repaired her house / It’s high time for Lan’s house to be repaired

II Use the suggest words and phrases to write complete sentences of a passage (10 points)

1 Jack London was a famous American writer (1 point)

2 He was born (0.5 point) on January 13, 1876 in San Francisco, California.(0.5 point) His family was very poor, (0.5 point) and he had to leave school to make money

(0.5 point)

4 He worked hard in different jobs (1 point)

5 Later, he returned to school (0.5 point) where he didn't stay long.(0.5 point) In 1897, he went to Alaska (0.5 point) to find gold.(0.5 point)

7 There he found ideas (0.5 point) for his books and stories.(0,5 point) He came back home (0.5 point) and started to write (0.5 point)

9 His writings were successful,(0.5 point) and he became (0.5 point)rich and famous in his twenties

(63)

Tổng điểm thi : Tổng số câu đúng/ 10.

- The end

-ĐỀ SỐ 6

SỞ GIÁO DỤC VÀ ĐÀO TẠO HẢI DƯƠNG

KỲ THI TUYỂN SINH LỚP 10 THPT CHUYÊN NGUYỄN TRÃI NĂM HỌC 2012-2013

Môn thi: Tiếng Anh (Chuyên) Thời gian làm bài: 120 phút Ngày thi 20 tháng năm 2012

(Đề thi có: 04 trang) Thí sinh làm vào tờ giấy thi.

Phần trắc nghiệm: Chỉ cần viết phương án chọn A, B, C, D. Phần tự luận: Viết đầy đủ theo yêu cầu bài.

(Thí sinh khơng sử dụng tài liệu gì.)

A PHONETICS:

Choose the word that has a diferent stress pattern from the others in each group. (5 points)

1 A original B comprehensive C complementary D illustration A electrician B Japanese C possibility D comfortable

3 A category B profitable C eventually D

ordinary

4 A suspicious B embroider C logical D religion

5 A mausoleum B government C correspond D

unpolluted

B GRAMMAR – VOCABULARY – LANGUAGE FUNCTIONS:

I Choose the option A, B, C or D which best completes or responds to each sentence. (20 points)

(64)

6 He is a specialist modern Vietnamese literature

A of B at C for D in

7 David has not seen Linda for fifteen years and has of her address

A no idea B nothing C no thought D no mind

8 The population of the world is growing at an alarming

A rate B measure C step D cost

9 After Freddie school, he joined the army

A finishing B finishes C had finished D has finished 10 Don't go too fast! I can't up with you

A go B walk C run D keep

11 It was unfair of you her for something she didn't

A so as to criticize B that criticizing C to criticize D criticized

12 My father didn't like coffee and did my mother

A none B either C so D neither

13 He said that he some flowers in the garden at that moment A is watering B would water C was watering D watered 14 For the last thirty years, space exploration great contributions to weather forecasting

A has made B makes C is making D has

been made

15 Miss Trang is wearing a dress

A beautiful blue new B new beautiful blue C blue beautiful new D beautiful new blue

16 "How about going fishing this Sunday?" - " "

A That's a good idea B Never mind C Yes, I am fishing D That's my pleasure

17 He seldom takes part in any of the class activities, ?

A doesn’t he B does he C is he D isn’t he 18 If you to be a doctor, you have to study harder

A want B will want C wanted D would want

19 The cupboard in rare books are kept is near the window

A where B which C that D whom

20 Huong went on working the noise the children made

(65)

A are B have been C will be D were 22 No sooner arrived home than it rained heavily

A had he B has he been C he has D he had

23 Give me a ring before nine tomorrow morning, I’ll be too busy to talk to you

A unless B therefore C if D otherwise

24 The soldiers are made ten miles a day

A run B to run C running D ran 25 All of us are that she has won the first prize in the competition

A amazingly B amazed C amazing D amazement

II Provide the correct form of the words in brackets in the sentences below (10 points)

26 John lost his job because he often behaved (POLITE) towards his customers 27 A great deal of (SCIENCE) research has been performed in this field

28 Hoang Anh was (COURAGE) to apply for the job

29 Mr Long is a very (CONSIDER) person, so we love him very much 30 He didn’t pass the final examination due to his (LAZY)

31 Many rural areas in Viet Nam have been (URBAN) in the past few years 32 Peter fell off the ladder, but his (INJURE) were not very serious

33 The (RESIDE) in this city are very friendly

34 Those clothes look smart, but they are very (COMFORT) 35 The movie we have just watched is (EXCEPTION) good

III Choose the underlined words or phrases in each sentence below that needs correcting (5 points)

36 Ninety percent of the earthquakes occur around the Pacific Rim knows as the “Ring of Fire”

A B C D

37 Walk in the country in the evening is very pleasantat this time of year A B C D

38 Ha Noi, along with Ha Long and Hue, areamong the most popular tourist

destinations in Viet Nam A B C D

39 It’s high timethe government spend more money on education A B C D

40 Some of the people were standing in the street watched the firework display while others were singing a song

(66)

C READING:

I Read the following passage and decide which option A, B, C, D best fits each space (10 points)

Everyone wants to reduce pollution But the pollution problem is as complicated _ (41) it is serious It is complicated because much _ (42) is caused by things that benefit people For example, exhaust _ (43) automobiles causes a large percentage of all air pollution _ (44) the automobiles provide transportation for millions of people Factories _ (45) much of the material which pollutes air and water, but factories give employment to a large number of people

Thus, to _ (46) or greatly reduce pollution immediately, people would have to stop using many things that benefit them Most of the people not want to that, of course But pollution can be _ (47) reduced in several ways _ (48) and engineers can work to find ways to lessen the _ (49) of pollution that such things as automobiles and factories cause Governments can pass and enforce laws that require businesses and individuals to stop, or cut _ (50) on certain polluting activities

41 A since B because C as D for

42 A pollution B pollutant C polluter D polluting

43 A in B from C at D for

44 A Therefore B However C Moreover D But

45 A offer B discharge C emit D

dissolve

46 A increase B pause C cause D end

47 A increasingly B gradually C hopelessly D dangerously

48 A Scientists B Doctors C Lecturers D Botanists 49 A number B figures C amount D numbers

50 A down B off C up D into

II Read the following passage and fill in each numbered blank with ONE suitable word (10 points)

The Internet is made _ (51) of millions of computers linked together around the world in _ (52) a way that information can be sent from any computer to any other 24 hours a day These _ (53) can be in homes, schools, universities, government departments, or businesses The Internet is often described as a network of networks _ (54) all the smaller networks of organizations are linked together into one giant network _ (55) the Internet All computers are pretty much equal once connected to the Internet, the _ (56) difference will be the speed of the connection _ (57) is dependent on your Internet Service Provider and your own modem

(67)

III Read the following passage and choose the correct answer A, B, C, or D (10 points)

It is often said that books are always a good friend and reading is an active mental process Unlike TV, books make you use your brain By reading, you think more and become smarter Reading improves concentration and focus Reading books takes brain power It requires you to focus on what you are reading for long periods Unlike magazines, Internet posts or e-emails might contain small pieces of information Books tell the whole story Since you must concentrate in order to read, you will get better at concentration Many studies show if you not use your memory, you lose it Reading helps you stretch your memory muscles Reading requires remembering details, facts and figures and in literature, plot lines, themes and characters

Reading is a good way to improve your vocabulary Do you remember that when you were at elementary school you learned how to infer the meaning of one word by reading the context of the other words in the sentence? While reading books, especially challenging ones, you will find yourself exposed to many new words

Reading is a fundamental skill builder Every good course has a matching book to go with it Why? Because books help clarify difficult subjects Books provide information that goes deeper than just classroom discussions By reading more books, you become better informed and more of an expert on the topics you read about This expertise translates into higher self-esteem Since you are so well-read, people look to you for answers Your feelings about yourself can only get better

Books give you knowledge of other cultures and places The more information you have got, the richer your knowledge is Books can expand your horizons by letting you see what other cities and countries have to offer before you visit them

61 Books have great influence on

A muscles B brain C friendship D TV

62 When you are reading a book, A you have to read small pieces of information B you have to read during a very long time C you use your brain in concentration and focus D you lose your memory

63 A challenging book

A helps you to improve your vocabulary B is only for primary pupils

C can translate all new words D contains a lot of difficult vocabulary

64 Books

A are compulsory in every course B contain less information than class discussions

C make a sick patient feel better D are not needed in most of the courses

(68)

B enrich your vocabulary C be self-confident

D become a good builder D WRITING:

I Finish each of the following sentences in such a way that it means exactly the same as the sentence printed before it (10 points)

66 Someone serviced his car last week

→ He

_ _

67 We haven’t seen each other for years

→ The last time

68 The exercise is so difficult that we can't it

→ The exercise isn’t

69 I think that no one in my class is more intelligent than Jack

→ I think Jack

_ 70 Nga would rather stay at home and watch TV than go to the movies

→ Nga prefers

_ II Use the suggested words and phrases to write complete sentences of a passage. (10 points)

71 Jack London / be / famous / American writer

72 He/ bear / January / 13 / 1876 / San Francisco / California 73 His family / poor / he / have / leave / school / make money 74 He / work / hard / different jobs

75 Later / he / return / school / he / not / stay / long

76 1897 / he / go / Alaska / find / gold / he / find / ideas / books and stories / instead

77 He / come / home / start / write

78 Writings / be / successful / he / become / rich / famous / twenties 79 Poor health / he / die / 1916 / be / 40

III Write a letter to an imaginary friend, Minh Thu, to tell her about your neighborhood (10 points)

(69)

- Where you live(You are required to use the name: Minh Duc village or Minh Duc town)

- What your house looks like

- What you can see from your house - How far it is from your school - How you get to school

- What kinds of facilities there are in your neighborhood - What things in your neighborhood you like best Why Begin and end your letter as follows:

Dear Minh Thu,

……… ……… ……… Love,

Tuan Anh

-THE

END -Họ tên thí sinh:……… Số báo danh:……… Họ tên, chữ ký giám thị 1:………. Họ tên, chữ ký giám thị 2:………. ĐÁP ÁN ĐỀ SỐ 6

SỞ GIÁO DỤC VÀ ĐÀO TẠO HẢI DƯƠNG

ĐÁP ÁN VÀ BIỂU ĐIỂM

ĐỀ THI TUYỂN SINH LỚP 10 THPT CHUYÊN NGUYỄN TRÃI NĂM HỌC 2012-2013

Môn thi: Tiếng Anh (Chuyên)

A PHONETICS:

5.0 points: 1.0 point for each correct answer.

1 A D C C B

(70)

6 D A A C 10 D

11 C 12 D 13 C 14 A 15 D

16 A 17 B 18 A 19 B 20 B

21 D 22 A 23 D 24 B 25 B

II 10 points: 1.0 point for each correct answer. 26 impolitely 27 scientific 28

encouraged

29

considerate

30 laziness 31

urbanized/ urbanised

32 injuries 33 residents 34

uncomfortable 35

exceptionally

III 5.0 points: 1.0 point for each correct answer.

36 D 37 A 38 B 39 C 40 C

B READING:

I 10 points: 1.0 point for each correct answer.

41 C 42 A 43 B 44 D 45 B

46 D 47 B 48 A 49 C 50 A

II 10 points: 1.0 point for each correct answer.

51 up 52 such 53 computers 54.because/since/as 55 called 56

only/unique

57 which/that 58 There 59 to/ in order to/

so as to

60 both

III 10 points: 2.0 points for each correct answer

61 B 62 C 63 A 64 A 65 D

C WRITING:

I 10 points: 2.0 points for each correct sentence 66 He had/ got his car serviced last week

(71)

68 The exercise isn’t easy enough for us to

69 I think Jack is the most intelligent (student/pupil/boy/person) in my class 70 Nga prefers staying at home and watching TV to going to the movies II 10 points

71 Jack London was a famous American writer (0.5 point)

72 He was born (0.5 point) on January 13, 1876 in San Francisco, California (0.5 point)

73 His family was very poor, (0.5 point) and/so he had to leave school (0.5 point)to make money (0.5 point)

OR: Because his family was very poor, (0.5 point) he had to leave school (0.5 point)to make money (0.5 point)

74 He worked hard in different jobs.(0.5 point)

75 Later, he returned to school (0.5 point) where he didn't stay long (0.5 point) OR: Later, he returned to school, (0.5 point) but he didn't stay there long (0.5 point) 76 In 1897, he went to Alaska(0.5 point) to find gold, (0.5 point) (but) he found ideas (0.5 point) (there) for his books and stories instead (0.5 point)

77 He came back home (0.5 point) and started to write (0.5 point)

78 His writings were successful, (0.5 point) and/so he became rich and famous in his twenties (0.5 point)

OR: Because his writings were successful, (0.5 point) he became rich and famous in his twenties (0.5 point)

OR: His writings were so successful (0.5 point) that he became rich and famous in his twenties (0.5 point)

79 In poor health/Because of his poor health(0.5 point), he died in 1916 (0.5 point) when he was only forty years old (0.5 point)

III 10 points

* Form: 1.0 point

- Correct form of a letter * Task fulfillment: 4.0 points

- Complete the task with relevant information - Well-organized

(72)

- Where you live (0.5 point)

- What your house looks like (0.5 point)

- What you can see from your house (0.5 point) - How far it is from your school (0.5 point) - How you get to school (0.5 point)

- What kinds of facilities there are in your neighborhood (0.5 point) - What things in your neighborhood you like best Why (1.0 point) * Language: 5.0 points

+ Appropriate vocabulary (2.0 points) + Correct grammar (2.0 points)

+ Punctuating/ Spelling (1.0 point)

Chú ý: Tổng số điểm 100, sau qui thang điểm 10, làm tròn đến số thập phân thứ nhất.

ĐỀ SỐ 7

UBND TỈNH ĐÀ NẴNG

SỞ GIÁO DỤC VÀ ĐÀO TẠO Kỳ thi tuyển sinh vào lớp 10 THPT chuyênNăm học 2008 - 2009 Mơn thi: Tiếng Anh (Cho thí sinh vào chuyên Anh)

Thêi gian: 120 phút ( không kể thời gian giao đề ) Ngày thi: 25/ 6/ 2008

A pronunciation (10 points).

I Choose the word (A, B, C or D) that has the underlined part pronunced differently from the others in each group (5 points)

6 A table B vegetable C innovation D stage

7 A ghost B most C lost D post

8 A aviation B language C attraction D applicant A lecturer B inventor C medium D president

10.A age B college C change D altogether

II Choose the word (A, B, C or D) whose main stress pattern is not the same as that of the others (5 points)

1. A interesting B surprising C amusing D successful 2. A understand B engineer C benefit D Vietnamese 3. A applicant B uniform C yesterday D employment

(73)

4. A paper B tonight C lecture D story

5. A money B army C afraid D people

B VOCABULARY AND GRAMMAR (40 points).

I Choose the best answers to completes each sentence (10 points)

1 Kathy was as pleased as when she heard she had passed the exam A punch B a poppy C a sunflower D pound notes I haven’t had an accident yet but I’ve had a number of shaves

A narrow B near C close D tiny

3 All the of the dances went to charity

A results B proceeds C rewards D finances Enough money has been raised to the hospital’s survival

A ensure B enlarge C enable D empower

5 Many students jobs in hotesl over the summerto earn money

A low B poor C menial D inferior

6 They could find no _ of the mising car despite an extensive search A clue B remnant C indication D trace When they retired they bought a tiny in the country

A mansion B bungalow C shack D barn It is too early in the to expect many visitors to the town

A term B season C time D calendar

9 Is promoting public health as state concern ?

A necessary B necessary being C necessarily D of necessary 10.There are different kinds of music to your taste

A miss B fit C tight D suit

II Give the correct form of the verbs in brackets (10 points)

1 He no sooner (reach) the door than he (come) back I (be) very careful not to hint or remind her in any way of what she

(tell) me

3 While he(learn) to drive he (have) twenty five accidents The man at the corner table (glance) my way to see if I (listen)

5 When I (look) at my suitcase, I could see that somebody (try) to open it

III Give the correct form of the verbs to complete the passage (10 points) The duty of the police is the law and order ( maintain )

2 Our tomatoes are nicely; they’ll be ready to eat in about a week (ripe) Look at this picture of Bill and his father, you can see the clearly, can’t

you? (like)

4 He said “Good morning” in a most way (friend)

(74)

6 It’s a lovely old house, I agree, but can we afford to it? (modern) George and I have been friends since : he use to live next door (child) In winter it is important for farmers to provide food and for their

animals (warm)

9 Pele Ronaldo to take part in 2002 World Cup (courage)

10.Playing for the national team for the first time was an experience for Huynh Duc (forget)

IV There are ten mistakes in the text Identify each mistake, write it down and give your correction (10 points)

As far back as 700 B.C, man has talked about children to becared for by wolves Romulus and Remus, the legendtwin founders of Rome, were purported to have been cared for by wolves It is believed that why a she-wolf loses her litter, she seeks a human child to take its place

This seeming preposterous idea did not become credible until the late nineteenth century when a French doctor actually had founda naked ten-year-old boy wandering in the woods He did not walk erect, could not speak intelligibly, or could lie relate to people He only growled and stared of them Finally the doctor won the boy's confidence and began to work withthem After many long years of devoted and patient instruction, the doctor was able to have the boy to clothe and feed himself, recognize and utter a number of word, as well as write letters and form words

1 2 3 4 5 6 7 8 9 10. C READING (30 points).

I Read the passage below and choose the best answer for each question (5 points) Mozart, who was born on January 27, 1756 in the Austrian city of Salzburg, was neither the first nor the last child prodigy, but he was certainly the greatest He was born into a moderately prosperous family where his unmatched musical genius made itself known extremely early

Mozart began leaning to play harpsichord at three and his earliest known work was composed in 1761 when he was five, the age at which he also first appeared in public

From the age of six, when his father took him on the first foreign tour, Mozart toured the courts and musical centres of Austria, Germany, France, England, Holland, Switzerland and Italy It has been calculated that Mozart spent almost a third of his short life - he died at the age of thirty five – travelling

As Mozart matured, he continued to tour and give concerts Mozart also wrote a lot of operas His first opera, Mitridate, Re di Ponto, was performed in Milan when he was 14, and it was the first of many successes in the theatre

1 How rich was Mozart’s family?

(75)

B As rich as a financial D Very poor

2 When did he take the first step into the world of music as a composer? A When he was three

B When he was five

C When he was fourteen D When he was thirty-five What did he on his first tour to some major European countries?

A He made so much sightseeing

B He toured the courts and musical centres

C He toured only D He did nothing Why is it possible to name Mozart “A travelling composer”?

A Because he spent almost a third of his short life travelling

B Because he liked travelling a lot

C Because he never travelled D Because he had good imagination

5 What expression in the passage means “an exceptionally clever child”? A A child normal

B A child clever

C A child prodigy

D Child playing harpsichord

II Read the text and decide which answer (A, B, C or D) best fits each space (15 points)

SAVE MONEY ON THE BOOK THAT AIMS TO SAVE ANIMALS

Do you want to take part in the battle to save the world’s wildlife? Animal Watch is the book which will (1) you in the fight for survival that (2) many of our endangered animals and show how they struggle on the (3) of extinction

As you enjoy the book’s 250 pages and over 150 colour photographs, you will have the (4) of knowing that part of your purchase money is being used to (5) animals (6) from the comfort of your armchair, you will be able to observe the world’s animals close-up and explore their habitats You will also discover the terrible results of human (7) for land, flesh and skins

Animal Watch is packed with fascinating facts Did you know that polar bears cover their black noses (8) their (9) so they can hunt their prey in the snow without being seen, for example? Or that (10) each orang-utan which is captured, one has to die?

(76)

1 A combine B involve C bring D lead

2 A meets B opposes C forces D faces

3 A edge B start C limit D end

4 A satisfaction B enjoyment C virtue D value

5 A enable B help C allow D assist

6 A preserve B conserve C revive D survive

7 A greed B interest C care D concern

8 A with B by C for D from

9 A feet B claws C paws D toes

10.A for B at C from D to

11.A publicy B periodical C publication D reference 12.A imposed B impressed C persuaded D admired

13.A symbol B title C trademark D nickname

14.A beginning B preparatory C original D introductory

15.A stores B stocks C goods D funds

III Fill in each blank with one suitable word to complete this passage (10 points) Maybe you recycle cans, glass, and paper Do you know that nature recycles, too? One of the things nature (1) _ is water Water goes from oceans, lakes, and rivers into the air Water falls from the air as (2) _ or snow Rain and snow eventually find their way back to the oceans Nature’s recycling program for water is (3) _ the water cycle

The water cycle has four stages: storage, evaporation, precipitation, and runoff Water on Earth gets stored (4) _ oceans, lakes, rivers, ice, and even underground Water goes from storage into the atmosphere by a process called evaporation When water evaporates, it changes from a liquid (5) _ a gas, called water vapor Water vapor goes up into (6) _ atmosphere Water returns to the Earth as precipitation in rain or snow by changing into drops of water (7) _ the air gets cold enough Clouds are collections (8) _ water droplets Most precipitation falls into the oceans and goes right back into storage

Water that falls on land always flows from (9) _ places to lower ones This flow is called runoff Water from land flows into streams Streams join together to make (10) _and eventually the water flows into storage in the oceans Then the water cycle starts all over again

(77)

I Rewrite the sentences in such a way that they mean almost the same as those printed before them (10 points)

1 If I met the author one day, I'd ask him to sign my copy of this book

Were I He always has his nose in a book and never pays attention to what I say

If he didn't Alice and Charles did not decide to move to a bigger house until after the birth of

their second child

Only when You are under no obligation to accept their offer

You can please Martin may not be very well but he still manages to enjoy life

Martin's poor The Pacific Ocean is on average deeper than the Atlantic

The average I don't really like her , even though I admire her achievements

Much There were not nearly as many people there as I had expected

There were far There was no need for you to have gone to all that trouble

You 10.This is the best essay I have ever written

Never II Use the promps provided to write full sentences to make a complete letter (10 points)

Dear Jane,

1 Not see you / long time

2 We / really pleased / move to / country /, I / get / fed up / live / London In/ city/ much traffic / noise / no time / relax

4 In / country / life / slower / more peaceful

5 Now / example / able / a walk / with / dog / across fields / before breakfast ! Children / enjoy fresh air / make new friends

7 people/ helpful / kind / friendly

8 But we miss / friends / London ,especially you Come / see us soon

(78)

Milly, Bob, Tim and Teresa

The end -(Đề thi có 04 trang)

(79)

ĐÁP ÁN ĐỀ SỐ 7

UBND TỈNH ĐÀ NẴNG

SỞ GIÁO DỤC VÀ ĐÀO TẠO KỲ THI TUYỂN SINH VÀO LỚP 10 THPT CHUYÊNNăm học 2008 - 2009 Môn thi : Tiếng Anh

Thời gian: 120 phút ( không kể thời gian giao đề ) Ngày thi: 25/ 6/ 2008

A pronunciation (10 points).

I Choose the word (A, B, C or D) that has the underlined part pronunced differently from the others in each group (5 points) - 1/ each.

1 B C A C D

II Choose the word (A, B, C or D) whose main stress pattern is not the same as that of the others (5 points) - 1/ each.

1 A C D B C

B VOCABULARY AND GRAMMAR (35 points).

I Choose the best answers to completes each sentence (10 points) - 1/ each.

1 A C B B C

6 D B B A 10 A

II Give the correct form of the verbs in brackets (10 points) - 1/ each. had no sooner reached - came

8 was - had told was learning - had

10.glanced - was listening 11.looked - had tried

III Give the correct form of the verbs to complete the passage (10 points) - 1/ each. 1. mainternance 2. repering 3. likeness 4. friendly 5. unconving 6. modernise 7. childhood 8. warmth 9. encouraged 10.unforgottable

IV There are ten mistakes in the text Identify each mistake, write it down and give your correction (10 points)

1 to be legend why seeming had found

(80)

C READING (30 points).

I Read the passage and choose the best answer to each question (5 points) - 1/ each.

1 C B B A C

II Read the text and decide which answer best fits each space (15 points)- 1/ each.

1 B D A A B

6 D A A C 10 C

11 C 12 B 13 B 14 D 15 B

II Fill in each blank with one suitable word to complete this passage (10 points) - 1/ each.

recycles rain called in into the when/ as of high 10 rivers

D WRITING (20 points).

I Rewrite the sentences in such a way that they mean almost the same as those printed before them (10 points) - 1/ each.

1 Were I to meet the author one day I would ask him to sign my copy of this book If he didn't have his nose in a book, he might ( sometimes) pay attention to what I

say

3 Only when Alice and Charles had ( had) their second child, did they decide to move to a bigger house

4 You can please yourself (yourselves) about whether you accept their offer or not Martin's poor health doesn't stop ( prevent ) him from enjoying life

6 The average depth of the Pacific Ocean is greater than that of the Atlantic Much as I admire her achievements, I don't really like her

8 There were far fewer people there than I had expected You didn't need to have gone to all that trouble

10.Never have I written such a good essay ( a better essay than this )

II Use the promps provided to write full sentences to make a … (10 points) - 1/ each. Dear Jane,

1 We haven’t seen you for (such) a long time

2 We are really pleased that we moved to the country because we were getting so fed up with living in London

(81)

5 Now, for example, we are able to go for a walk with the dog across the fields before braekfast

6 The children are enjoying the fresh air and have made a lot of new friends The people here are helpful, kind and (ever so) friendly

8 But we miss all our friends in London, especially you (Do) come and see us soon

10.We are all looking forward to seeing you again Love,

Milly, Bob, Tim and Teresa

Tổng điểm thi : Tổng số câu đúng/ 10.

The end

-ĐỀ SỐ 8

UBND TỈNH BẮC NINH SỞ GIÁO DỤC - ĐÀO TẠO

ĐỀ THI TUYỂN SINH VÀO LỚP 10 THPT CHUYÊN Năm học 2009-2010

MƠN THI: Tiếng Anh (Dành cho thí sinh thi vào chuyên Anh)

Thời gian làm bài: 120 phút (không kể thời gian giao đề);

Ngày thi: 09/07/2009

A PHONETICS (10 POINTS)

I Choose the word (A, B, C or D) that has the underlined part pronounced diferentlyfrom the others in each group (5 points)

1 A climate B comic C hike D website

2 A worked B laughed C hoped D naked

(82)

3 A cover B category C ancient D decorate A erupt B humor C UFO D communicate A Buddhish B bomb C viable D bulb

II Choose the word ( A, B, C or D) whose main stress pattern is not the same as that of the others (5 points)

1 A comprise B depend C.design D novel

2 A tropical B collection C tendency D charity A friendliness B occasion C pagoda D deposit

4 A importing B specific C impolite D important A federation B unpolluted C disappearing D profitable

B VOCABULARY AND GRAMMAR (35 POINTS)

I Choose the best answer to complete each sentence.(10 points) All the sentences below use “the”, which is the correct one?

A Can you pass the sugar please? B The crime is a problem in many big cities C The apples are good for you D I love the skiing

2 Our friends have lived in Ho Chi Minh city _ 2002 A for B since C in D about

3 He prefers soccer tennis A from B than C to D or

4 If they worked more carefully, they _ so many mistakes A won’t make B don’t make C wouldn’t make D didn’t make

5 He _to his friend’s party when his parents asked him to go home A goes B was going C went D has gone

(83)

C we are being following D we are following Which of the following is the strongest advice?

A You should get a hair-cut B If I were you,I’d get a hair-cut C You ought to get a hair-cut D You really must get a hair-cut you mind if I use your dictionary?

A Will B Do C Can D Did

9 Mr Vo Van Kiet, _ was our former Prime Minister, was born in Vinh Long A that B whose C Who D whom

10 All that rubbish will have to be at once

A get rid of B got rid ofC got rid D getting rid of

II Give the correct form of the verbs in brackets to complete the following sentences. (10 points)

1 My friend was (make) _ (pay) _ back the book

2 Hardly he (take) up the book when the phone (ring)

3 Can you imagine what I (come) _ across when I (roll) up the carpet yesterday?

4 If she wins the prize, it(be) because she(write) very well

5 He resented (ask) (wait) _ He had expected the minister to see him at once

III Give the correct form of the word in brackets to complete each sentence.(10 points)

1 A lot of toys encourage children’s _ (imagine)

(84)

3 Burning coal is an _ way of heating a house Gas is much cheaper (economy) According to some scientists the earth is losing its outer atmosphere because of _.( pollute)

5 She has one of the biggest art in Britain ( collect)

6 has caused many so-called man-made disasters (forest)

7 often walk through the streets shouting or singing about something they sell (cry)

8 Gas and oil always increases in cold weather (consume)

9 The police are interested in the sudden of the valuable painting (appear) 10 He claimed that his _ had caused him to become a criminal (bring up)

IV Each of the following sentences has four words or phrases underlined The four underlined parts of the sentence are marked A, B, C, D You are to identify the one underlined word or phrase that should be corrected or rewritten.(5 points)

1 Preserving natural resources meanreserving them for our future A B C D

2 Despite of the increase in air fares, most people stillpreferto travel by plane A B C D

3 Regardless of your teaching method, the objective of any conversation class should be for

A B students to practicespeaking words

C D

4 Theinjured man was taken to the hospital A B C D

5 In order for one to achieve the desired results in this experiment, it is necessary that hework as fastly as possible

(85)

C READING (30 POINTS)

I The reading is followed by several questions about it There are four possible answers (A, B, C, or D) for each question Choose the best answer.(15 points)

BENJAMIN FRANKLIN

Few people can embody the spirit of early America as much as Benjamin Franklin He lived through almost the whole of the eighteenth century, being born six years after, and dying ten years before it ended In this time he saw the American colonies grow from tiny settlements into a nation, and he also contributed much to the development of the new state

At the age of 17 Franklin ran away to Philadelphia He had already received some training as a printer’s apprentice, and this helped him seven years later, with his first publication, the Pennsylvania Gazette He also received a contract to government printing work, which helped him to rise from his poor background to become a successful entrepreneur Some of his experience in business was shared in his famous Poor Richard’s Almanak, which established his reputation throughout the American colonies In another of his works, the Autobiography, which was written toward the end of his life, he shows the same quiet common sense

He was deeply interested in science and natural history, and his experiments with electricity and lightning led directly to the invention of the lightning rod He was also interested in improving the conditions of his fellow men He was involved in a number of projects in his native Philadelphia, including the setting up of a library, a university, a philosophical society, and–because he was a pragmatic man–a fire prevention service In 1753 he became Postmaster–General of the colonies Through this experience he began to develop the idea that the colonies of North America should be a single nation Later, he went to London to try to persuade the British government to change the conditions, especially the taxes, that later led the American colonists into rebellion

(86)

constitutional congress This was his last contribution, for he died later that year He is still fondly remembered by Americans as one of the creators of the United States

1 What is a good description for this text?

A An autobiography of Benjamin Franklin B The life ofBenjamin Franklin

C The works ofBenjamin Franklin D Franklin and American Independence 2 When was Benjamin Franklin born?

A 1806 B 1794 C 1717 D 1706 3 Which of these happened first?

A Franklin trains as a painter B Franklin runs away to Philadelphia C the American colonies rebel D Franklin starts his first publication 4 What was Poor Richard’s Almanak about?

A Franklin’s reputation B How to succeed in business C how to government printing D The text does not say 5 In which countries did Franklin live?

A England and France B Philadelphia and England C London and America D England, America and France

II Read the text below and think of the word which best fits each space Use only one word in each space There is an example at the beginning (0).(15 points)

SAVING THE TIGER.

(87)

left there The government had to clear twelve small villages, which means moving nearly 1,000 people and 10,000 cattle so the land (8) be handed back to nature Today, Ramthambhore is a very different place, with grass tall _(9) _ for tigers to hide in, and There are now at _(10) _ forty of them (11) the park, wandering freely about Other animals have also benefited For example, there are many _(12) deer and monkeys than before The people (13) were moved are now living in better conditions They live in new villages away (14) _ the tiger park, with schools, temples and fresh water supplies There are now sixteen such tiger parks in India and the animals’ future looks _ (15) little safer

D WRITING (25 POINTS)

I Complete the second sentence so that it has a similar meaning to the sentence printed before it, begin with the given word(s) or phrase.(20 points)

1 You should review your lessons for the exam

It’s time 2.That man used to work with me when I lived in New York

That’s 3.What a pity they close the shops at lunch- time

I wished We like ice- cream but we don’t have it every day

Although _ 5.When did you start working in that factory?

How long _ Oil was slowly covering the sand of the beach

The sand _ “ Where’ s the station car- park?” Mrs Smith asked

(88)

He regretted _ I advise you not to buy that car

If _ 10 Why don’t you ask her yourself?

I suggest that _ II Make full sentences based on given cues (5 points)

1 If you/ not go/ the party/ I / go/ either

2 Unless/ I/ have/ quiet room/ I/ not/ able/ do/ work nurse/ kind enough/ help him/ walk

4 He/ rather spend/ holidays/ a farm/ seaside I/ return/ visit/ Japan It/ wonderful trip

THE END

(Đề thi gồm 04 trang)

Họ tên thí sinh: ……… Số báo danh: ………

ĐÁP ÁN ĐỀ SỐ 8

ĐÁP ÁN CHUYÊN 10 (2009-2010)

A PHONETICS(10pts)

I Choose the word in each that has the underlined part pronounced diferently from the others in each group.(5pts)( câu viết 1đ)

1 B D C A B

II Choose the word whose main stress pattern is not the same as that of the others. (5pts)

( câu viết 1đ)

1 D B A C D

(89)

1 A B C C B B D B C 10 B I Choose the best answer to complete each sentence.(10pt)s( câu viết được 1đ)

II Give the correct form of the verbs in brackets to complete the following sentences. (10pts)

(( câu viết 1đ)

1 made - to pay came – was rolling

2 had he taken – rang will be – writes being asked – to wait

III Give the correct form of the word in brackets to complete each sentence.(10pts) ( câu viết 1đ, viết đượcc từ thiếu “s” từ khơng được tính điểm)

1 imagination products

uneconomical uneconomic

4 pollution collections

6 Deforestation Criers consumption

disappearance re -

appearance

10 upbringing

IV Each of the following sentences has four words or phrases underlined The four underlined parts of the sentence are marked A, B, C, D You are to identify the one underlined word or phrase that should be corrected or rewritten.(5pt)(mỗi câu viết đúng 1đ)

1 B A D D C

C READING (30PTS)

I The reading is followed by several questions about it There are four possible answers (A, B, C, or D) for each question Choose the best answer.(15pts)( câu viết đúng 3đ)

(90)

II Read the text below and think of the word which best fits each space Use only one word in each space There is an example at the beginning (0) (15pts) (( câu viết đúng 1đ)

1 the up to 4.so

that/which since/as/

because/for

7 were might/could enough 10 least

11 in 12 more 13 who/ that 14 from 15 a

C WRITING (25pts)

I.Complete the second sentence so that it has a similar meaning to the sentence printed before it, begin with the given word(s) or phrase.(20pts( câu viết đúng được 2đ)

1 (It’s time) you reviewed your lessons for the exam

2 (That’s) the man who used to work with me when I lived in New York (I wish) they would open the shops at lunch time/ Or (I wished) they

wouldn’t close the shops at lunch time

4 (Although) we like ice- cream, we don’t have it everyday (How long) have you been working in that factory?

6 ( The sand) of the beach was being slowly covered by oil ( Mrs Smith asked) (me) where the station car-park was

8 (He regretted) not saying/ having said goodbye to her at the airport (If I) were you, I wouldn’t by that car

10.(I suggest that) you (should) ask her yourself

II Make full sentences based on given cues (5 points) ( câu viết 1đ) If you don’t go to the party, I won’t go either

2 Unless I have a quiet room, I will not/ won’t be able to my homework/ any work

3 The nurse is /was kind enough to help him (to) walk

4 He would rather spend his holidays on a farm than at the seaside I have (just) returned from a visit to Japan It was a wonderful trip

Tổng sốđiểm 100.

(91)

Cách làm trịn điểm; ( ví dụ) 7,1  7,0 7,2; 7,3  7,25 7,4; 7,6  7,5 7,7; 7,8  7,75 7,9  8,0

ĐỀ SỐ 9

SỞ GIÁO DỤC VÀ ĐÀO TẠO KỲ THI TUYỂN SINH LỚP 10 CHUYÊN QUỐC HỌC THỪA THIÊN HUẾ MÔN: TIẾNG ANH - NĂM HỌC 2007-2008 ĐỀ CHÍNH THỨC Thời gian làm : 150 phút

(Đề thi gồm có trang - Học sinh làm đề thi này) -

Tổng điểm: Giám khảo 1: Giám khảo 2: Mã phách:

I GRAMMAR AND VOCABULARY ( 35 points) Part 1: Choose the best answer among A,B,C or D (15 points)

My mother used to ……… research in this library when she was a student

A make B C making D doing

Let’s go to the beach this weekend, ………… ?

A shall we B we C don’t we D will we

I wish he ……… that

A doesn’t say B didn’t say C hasn’t said D

hadn’t said

My father has gone away He’ll be back ……… a week

(92)

Don’t this all at once; Do it little little

A by B to C from D with

They are both good because they type carefully

A typewriters B typists C typers D typemen

We didn’t to the station in time to catch the train

A get B reach C arrive D approach

I don’t think that purple shirt with your yellow skirt

A suits B fits C goes D wears

We had to stop for gasoline at a filling

A garage B service C station D pump

10 you study harder, you won’t pass the examination

A If B Although C Despite D Unless

11 He ordered them …… it again

A don’t B not C not to D didn’t 12 Quite soon, the world is going to energy resources

A run out of B get into C keep up with D come up again 13 We decided not to go camping because of the ………rain

A great B strong C heavy D extra

14 Mary works at a supermarket She ……… $ an hour

A pays B brings C takes D earns

15 The students have got ……… news about their exams

A many B some C few D a few

Part 2: Put the verbs in brackets into the correct tense or form (10 points)

Quang (talk) to another student when I (see) him today

That proposal .( consider) by the members right now Less than half of the cans of paint .(use) up to now

(93)

B: Don’t worry I .(do) the same thing the other week but I (find) them a few days later

He is working tomorrow If he (have) a day off tomorrow, he

(go) to the beach

Part 3: Correct the words in bold (10 points)

Ex: Jane did a lot of mistakes in her composition … made…… … I’m afraid you’ve lost your test You only got 23%

………

Congratulations! You’ve gained first prize in the competitions! ………

I’ve asked everyone where Jane is, but none can tell me ………

I’m a bit short of cash Can you borrow me some until tomorrow? ………

Gills has got a good job and wins a good salary ………

Do you mind if I make a photograph of you? ………

Our school is nearby to the town centre ………

It’s so peaceful living in the nature away from the city ………

I feel asleep Do you mind if I go to bed? ………

(94)

II READING COMPREHENSION ( 35 points)

Part 1: Use one of these words in its correct form to complete the text (10 points)

retire, promote, redundant, interview, contract, reference, pensioner, applicant, career, train, employment

ADVICE TO YOUNG PEOPLE ABOUT TO START WORK

In these days of high (0) unemployment, it is often difficult for young people to find a job If they are lucky enough to be asked to go for a(n) (1)……….………… , they may find that there are at least 20 other (2) ……… ……for the job If a company is thinking of offering you a job, they will ask you for at least one (3) ……… ……from either your previous employer or someone who knows you well Before taking up your job, you may have to sign a (4) ……… You will probably have to some (5) ……… , which help you to the job more successfully Once you have decided that this is your chosen (6)………, you will then have to work hard to try and get (7) ………, which usually brings more responsibility and more money! If you are unlucky, you may be made (8) ………, and not be able to find another job It is also a good idea to pay some money into a (9) ………scheme, which will help you to look after yourself and your family when you are (10) ……… Finally, good luck!

Part 2: Read the passage below and choose the correct answer among A, B, C or D (10 points)

GREEK THEATRES

(95)

in modern theatres, so the (5) stood or sat on the slopes of the hillside Gradually, special theatres were made by building large stone or wooden steps

one (6) another up the hillside In later times, a hut was built at the far side of the acting area where it formed a background for the actors (7) the parts of the different characters Eventually, a (8) platform was built so that the actors could be seen more clearly This was the first appearance of anything . (9) our modern stage As well as these permanent theatres, there were simple wooden stages (10) around by actors wandering from one place to

another There was also a hut with curtains that served both as background scenery and as a dressing room.

A scene B performance C scenery D stage A Whole B Complete C Full D All

A In B For C At D On

A chairs B benches C seats D stools A watchers B players C viewers D audiences A among B between C around D behind A playing B making C doing D being

8. A lifted B raised C moved D pulled

A as B like C equal D similar

10 A carried B held C brought D fetched

Part 3: Read the text below and think of the word which best fits each space Use only ONE word in each space (15 points)

A POP STAR

(96)

John (3) married to Cynthia in 1962 and they had a son (4) name was Julian At that time, John was a member of a group (5) “ The Beatles” Many beautiful songs (6) written by John and wherever the group went, crowds of fans gathered to see them They (7) scream and faint when “ The Beatles” played and lots of people have their hair cut in a Beatles style Soon, everyone had heard (8) “The Beatles” and John was (9) richer than he had ever thought

Having achieved world-wide success, John started to make records (10) his own after 1968 , and it was in the same year when (11) marriage to Cynthia came to an end He had met Yoko Ono (12) he married the following year John lost weight and grew his hair long , as can (13) seen on the covers of the records that he made with Yoko He set up home in the United States and had a son called Sean

Many people considered John Lennon to be (14) most talented of all “The Beatles” He sang about peace and love and so when he was murdered by one of his fans, outside his New York apartment , the whole world (15) shocked More than 50,000 fans turned up to a ceremony in his memory He was only forty when he died

III WRITING: ( 30 points)

Part 1: There is a mistake in each sentence below Underline the mistake and write the correct sentence (10 points)

(97)

My mother, that is nearly 60, is coming to live with us

……… … … … Andrew has been a teacher of English since a very long time

……… …… It is not easy to grow up children in a modern world

……… … I think we should go some sightseeing this afternoon

……… … … The room where I am staying in is very noisy

……… … This is Peter, who his sister works at your school

……… …… Do you want to see the pictures that we took them on holiday?

……… … Have you ever considered to become a professional footballer?

……… … I’ll give Tania your letter if she will come to school tomorrow

……… 10 I’ve really got to cut up smoking

(98)

Part 2: Complete the second sentence so that it has the similar meaning to the first sentence using the word given Do not change the word given ( 10 points)

Could you possibly help me with this box? mind Would ………

“Who ate the cake?” she asked had

She wanted ……… … What sort of weather did you have on holiday? like

What ……….……

I am not good at skiing, but I’d like to learn not I do………

The exam wasn’t as hard as I expected than The exam………

Don’t forget to contact me if you come to London touch Don’t forget to………

Make sure that you don’t arrive late turn Make sure that………

Your English will improve if you keep practising get Your English………

Normally I have a ten-minute journey to school me Normally……… …

(99)

She wants……… …

Part 3: In about 250 words, write about the advantages and disadvantages of a child in a large family (10 points)

(100)

…… The end ……… ĐÁP ÁN ĐỀ SỐ 9

(101)

-

HƯỚNG DẪN CHẤM

I GRAMMAR AND VOCABULARY (35 points)

Part 1: 15 points: one point for each correct answer

Q 10 11 12 13 14 15

Key B A D D A B A C C D C A C D B

Part 2: 10 points: one point for each correct answer was talking – saw

2 is being considered has been used

4 have lost – have been looking for/have looked for– did – found had – would go

Part 3: 10 points: one point for each correct answer

1 failed 2.won nobody/ no one lend earns/gets take near/close countryside/ country 9.sleepy 10 prevent/stop

II READING COMPREHENSION (35 points)

Part 1: 10 points: one point for each correct answer

1 interview applicants reference contract training

6 career promotion redundant pension 10

(102)

Part 2: 10 points: one point for each correct answer

Q 10

Key B D C C D D A B B A

Part 3: 15 points: one point for each correct answer

1 keen / hooked at got whose called

6 were would of/about much /far 10 on

11 his 12 whom 13 be 14 the 15 was

III WRITING (30 points)

Part 1: 10 points: one point for each correct answer

1 My mother, who is nearly 60, is coming to live with us Andrew has been a teacher of English for a very long time It is not easy to bring up children in a modern world

4 I think we should some sightseeing/ go sightseeing this afternoon The room where I am staying is very noisy./ which I am staying in This is Peter, whose sister works at your school

(103)

Part 2:10 points: one point for each correct answer Would you mind helping me with this box? She wanted to know who had eaten the cake

3 What was the weather like on your holiday/when you were on holiday? I not ski very well, but I’d like to learn

5 The exam was easier than I expected

6 Do not forget to get/keep in touch with me if you come to London Make sure that you turn up in (good) time

8 Your English will get better if you keep practising Normally it takes me 10 minutes to get to school

10 She wants to meet people who are /come from other countries Part 3: 10 points

Total: 100 points

ĐỀ SỐ 10

UBND TỈNH VŨNG TÀU SỞ GIÁO DỤC - ĐÀO TẠO

KY THI TUYỂN SINH VÀO LỚP 10 THPT CHUYÊN Năm học 2009 - 2010

Môn thi: Tiếng Anh (Cho thí sinh thi vào chuyên Anh)

Thời gian: 120 phút ( không kể thời gian giao đề ) Ngày thi: 25/ 6/ 2009

A pronunciation (10 points).

I Choose the word (A, B, C or D) that has the underlined part pronunced differently from the others in each group (5 points)

(104)

11.A author B other C there D they

12.A forest B control C product D ecology

13.A industry B translate C construction D satisfy

14.A baggage B courage C damage D invasion

15.A advantage B addition C advertise D adventure

II Choose the word (A, B, C or D) whose main stress pattern is not the same as that of the others (5 points)

A deposit B festival C institute D resident A surprising B astonishing C amazing D interesting A interview B industry C essential D difficult

A extreme B generous C lonely D clothing

A friendly B extra C along D orphanage

B VOCABULARY AND GRAMMAR (40 points).

I Choose the best answers to completes each sentence (10 points)

1 One problem for teacher is that each student has his/ her own needs A separate B divided C individual D distinctive I was in the book I was reading and didn’t hear the phone

A submerged B distracted C gripped D engrossed If we hurry, we might get there to catch the early train

A right B in time C on time D before time To get a passport, you must send in your birth and two photos

A certificate B licence C paper D card She tries to set an hour a week for practice

A about B down C aside D in

6 stay the night if it’s too difficult to get home

A At all costs B By all means C In all D On the whole Robert and his wife to my house for tea yesterday evening

A came round B came about C came down D came away Each of the guests a bunch of flowers

(105)

9 To everyone’s surprise, Mr Brown at the Trade Union meeting

A turned in B turned over C turned up D turned round 10 Everyone burst out laughing There was a lot of

A laughs B laughter C laughings D laugh II Give the correct form of the verbs in brackets (10 points)

1 I (meet) John before I (be) a fortnight in Paris

2 The children were frightened because the lights suddenly (go) out and they (sit) in the dark

3 What tune (play) when we (come) in?

4 She was badly hurt when her car hit another car If she (wear) her seat belt, she (not hurt) so badly

5 Coming into the room he (see) Mary where he (leave) her III Give the correct form of the verbs to complete the passage (10 points)

1 We go to the restaurant for whenever we ‘re tired of eating meat (vegestable)

2 Don’t worry I ‘ll be waiting for you at the to the pagoda (enter) The old theater of our city is being enlarged and (modern)

4 Advertisements acount for three fourths of the of the newspaper (come) What is the of the adjective “solid” ( oppose)

6 should complain if they are not satisfied with the service they recieve (Consume)

7 If your work is you won’t get a raise (satisfy)

8 are alarmed by the rate at which tropical rainforest are being destroyed (Conserve)

9 The athletes take part in the Olympic Games in the true spirit of (sport) 10.You can never be sure what he is going to He is so (predict)

IV Arrange these sentences to make a complete passage (10 points).

a As they built bigger and bigger ships, they were able to sail further and further away b They don’t really mean that the world is getting smaller, of course

c They found that as they sailed further away from their home countries, the world became bigger

d Then men began to use ships to explore the world

e Today there are no undiscovered countries left to put on the map, but there are many new worlds to discover out in space

f But 2,300 years ago the Egyptians and the Greek thought that the world was very small and flat that around it was the ocean

g We often hear people saying that the world is getting smaller

(106)

i And they discovered new countries that they didn’t know about

j What they mean is that with modern means of transport, we can now reach every part of the world very quickly

C READING (30 points).

I Read the passage below and choose the best answer for each question (5 points) People have been playing with marbles for thousands of years The first marbles were probably either river stones that happened to be naturally round enough to roll or, more likely, rounded globs of clay that were baked hardness Such very old clay marbles have been found in both Greek and Roman ruins, and quartz spheres have been dated at around 6000 B.C Harder and more durable marbles tend to inspire different kinds of games than soft clay marbles, which crack very easily So with the advent of hand rounded and polished marbles made of agate or some other rugged, igneous rock, the ‘golden age’ of marbles and marble play flowered Stone marbles began to appear in the early 1800s in what is now the southern part of Germany Shortly after, handmade glass marbles appeared in the same part of Europe For the next 120 years, marbles and marble playingthere were literally hundred of games -flourished in both Europe and America Marble players developed their own vocabulary for different sizes and materials of marbles, as well as for the many kinds of games to be played and the way marbles were used in the games For example, if you were going to play a game of Ring-Taw, one of the most popular and enduring marbles games, you would lag for the first shot, and then knuckle down from the baulk, trying your best to get a mib or two with your opponent’s immie

1 The author makes the point in the passage that playing with marbles A has been going on since ancient times

B is a relatively recent phenomenon

C is losing popularity

D is a very expensive pastime

2 According to the passage, which of the following was the least used substance for making marbles?

A agate B glass C rock D clay

3 It can be inferred from the passage that the use of marbles became very popular in Europe and America

A in the 18th century B in the 1970s

C in 6000 B.C

D after glass marbles were developed We can infer from the passage that marble playing

A is a game only for children B has many variations in games

(107)

D uses only one kind of marble

5 We can conclude from lines 11-12 of the passage that the terminology of marble playing is

A specialized

B used only by children

C easy to understand

D derived from an ancient language

II Read the text and decide which answer (A, B, C or D) best fits each space (15 points)

Look on the bright side

Do you ever wish you were more optimistic, some one who always (1) _ to be successful? Having some one around who always (2) _ the worst isn’t really a lot of (3) _ we all know someone who sees a single cloud on a sunny day and says,b “It looks (4) _ rain.” But if you catch yourself thinking such things, it’s important to something (5) _ it You can change your view of life, (6) _ to psychologists, it only takes a little effort, and you’ll find life more rewarding as a (7) _ Optimism, they say, is partly about self-respect and confidence, but it’s also a more positive way of looking at life and all it has to (8) _ Optimists are more (9) _ to start new projects and generally more prepared to take risks

Upbringing is obviously very important in forming your (10) _ to the world Some people are brought up to (11) _ too much on others and grow up forever blaming other people when anything (12) _ wrong Most optimists, on the (13) _ hand, have been brought up not to (14) _ failure as the end of the world-they just (15) _ with their lives

(108)

15.A get up B get on C get out D get over III Fill in each blank with one suitable word to complete this passage (10 points)

Travelling to all corners of the world is (1) _ easier and easier We live (2) _ a global village, but how well we know and understand each other ? Here is (3) _ simple test Imagine you have arranged a meeting at p.m What time should you expect your foreign business colleagues to arrive? If they are German, they ‘ll be exactly (4) _ time If they are American, they ‘ll probably be 15 minutes early If they are Bristish, they be 15 minutes (5) _, and you should allow up to an hour for the Indians Therefore, these are some small advice in (6) _ not to behave badly abroad In France you shouldn’t sit down in a cafard untill you are shaken hands with everyone you know.In Afghanistan you ‘d better spend at last minutes (7) _ hello In Pakistan you mustn’t wink It is offensive In the Middle East you must never use the left hand for greeting, eating, drinking or smoking Also, you should care not to admire (8) _ in your host’s home They will feel that they have to give it to you.In Russia you must your hosts drink for drink or they will think you are unfriendly In Thailand you should clasp your hands (9) _ and lower your head and your eyes when you greet someone In America you should eat your hamburger with both hands and as quickly as (10) _ You shouldn’t try to have a conversation until it is eaten

D WRITING (20 points).

I Rewrite the sentences in such a way that they mean almost the same as those printed before them (10 points)

1 My brother studies now harder than he used to My

brother Please don’t ask me that question

I’d

rather I last saw Bob when I was in Ho Chi Minh City

I

haven’t

(109)

John could

hardly Tom will be twenty next week

It’s

Tom’s She is fond of her nephew although he behaves terribly

She is fond of her nephew

in He prefers reading books to watching TV

He

would He didn’t hurry, so he missed the bus

If

9 The train takes longer than the plane The

plane 10.I don’t suppose you have change for pounds, you?

Do you

happen II Use the promps provided to write full sentences to make a complete letter (10 points)

Dear Sir / Madam,

1 I / like / express / concern / increasing number / karaoke bars / city There be / lot / reason / I object / places

3 Firstly, / owners / take / much money / those / come / sing Secondly, / they cause / much noise / neighborhood

5 Thirdly, / there / number / pupils / play truant / just / go / those places / sing

6 Last / least / these bars / harm / appearance / city / because / their / ugly flashing lights

7 I want / say / I / not / old fashioned person

8 I hope / authority / take / matter / careful consideration

9 I / not mean / ban them / but / there should / effective way / control / this kind / entertainment places

(110)

Yours truly, Thomas Cruise

The end -(Đề thi có 04 trang)

Họ tên thí sinh: Số báo danh: ………

ĐÁP ÁN ĐỀ SỐ 10

UBND TỈNH VŨNG TÀU SỞ GIÁO DỤC - ĐÀO TẠO

ĐÁP ÁN TUYỂN SINH VÀO LỚP 10 THPT CHUYÊN Năm học 2009 - 2010

Mơn thi: Tiếng Anh (Cho thí sinh thi vào chuyên Anh)

Ngày thi: 25/ 6/ 2009

A pronunciation (10 points).

I Choose the word (A, B, C or D) that has the underlined part pronunced differently from the others in each group (5 points) - 1/ each.

1 A B B D C

II Choose the word (A, B, C or D) whose main stress pattern is not the same as that of the others (5 points) - 1/ each.

1 A D C A C

B VOCABULARY AND GRAMMAR (35 points).

I Choose the best answers to completes each sentence (10 points) - 1/ each.

1 C D B A C

6 B A B C 10 B

II Give the correct form of the verbs in brackets (10 points) - 1/ each. 12.met – had been

13.had gone – were sitting 14.was being played – came

15.had been wearing – wouldn’t have been hurt

16.saw – had left

III Give the correct form of the verbs to complete the passage (10 points) - 1/ each.

1 vegetarians 17.consumers

(111)

2 entrance untisfactory

3 modernized Conservationists

4 incomes sportsmanship

5 opposite 10.unpredictable

IV Arrange these sentences to make a complete passage (10 points). g  b  j  f  d  c  i  a  h  e

C READING (30 points).

I Read the passage and choose the best answer to each question (5 points) - 1/ each.

1 A D D B A

II Read the text and decide which answer best fits each space (15 points)- 1/ each.

1 B C D D C

6 B A C B 10 B

11 C 12 A 13 C 14 A 15 B

II Fill in each blank with one suitable word to complete this passage (10 points) - 1/ each.

getting in a on late

order saying anything togethet 10 possible D WRITING (20 points).

I Rewrite the sentences in such a way that they mean almost the same as those printed before them (10 points)

1 My brother didn't use to study as/ so hard as he does now/ used to study more lazily than he does now

2 I’d rather you didn’t ask me that question

3 I haven’t seen Bob since I was in Ho Chi Minh City John could hardly understand what the teacher said It's Tom's twentieth birthday next week

6 She is fond of her nephew in spite of his terrible behaviour He would rather read books than watch TV

8 If he had hurried he would/could have caught / wouldn’t have missed the bus

9 The plane does not take as long as/takes a shorter time than/ is quicker/faster than the train

10.Do you happen to have change for pounds?

(112)

Dear Sir/ Madam,

1 I would like to express (my) concern about the increasing number of karaoke bars in the/ our city

2 There are a lot of reasons that/ why I object to these/ those places

3 Firstly, the owners take too much money from those people who come to sing Secondly, they cause/ are causing too much noise in/ to the neighborhood

5 Thirdly, there are a number of pupils who/ that play truant just to go to those places to sing

6 Last but not least, these bars harm to the appearance of the city because of their ugly flashing light

7 I (also) want to say (that) I am not an old fashioned person

8 I hope (that) the authority will take this matter into careful consideration

9 I not mean to ban them, but there should be an/ some effective way to control this kind of entertainment places

10.I look/ am looking forward to seeing the city council doing something about this matter

Yours truly, Thomas Cruise

Tổng điểm thi : Tổng số câu đúng/ 10. THE END

ĐỀ SỐ 11

SỞ GIÁO DỤC & ĐÀO TẠO KỲ THI TUYỂN SINH VÀO LỚP 10 – THPT CHUYÊN LAM SƠN

THANH HOÁ Năm học 2004-2005

Đề thức Mơn:TIẾNG ANH

Thời gian: 150 phút ( không kể thời gian giao đề.) Question 1: Supply the correct form of the verb in brackets.

1 It …… (be) my birthday tomorrow

2 I …… (not play) football since I …… (break) my leg

3 I …… (telephone) Bill as soon as I …… (get) home this evening

(113)

5 I….(think) your father’s health… (improve) so much since I….(see) him last

Question 2: Fill each blank with the correct form of the word given in capital letters 1 ACT

a We must take ………… before things get worse

b Don’t worry about the volcano It’s been ………… for years c She said she wanted to be a television ………

2 BASE

a My grandfather only had a very ……… education b The organization is run on a voluntary ……… 3 CONTINUE

a His latest book is a ………… of his previous one

b The train service was ………… because it wasn’t used by many people

c Peter couldn’t get much work done as he was ………… being interrupted by people telephoning him

4 COMPARE

a I’m just a beginner in …… with her

b What happened two years ago is not really …… in the situation now

Question 3:Fill each of the numbered blank spaces with the most suitable preposition. There has been a university in Oxford …… more than eight hundred years

2 They have been married ………2001

3 From 1992 …… last year we had a flat in the center of town Can you wait for a few minutes ….… I’m ready ?

5 I haven’t spoken to Bill ….… we were at school

Question 4: Fill each of the numbered blanks in the following passage Use only ONE word in each space.

A good memory is a great (1) …… in learning a language Everybody learns his own language by …….2…… what he hears when he is a small child, and some children like boys and girls who live ……3…… with their ……4…… seem to learn …….5…… languages almost as easily as …… 6…… In school, it’s not so …….7…… to learn a second language because the pupils have so little ……8…… for it, and they are busy ……9… other ……10…… as well

The best ……11…… for most of us to remember things is to join them in our mind with something ……12…… we know already, or which we easily remember because we have a picture of it in our ……13…… That is why it is ……14…… to learn words in sentences, not by ……15…… , or to see, or to do, or to feel what a word means when we first use it…

(114)

a- Choose the word or phrase that best completes each unfinished sentence below. 1- Each birthday is an occasion for various members of the family and friends …

(together / to gather / to show )

2- The essay …( reminds/ remembers / makes ) him of his family members and friends 3- … (Rush-hours / Traffic-lights / Traffic-jams) usually happen in the busiest areas of

the city

4- We heard the birds … (sang / singing / to sing ) a beautiful song 5- He tries to avoid … (took / take / taking ) his lesson

b- Correct the mistake in the following sentences. Here she comes the new employee

2 She isn’t thought she is a nine year old girl Can you remember when did he left ?

4 We bought a new furniture for the living-room We saw several kinds birds at the wildlife preserve

Question 6: Finish each of the following sentences in such a way that it means exactly the same as the sentence printed before it.

1 Nobody has heard of John since he went to live in America John ………

2 It was dark when we arrived because the journey took a very long time ……… so ……… that ………

3 “You’ve passed your driving test, Ron! Well done !” said Carol Carol congratulated ………

4 He felt ill to get up

He did not ………

5 I was sitting next to a boy in the exam He told me the answers The boy I ………

6 The days get colder and colder Each day ………

7 I thought that learning to drive would be difficult, but it isn’t Learning to drive is ………

8 If I take the job I’ll have to move to London Taking the job will ……… A week ago today we were in China

This ………

10 Mary hasn’t finished her work yet Mary is ………

(115)

Chữ ký giám thị:

ĐÁP ÁN ĐỀ SỐ 11

SỞ GIÁO DỤC & ĐÀO TẠO ĐÁP ÁN - HDC ĐỀ THI TUYỂN SINH VÀO LỚP 10 – THPT LAM SƠN

THANH HỐ Năm học 2004-2005

Mơn: TIẾNG ANH( Đề thức ) A- ĐÁP ÁN

Question 1:( 10 points ) 1 is

2 have not played / broke 3 will telephone / get 4 had just gone / rang

5 think / has improved / saw Question 2: ( 10 points )

1 a- action b inactive c actress

2 a basic b basis

3 a continuation b discontinued c continually 4 a comparison b comparable

Question 3: ( points ) for

2 since until until since

Question 4: ( 15 points )

1 help 6 one 11 way

2 remembering 7 easy 12 which

3 abroad 8 time 13 mind

4 parents 9 with 14 better

5 two 10 subjects 15 themselves Question 5: ( 10 points )

(116)

3- traffic-jams 4- singing 5- taking

b 1- Here she comes … Here comes… (without she)

2- She isn’t thought she is … She isn’t thought to be … 3- … when did he left ? … when he left ? (without did ) 4- We bought a new … We bought new … (without a) 5-We saw several kinds birds … We saw several kinds of birds …

Question 6:( 10 points )

1 John has not been heard of since he went to live in America The journey took so long that it was dark when we arrived Carol congratulated Ron on passing his driving test

4 He did not feel well enough to get up

5 The boy I was sitting next to in the exam told me the answers Each day it gets colder

7 Learning to drive is not as difficult as I thought it would be Taking the job will mean moving to London

9 This time last week we were in China 10 Mary is still working

ĐỀ SỐ 12

Sở Giáo dục & Đào tạo kỳ thi tuyển sinh vào lớp 10 – THPT chuyên lam sơn

Thanh hoá Năm học 2005-2006

Đề dư bị Môn:Tiếng Anh

Thời gian: 150 phút ( không kể thời gian giao đề.) Question 1: Supply the correct form of the verb in brackets

1 A lot of people (kill) by aids recently, and I wish nobody (die) any more

2 He kept (ring) up and (ask) for an explanation and she didn’t know what (do) about him

3 + You look exhausted

+ Yes, I (play) tennis, and I (not play) for many years so I’m not used to it since his wife (die) he (take) to (smoke) a lot

Question 2: Fill each blank with the correct form of the word in brackets

(117)

and pitch the tent Those who like to be …5…(adventure) can go hiking in the mountains Others looking for a …6…(please) and relaxing holiday, will find it by the sea Camping is also an …7…(economy) alternative to hotels and …8…(expense) restaurants In addition, cooking can be fun on the …(tradition) camp - fire It’s not so surprising then, that camping is gaining in …10…(popular)

Question 3:

a- Choose the word or phrase that best completes each unfinished sentence below. 1- Each birthday is an occasion for various members of the family and friends …

(together / to gather / to show )

2- The essay …( reminds/ remembers / makes ) him of his family members and friends 3- … (Rush-hours / Traffic-lights / Traffic-jams) usually happen in the busiest areas of

the city

4- We heard the birds … (sang / singing / to sing) a beautiful song 5- He tries to avoid … (took / take / taking) his lesson

b- Correct the mistake in the following sentences. 1- Here she comes the new employee

2- She isn’t thought she is a nine year old girl 3- Can you remember when did he left ?

4- We bought a new furniture for the living-room 5- We saw several kinds birds at the wildlife preserve

Question 4: Fill each of the numbered blanks in the following passage Use only ONE word in each space.

Television is one of man’s most important …1… of communication It brings …2… and sounds from the …3… into millions of homes A person with a TV set can sit in his house and …4… the President …5… a speech or visit a foreign country He can see a war being fought and watch statement try to bring about peace …6… TV, home viewers can see and learn about people, places and things in faraway lands

In …7… to all these things, TV …8… its viewers a steady streams of programmes … 9… are designed to entertain In fact, TV provides many …10…entertainment programmes than any other kind

Question 5: Fill each of the numbered blank spaces with the most suitable preposition.

1 Since graduating ……… school she has got very interested ……… environmental issues

2 I’m going to discuss it ……… them tomorrow Don’t laugh ……… her or she’ll get angry

(118)

5 Foreigners who visit my country are always impressed ……… how polite and friendly everyone is

6 I dreamed ………… prehistoric monsters last night Everyone praised him ……… doing so well on the exam The river divides the city …… two parts

Question 6: Finish each of the following sentences in such a way that it means exactly the same as the sentence printed before it.

1 The cake is so hard that I can’t eat it

It is ………

2 Because of the bad weather, we had to stay at home Because the ………

3 “You’ve passed your driving test, Ron! Well done !” said Carol Carol congratulated ………

4 He felt ill to get up

He did not ………

5 I was sitting next to a boy in the exam He told me the answers The boy I ………

6 The days get colder and colder Each day ………

7 I thought that learning to drive would be difficult, but it isn’t Learning to drive is ………

8 If I take the job I’ll have to move to London Taking the job will ……… They say the doctor has made a serious mistake

The doctor ……… 10.I prefer tennis to football

I ‘d rather ………

ĐÁP ÁN ĐỀ SỐ 12

Sở Giáo dục & Đào tạo đáp án - hdc đề thi tuyển sinh vào lớp 10 – THPT lam sơn

Thanh hoá Năm học 2005-2006

Môn: Tiếng Anh( Đề dự bị ) A- Đáp án

(119)

1 have been killed / would die 2 ringing / asking / to do

3 have been playing / haven’t played 4 died / has taken / smoking

Question 2: ( 10 points )

1 ideal 2 endless 3 responsibility (ies) 4 Suitable 5 Adventurous 6 pleasant 7 economical 8 expensive 9 traditional 10 popularity

Question 3: ( 15 points ) a 1- to gather

2- reminds 3- traffic-jams 4- singing 6- taking

b 1- Here she comes … Here comes… (without she)

2- She isn’t thought she is … She isn’t thought to be … 3- … when did he left ? … when he left ? (without did ) 4- We bought a new … We bought new … (without a) 5-We saw several kinds birds … We saw several kinds of birds …

Question 4: ( 15 points ) 1 forms 6 through 2 pictures 7 addition 3 world 8 brings 4 watch 9 which

5 make 10 interesting Question 5: ( 10 points )

1 from / in with at for / to by

6 about (of) for

8 into

Question 6:( 10 points )

1 It is such a hard cake that I can’t eat it.

(120)

4 He did not feel well enough to get up.

5 The boy I was sitting next to in the exam told me the answers. 6 Each day it gets colder.

7 Learning to drive is not as difficult as I thought it would be. 8 Taking the job will mean moving to London.

9 The doctor is said to have made a serious mistake. 10 I’d rather play tennis than football.

I- Fill each of the numbered blanks in the following passage Use only ONE word in each space.

A good memory is a great ……1…… in learning a language Everybody learns his own language by …….2…… what he hears when he is a small child, and some children like boys and girls who live ……3…… with their ……4…… seem to learn …….5…… languages almost as easily as …… 6…… In school, it’s not so …….7…… to learn a second language because the pupils have so little ……8…… for it, and they are busy ……9… other ……10…… as well

The best ……11…… for most of us to remember things is to join them in our mind with something ……12…… we know already, or which we easily remember because we have a picture of it in our ……13…… That is why it is ……14…… to learn words in sentences, not by ……15…… , or to see, or to do, or to feel what a word means when we first use it…

help one 11 way

2 remembering 7 easy 12 which

3 abroad 8 time 13 mind

4 parents 9 with 14 better

5 two 10 subjects 15 themselves II- Word-formation

1 ACT

a We must take ………… before things get worse b There’s a lot of ………… outside the stadium

c Don’t worry about the volcano It’s been ………… for years d She said she wanted to be a television ………

2 BASE

a My grandfather only had a very ……… education b The organization is run on a voluntary ……… 3 CONTINUE

a His latest book is a ………… of his previous one

b The train service was ………… because it wasn’t used by many people

(121)

d After four hours’ ……… typing I had a terrible headache 1 a- action b activity c inactive d actress 2 a basic b basis

3 a continuation b discontinued c continually d continuous III- Rewrite

1 Nobody has heard of John since he went to live in America John ………

John has not been heard of since he went to live in America. It was dark when we arrived because the journey took a very long time ……… so ……… that ………

The journey took so long that it was dark when we arrived. “You’ve passed your driving test, Ron! Well done !” said Carol

Carol congratulated ………

Carol congratulated Role on passing his driving test. He felt ill to get up

He did not ………

He did not feel well enough to get up.

5 I was sitting next to a boy in the exam He told me the answers The boy I ………

The boy I was sitting next to in the exam told me the answers. The days get colder and colder

Each day ………

Each day it gets colder.

7 I thought that learning to drive would be difficult, but it isn’t Learning to drive is ………

Learning to drive is not as difficult as I thought it would be. If I take the job I’ll have to move to London

Taking the job will ………

Taking the job will mean moving to London. A week ago today we were in China

This ………

This time last week we were in China. 10 Mary hasn’t finished her work yet

Mary is ……… Mary is still working IV- Verb-form

1 It was ten o’clock and I still (not finish) my homework It (be) my birthday tomorrow

(122)

ĐỀ CHÍNH THỨC

4 I (telephone) …… Bill as soon as I (get) …… home this evening

5 It was eleven o’clock and we (just go) …… to bed when the telephone (ring)…… I (think) your father’s health (improve) so much since I (see) him last

7 This tooth (kill) …… me lately! So I (make) …… an appointment with the dentist for Tuesday

8 It says in the paper that they (discover) … oil in Wales They (look for) …… it for ages 1 had not finished

2 is

3 have not played / broke 4 will telephone / get 5 had just gone / rang

6 think / has improved / saw 7 has been killing / have made

8 have discovered / have been looking for V- Prepositions

1 There has been a university in Oxford ……for…… more than eight hundred years They have been married ……since…… 1966

3 The first World War lasted ……from….1914 ……until… 1918

4 ……from…… 1992 ……until…… last year we had a flat in the center of town Can you wait for a few minutes …until… I’m ready

6 I haven’t spoken to Bill ……since… we were at school

7 We usually stop for lunch ……from……one ……until…… two thirty It has been raining ……since…… early this morning

9 It’s nearly five years ……since… Jenny left school

10 She was at college …for… two years and she’s been working here …for… almost three years

ĐỀ SỐ 13

SỞ GIÁO DỤC VÀ ĐÀO TẠO KÌ THI TUYỂN SINH LỚP 10 THPT CHUYÊN

HẢI PHÒNG NĂM HỌC 2009 - 2010

MÔN THI: TIẾNG ANH

Thời gian làm bài: 120 phút (không kể thời gian giao đề) CHÚ Ý: -Thí sinh làm vào phiếu trả lời Đề không thu lại.

(123)

Họ tên thí sinh:

Số báo

danh:

Giám thị

1:

Giám thị

2:

Part Choose the word whose bold part is pronounced diferently from that of the others in each group

Write your answer (A, B, C or D) on the answer sheet (5 pts) A checked B backed C naked D cooked A theatre B bathing C breathe D southern

3 A tone B shone C bone D alone

4 A cholera B character C mechanic D charity A houses B horses C rises D closes

6 A here B health C honour D heat

7 A break B meant C breakfast D head

8 A gift B get C game D gentle

9 A fool B bamboo C food D flood

10.A psychology B photography C geology D classify

Part Choose the word whose stress position is diferent from that of the others in each group

Write your answer (A, B, C or D) on the answer sheet (5 pts) A expectation B introduction C volunteer D foundation A extensive B exciting C excellent D existence A successful B wonderful C beautiful D plentiful A university B understand C geography D biological A mechanic B official C preference D convenience A residential B volunteer C Vietnamese D dramatic

7 A refer B suffer C enter D offer

8 A professional B environmental C responsibility D mathematician A constant B control C contrary D concert

(124)

Part Choose the best word or phrase to complete each of the sentences below Write your answer (A, B, C or D) on the answer sheet (10 pts)

1 He always takes full of the mistakes made by these competitors A advantage B benefit C profit D advice

2 The girl must be very _ to draw such a beautiful picture

A imaginative B imaginable C imaginary D imagination Everyone in the house woke up when the burglar alarm

A got out B went off C put down D set out

4 Some of the survivors spoke to reporters about their in the air crash A occasion B happening C event D experience

5 Bill has _ a very good idea for the next class party

A looked up to B come up with C gone in for D made up of No one helped him; he did it _

A himself all B all by himself C by all himself D by himself all Many of the minerals near the Earth’s surface exist in small

A numbers B amounts C amount D number

8 She wondered her father looked like now, after so many years away

A how B whose C what D that

9 Alice Freeman, president of WellesleyCollege at the age of 27, is one of the youngest presidents in history

A who is appointed B who was appointed C that is appointed D whom has been appointed

10.Tom is younger than Jane

A very B many C quite D much

11.Of the two drivers, Mr Blake is very experienced and _

A the others are not B the other is not C another is not D other is not

12.The weekend football match was cancelled _ the heavy rain

(125)

13.It’s urgent that the boss _ a meeting to discuss the problem

A called B calls C to call D call

14.Either you or Daisy _ the vase

A has broken B have broken C has been broken D have been broken 15. Mary, give her my best regards

A If you had seen B Do you see C By seeing D Should you see 16.In the US, the states but Hawaii is an island

A neither of B none of C no of D all of 17.I'm a bit busy right now, can you _?

A recall me B return my call C rephone D call back 18.Now, boys Just watch me, and _ I tell you

A whatever B whoever C however D whenever 19.The Olympic Games are held

A four years once B each four years C every four years D once four years

20. _ I admire his intelligence, I can’t stand his stubbornness A Much as B Whatever C Despite D As long as

Part Choose the word or phrase which best fits each of the numbered blanksin the passage below

Write your answer (A, B, C or D) on the answer sheet (5 pts)

In this age of -(1) - telephone networks and electronic mail, it seems that fewer and even fewer people are taking time to sit down and write letters -(2) - friends and relatives For hundreds of years, letters were the only way to keep -(3) - with people who were any distance away and letter-writing was seen as an important skill for all learned people -(4) -

(126)

letter, an employee who sends out long letters is often regarded as -(8) - Many people prefer the telephone in all circumstances and its speed is essential in many situations but -(9) - have you put the telephone down, dissatisfied with what you have managed to say? I don’t think I’ll throw my -(10) - away yet

1 A progressive B advanced C highly-developed D all are correct

2 A to B from C for D with

3 A in step B in contact C on good terms D up

4 A to be mastered B mastering C to master D mastered

5 A therefore B however C in short D for example

6 A make B take C cause D create

7 A Even though B As though C Despite D However A unimportant B inefficient C unusual D impossible A how often B how long C how much D how about 10.A letter B telephone C pen D effort

Part Choose the most suitable heading from the list (A-K) for each part (1-10) of the article There is one extra heading which you not need to use Write your answers (A K) on the answer sheet (10 pts)

A Missing and meeting people

B Essential items C Be yourself

D Luggage E Leisure items F All the best

G Daily dress H Attitudes

I The right frame of mind

J Bedding K Formal dress

1

After you decide what to study and where, you need to think about what to bring to the USA Plan carefully because most international airlines will charge you high rates for overweight or oversized baggage You may check in, free of charge, two pieces of luggage on most international flights, and you may bring two carry-on bags on board 2

(127)

Students wear jeans, tennis shoes, sandals, T-shirts and sometimes shorts to class 3

There probably will be some occasions for more formal wear Men will want to have a sports jacket, trousers (or a suit), shirt and tie Women will want to have at least one smart outfit

4

Should you bring any sports equipment? Certainly you will want your swimming trunks or costume You may also want to bring your tennis racquet Leave heavy golf clubs, bicycle, soccer balls, etc at home You can easily rent or borrow this equipment

5

If you are going to live in a hall of residence, you may need to provide your own sheets, pillows and towels Some schools furnish bed linen; in other schools you can purchase a service that will provide linen and also launder it Some schools provide nothing but your bed If you don't know what you will find, bring soap and a towel for the first night, then buy anything else you need after you have settled in

What else to bring? 6

Some things you need to bring to the USA won't be packed in a suitcase - they will be packed in your mind Your expectations and attitudes can make a difference to how you enjoy your summer in the USA

7

North American people value individual experience So be prepared to be independent! 8

You will miss home, family and friends at first This will be temporary until you meet new friends and become familiar with your new environment

9

(128)

strangers for friendship and to ask for directions or assistance if you need them 10

Have a good trip, and enjoy your educational adventure

Part Point out one of the four underlined parts in each of the sentences below which is not correct

Write your answer (A, B, C or D) on the answer sheet (5 pts) When I entered the laboratory, everyone weredoing experiments A B C D

2 After questioning for a while, the man finally admitted having stolen the money A B C D

3 The result of that test must beinform before August A B C D

4 He is not a professional musician, but he can playguitar very well A B C D

5 Emily Dickson, whose poems were published after her dead, wrote about love, nature, and eternity

A B C D

6 Poverty in theUnited States is noticeably different from that in other country A B C D

7 Despite of the increase in air fares, most people still enjoy flying A B C D

8 We admire Lucy forher intelligence, cheerful disposition and she is honest A B C D

9 For his retirement, Mr Black was presentedfor an extremely expensive gift A B C D

10.However small, the sitting room is well designed and nicely decorated A B C D

(129)

Write your answers on the answer sheet (10 pts)

Wild animals (and wild plants) and the wild places -(1) - they live are seriously threatened almost everywhere on our earth One species has become -(2) - in each year of this century, but many hundreds are now being -(3) - Lack of attention would -(4) - to the rapid advance of the process of extinction

Already many kinds of wild animals have been -(5) - reduced in number that their role in the ecosystem is forgotten Animals like the great apes, the whales, seals etc., -(6) - thought to be in danger of extinction

But even more important, perhaps, -(7) - individual kinds of animals and plants, whole habitats are in danger of vanishing: marshes are being drained, and the world forests, especially the tropical forests, are being -(8) - down to satisfy man's need of timber and paper

What would our world be like (9) all the wild animals and wild plants vanished? -(10) - our life still exist then?

Part Supply the correct form of the words given in brackets to complete each of the sentences below

Write your answers on the answer sheet (10 pts)

1 The manager spoke (PERSUADE) about his plan for the restructure of the department

2 John admitted that his cruel joke to his friend was (INTEND) When he retired, he took up (CARPENTER) as a hobby

4 Your new trousers are too long They need (SHORT) The rule is not (APPLY) in this case

6 Dian Fossey was an American (ZOO)

7 I’d lost my key so I couldn’t (LOCK) the door when I got home

8 The meeting began in (AGREE), but finally everyone accepted the manager’s proposal

9 We live in a(n) (REAL) world, so don’t be too disappointed if things go wrong! 10.I’m applying for (MEMBER) of the tennis club

(130)

a It’s high time you (1.have) the house (2.repaint)

b He (3.usually see) (4 walk) with his niece in the park on Sundays

c He (5.work) in this office for 30 years by the time he (6 retire) next October d At that moment I (7.not know) what (8.say) I was too shocked, you know e Not until we (9.arrive) at his house (10.we know) that he was seriously ill

f By the time the police (11.get) to their house, the burglar (12.disappear) into the dark

g (13.Work) in the garden all day that day, my mother (14.be) totally exhausted h I can’t remember (15.meet) this man His face (16 not look) familiar to me at all i I’d rather you (17.tell) me what you (18 do) over the last weeks

j I (19.hear) the explosion while I (20.jog) in the street

Part 10 Fill in each blank in the sentences below with a suitable preposition or adverb particle

Write your answers on the answer sheet (10 pts)

a (1) weekends, the library is crowded (2) students b I am not ashamed (3) what I have done; neither am I proud

(4) it

c Mr Jones is (5) charge (6) the planning department d After being informed (7) the accident, the police quickly arrived

(8) _ the scene

e I’m (9) the impression that Thomas is guilty (10) _ the crime

f (11) some reason, the boss is opposed (12) my suggestion

g They usually stay in (13) Christmas, but go out (14) _ New Year’s Eve

h (15) my opinion, the old vase is worth $20,000 (16) _ least

i It’s obvious (17) _ everyone that Peter is eager (18) _ a chance to play in the final match

j (19) my relief, my mother has got (20) her illness

Part 11 Finish each of the sentences below in such a way that it means the same as the original sentence

(131)

1 The children should be working by now → It’s time Galileo is considered to be the father of modern astronomy

→ Galileo is regarded

3 They had to go by air because there was no train → If _ People say she has been to the island of Nagua many times

→ She

5 She asked where I had bought the shirt → She asked, " _ I'm sure Jane did not break your glasses on purpose → Jane must Why didn't they tell us about the delay earlier? → We should He regrets treating her so badly yesterday → He wishes _ Tom failed the exam, which surprised all of us → All of us

10.The exam is so easy! → What

Part 12 Rewrite each of the sentences below in such a way that its meaning stays the same, using the word given in capitals Do not change this word in any way. Write your answers on the answer sheet (10 pts)

1 My father spent two hours planting the trees in the garden yesterday TOOK The motor in this machine needs cleaning once a week HAS

3 She always has a good relationship with the children GETS "I'm sorry, but I can't lend you my laptop today, David," Mary said

REFUSED

5 Nobody can take better care of me than my mother BEST He did the math problem in only two minutes SOLUTION The price of oil has increased considerably in the last two months RISE

8 The new teaching method strongly influences children’s way of thinking INFLUENCE

9 The accident was not his fault BLAME

10.The question was so difficult that he couldn’t answer it TOO

THE END

ĐÁP ÁN ĐỀ SỐ 13

SỞ GIÁO DỤC VÀ ĐÀO TẠO KÌ THI TUYỂN SINH LỚP 10 THPT CHUYÊN

(132)

ĐỀ CHÍNH THỨC MÔN THI: TIẾNG ANH ĐÁP ÁN VÀ HƯỚNG DẪN CHẤM

Part Choose the word whose bold part is pronounced diferently from that of the others in each group (5 pts – 0.5/each)

1 C A B D B C A D D 10 D

Part Choose the word whose stress position is diferent from that of the others in each group (5 pts -0.5/each)

1 D C A C C D A A B 10 B

Part Choose the best word or phrase to complete each of the sentences below.(10 pts - 0.5/each)

1 A A B D B B B C B 10 D

11 B 12 B 13 D 14 A 15 D 16 B 17 D 18 A 19 C 20 A Part Choose the word or phrase which best fits each of the numbered blanks in the passage below.(5 pts - 0.5/each)

1 D A B C B A A B A 10 C

Part Choose the most suitable heading from the list (A-K) for each part (1-10) of the article There is oneextra heading which you not need to use (10 pts – 1.0/each)

1 D G K E J H/I C A I/H 10 F Part Point out one of the four underlined parts in each of the sentences below which is not correct (5 pts - 0.5/each)

(133)

Part Fill in each of the numbered blanks in the passage below with ONE suitable word (10 pts - 1.0/each)

1 where extinct

3

endangered/threatened lead so

6 are than cut/chopped if/when 10 Would

Part Supply the correct form of the words given in brackets to complete each of the sentences below (10 pts - 1.0/each)

1 persuasively intentional carpentry shortening applicable zoologist unlock disagreement unreal 10 membership

Part Give the correct tense or form of the verbs in brackets.(10 pts - 0.5/each)

1 had retires 11 got 16 does not

2 repainted did not know 12 had disappeared 17 told

3 is usually seen to say 13 Having worked/ 18 have done walking arrived Having been working 19 heard will have worked/ 10 did we know 14 was 20 was jogging

will have been working 15 meeting

Part 10 Fill in each blank in the sentences below with a suitable preposition or adverb particle (10 pts - 0.5/each)

1 On/At of 11 For 16 at

2 with of/about 12 to 17 to

3 of at 13 at 18 for

(134)

5 in 10 of 15 In 20 over

Part 11 Finish each of the sentences below in such a way that it means the same as the original sentence.(10 pts - 1.0/each)

1. It’s time the children were working

2. Galileo is regardedas the father of modern astronomy

3. If there had been a train, they would not have had to go / would not have gone by air.

4. She is said to have been to the island of Nagua many times. 5. She asked, "Where did you buy the shirt?”

6. Jane must have broken your glasses by accident/ by chance 7. We should have been told about the delay earlier

8. He wishes he had not treated her so badly yesterday 9. All of uswere surprised by/at Tom’s failure in/on the exam 10.Whatan easy exam (it is)!

Part 12 Rewrite each of the sentences below in such a way that its meaning stays the same, using the word given in capitals Do not change this word in any way. (10 pts - 1.0/each)

1 It took my father two hours to plant the trees in the garden yesterday The motor in this machine has to be cleaned once a week

3 She always gets on (well) with the children

4 Mary refused to lend David her laptop/ to lend her laptop to David that day

5 My mother is the best person/woman/one to take care of me / who can take care of me

6 He found a/the solution to the math problem in only two minutes

(135)

9 He was not to blame for the accident/ We cannot put the blame on him for the accident / for the accident on him

10.The question was too difficult for him to answer

THE END ĐỀ SỐ 14

SỞ GIÁO DỤC VÀ ĐÀO TẠO NAM ĐỊNH

BÀI THI TUYỂN SINH LỚP 10 TRƯỜNG THPT CHUYÊN Năm học 2013-2014

Môn: TIẾNG ANH(chuyên)

Thời gian làm bài: 120 phút

Đề thi gồm 05 trang

Điểm thi

Bằng

số: Bằng

chữ:

Họ, tên chữ ký giám khảo

Giám khảo

1: Giám khảo

2:

SỐ PHÁCH (Do hội đồng chấm thi ghi)

(136)

I Choose the word whose underlined part is pronounced differently from that of the others Put a tick (√) next to the correct answer A, B, C or D There is an example at the beginning (0) (0.5 p)

1 A farmers B quarters √C.visits D prisons

2 A finished B excited C fascinated D landed

3 A comfort B hobby C knowledge D respond

4 A design B conserve C basic D physics

5 A luggage B village C manage D teenage

6 A noses B speeches C leaves D buses

II Choose the word whose stress pattern is different from that of the others Put a tick (√) next to the correct answer A, B, C or D There is an example at the beginning (0) (0.5 p)

1 A teacher B village √C police D worker A discuss B attract C suppose D finish

3 A daughter B decide C provide D enjoy

4 A excellent B different C quality D dependent A generous B industrial C internet D general A diversity B biology C communicate D education

PART B GRAMMAR AND VOCABULARY (3.0 points)

I Choose the best word or phrase to complete the following sentences Put a tick (√) next to the correct answer A, B, C or D There is an example at the beginning (0). (1.0p)

1 Peter tried to avoid my questions

A answer B to answer√ C.answering D answered My friend gave me a surprise party, was good of them

A which B that C whose D who

2 You should get a license right away, you’ll have to pay a fine

A yet B otherwise C.still D consequently

3 We couldn’t cut the string because the of the knife was not safe enough

A. side B edge C border D verge

(137)

A make B pay C get D do I supposed that he would come on time, ?

A didn’t I B.wouldn’t he C did I D would he

6 What we have to now is to calm and try to think reasonably

A maintain B.sustain C suffer D remain

7 She wrote report on the Vietnam war

A ten-page B a ten-page C ten-pages D a

ten-pages

8 After we for three hours, we stopped to let the others catch up with us

A walk B have walked C had walked D

walked

9 you to be offered that job, would you have to move to another city?

A Provided B Should C Were D Had

10 I’m really sleepy today I wish I Bob to the airport late last night

A weren’t taking B hadn’t had to take C didn’t have to take D didn’t take

II Each of the following sentences has ONE mistake Underline the mistake and write the correction in the provided blank on the right There is an example at the beginning (0) (0.5 p)

Questions Correction

1 He collided with a car because he was driving too fastly fast You mustn’t have seen my sister because I have no sister

living on the other end of this city

3 What he told you seem to be of no importance

4 There are many researches show that various species of animals have been extinct

(138)

6 The disappearance of one or several species may result from the loss of biodiversity

III Give the correct form of the words in capital letters Write your answers in the blanks There is an example at the beginning (0) (0.5p)

1 If you get a burn, cool it (IMMEDIATE) _ immediately _

2 In many countries it is (LEGAL) to smoke cigarettes under the age of 16

3 Good hygiene helps to (MINIMUM) the risk of infection

4 She at first wanted to be a (LAW) but now she works as a doctor

5 With (IMPROVE) made in transportation, people have more options when moving around from place to place

6 Most (ENVIRONMENT) at the international workshop in Tokyo focused on the increase of the earth temperature

IV Choose one given phrasal verb to complete each of the sentences You may have to change the verb form but cannot use one phrasal verb twice Write your answers in the spaces provided There is an example at the beginning (0) (1.0p)

bring up cut down onget on with with put of go into take after give up take of settle down put up with

1 The flight took off two hours ago

2 I’m trying to lose weight by bread and sweet things How you your boss? Is he as unfriendly as people say? You have almost solved the crossword puzzle You mustn’t

(139)

5 If you have anything important to do, it straightaway Don’t _it

6 When Kathy finishes her studies, she’ll probably teaching I think she would be a very good teacher

7 Jane plays the piano beautifully She her father who used to be a concert pianist

8 Jeff is over forty and still a bachelor I don’t think he will ever I’m hot and tired I could a cold shower

10.I can’t the hammering from the flat above much longer It’s giving me a headache

11.Jim’s parents were killed in an air crash when he was very young An aunt _him _

PART C READING (3.0 points)

I Read the text below and decide which answer (A, B, C or D) best fits each space Put a tick (√) next to the correct answer There is an example at the beginning (0).(1.0 p)

It is well-known that (0) expectancy is longer in Japan than in most other countries A (1) report also shows that Japan has the longest health expectancy in the world A healthy long life is the result of the (2) in social environment Scientists are trying to work (3) exactly what keeps elderly Japanese people so healthy, and whether there is a lesson to be learnt from their lifestyles for the rest of us Should we (4) any changes to our eating habits, for instance, or go jogging each day before breakfast? Is there some secret ingredient in the Japanese diet that is particularly (5) to the human body?

(140)

(9) the retirement age from 65 to 70 could be one solution to the problem Work can give the elderly a (10) _ of responsibility and mission in life It’s important that the elderly play active roles in the society and live in harmony with all generations

1 A living √ B life C alive D live

2 A late B recent C contemporary D new

3 A increase B progress C growth D improvement

4 A for B out C in D off

5 A B make C set D give

6 A helpful B supportive C advantageous D beneficial A contributing B helping C resulting D causing A facing B meeting C adopting D encountering

9 A for B after C over D out

10.A Putting B Moving C Rising D Raising

11.A sense B sentiment C feeling D characteristic II Read the passage and put a tick (√) next to the correct answer (A, B, C or D) for each of the questions below There is an example at the beginning (0) (2.0 p)

Throughout history, pigs have been useful to humans Their bodies work much like ours Doctors have used chemicals from pigs’ bodies to help treat diseases in people We also use pigs for food We even make objects from their skin, such as bogs and balls Domestic pigs live on farms These kinds are useful for people Wild pigs live in the mountains and forests They are causing big problems in the United States

In the early 1990s, wild pigs – called boars – were brought from Europe to North Carolina Boars have two sharp horns, or tusks, on each side of their mouth They are aggressive and mean At first, people kept the boars together in a forest Hunters enjoyed trying to shoot them But by the 1920s, some boars had escaped Boars have lived in the mountains of North Carolina ever since Problems began when the boars entered a national park They dug holes in the soil and destroyed plants that other animals need for food

(141)

Some hunters brought them They built a fence around the boars However, several pigs escaped (***) Later, some of the wild pigs mated with escaped domestic pigs Today, the number of wild pigs keep growing (****) They are ruining lots of land A scientist found 2,000 rare flowers in the stomach of one dead boar

How can people stop these hungry pigs? There are no easy answers One national park in Hawaii built a 15-meter fence around its entire 28,000 acres Other states encourage hunters to kill the animals However, pigs are intelligent, stubborn, and tough We domesticated them for use on the farm Now, our challenge is to control them when they leave the barnyard

1 The word “ours” in paragraph refers to

A. pigs √B people C bodies D doctors All of the following are mentioned in the passage EXCEPT

A how boars come to the US B how boars got to Europe C the types of damage boars cause D the characteristics of boars How long have wild pigs been a problem in the US, according to the passage?

A 20-25 years B 80-85 years C 90-95 years D. more than 100 years

4 What is the topic of paragraph 3?

A Differences between domestic pigs and wild pigs B How Californian pigs are different from other pigs C The problems pigs are making in one state

D Why people in California like wild pigs

5 What does the word “them” in the last sentence refer to?

A.hunters B.national parks C.pigs D.states

6 Which statement best reflects the author’s opinion?

A Wild pigs are a necessary part of nature B All wild boars should be killed C Domestic pigs are dangerous D Controlling wild pigs is a difficult challenge

7 From paragraph 3, we can infer that

(142)

8 The word “escaped” could best be replaced by which of the following? A got away B become lost C rebelled D died

9 The word “rare” in paragraph is closest in meaning to which of the following? A beautiful B exquisite C common D special

10.The word “entire” in the last paragraph is closest in meaning to

A very large B whole C make sleepy D hot

11.Where would the sentence “Some of these boars are descendants of domestic pigs the Spanish settlers brought with them in the 1760s.” be best added to the passage?

A (*) B (**) C (***) D (****)

PART D WRITING (3.0 points)

I Finish each second sentence so that it has similar meaning to the first sentence, using the clues at the beginning of each sentence There’s an example at the beginning (0) (1.0p)

1 It takes him at least three hours a day to chat on facebook Everyday he spends at least three hours chatting on facebook ‘I think you ought to see a doctor,’ he said

He suggested……… ……… Could you tell me the time of the last train to London?

Could you tell me when……… ……… It’s thought that the Prime Minister is considering raising taxes

The Prime Minister……… ……… Next Sunday is our tenth wedding anniversary

We will have been……… ……… I not intend to discuss this matter any further

(143)

It was not ……… It began to pour with rain as soon aswe had started our walk

No sooner.……… It’s pointless to appeal against your sentence

There’s no……… …………

10.It was the weakness of the foundations that led to the collapse of the building If the……… ………

11.Basically, a couple’s happiness depends on their ability to communicate Basically, the more……… ….……

II Rewrite each of the following sentences so that it has a similar meaning to the given sentence, using the word given in capital letters at the end Do not change the word given There is an example at the beginning (0) (1.0 p)

1 They sent him to prison for three years SENTENCED He was sentenced to three years in prison

2 The report says that the pyramids will deteriorate ACCORDING

……… ………… He didn’t lock the front door when he left the house WITHOUT

……… ………… Pandas need a special diet, without which they die UNLESS

……… ………… Many species of wild life are threatened with extinction VERGE

……….……… We were not surprised to hear that he had been stealing money from the

company CAME

(144)

III Writing an argument (1.0 p)

It is said that “There is no place like home.”What’s your opinion?

Give reasons for your answer and include any relevant examples from your own knowledge or experience

You should write about150 words

(145)

……… ……… ……… ……… ……… ………

-THE

END -ĐÁP ÁN ĐỀ SỐ 14

SỞ GIÁO DỤC VÀ ĐÀO TẠO NAM ĐỊNH

ĐÁP ÁN VÀ HƯỚNG DẪN CHẤM THI

ĐỀ THI TUYỂN SINH LỚP 10 TRƯỜNG THPT CHUYÊN Năm học 2013-2014

Môn: TIẾNG ANH (chuyên)

PART A PHONETICS (1.0 point) I Pronunciation (0.5 p) 0.1 point for each correct answer

1A 2A 3B 4D 5C

II Stress (0.5 p) 0.1point for each correct answer

1D 2A 3D 4B 5D

PART B GRAMMAR AND VOCABULARY (3.0 points) I MCQ (1.0 p) 0.1 point for each correct answer

1A 2B 3B 4C 5B 6D 7B 8C 9C 10B

(146)

1 mustn’t can’t seem  seems/seemed show  showing their  its from  in

III Word form (0.5 p) 0.1 point for each correct answer

1 illegal minimize/minimise lawyer improvements environmentalists

IV Phrasal verbs (1.0 p) 0.1point for each correct answer

1.cutting down on 2.get on with 3.give up 4.put…off 5.go into

6.takes after 7.settle down 8.do with 9.put up with 10.brought…up

PART C READING (3.0 points) I Gap fill (1.0 p) 0.1point for each correct answer

1B 2D 3B 4B 5D 6A 7A 8B 9D 10A

II MCQ reading (2.0 p) 0.2point for each correct answer

1B 2C 3C 4C 5D 6D 7A 8D 9B 10B

PART D WRITING (3.0 points)

I Sentence completion (1.0 p) 0.1point for each correct answer He suggested that I should see a doctor

2 Could you tell me when the last train to London leaves/ departs? The Prime Minister is thought to be considering raising taxes We will have been married for ten years (by) next Saturday I have no intention of discussing this matter any further

6 It wasn’t until he was nearly 30 that he passed his driving test

7 No sooner had we started our walk than it began to rain/to pour with rain There’s no point (in) appealing against your sentence

9 If the foundations hadn’t been weak, the building wouldn’t have collapsed

10.Basically, the more a couple (can) communicate/ communicates, the happier they are/ will be

(147)

1 According to the report, the pyramids will deteriorate He left the house without locking the front door Unless pandas have a special diet, they die/ will die Many species of wild life are on the verge of extinction

5 It came as no surprise (to us) to hear that he had been stealing money from the company

III Writing an argument (1.0 p) Marking scheme

The impression mark is based on the following scheme:

1 Format (0.1 point) - the argument has parts: the Introduction, the body (01 or more para) and the conclusion

2 Content (0.5 point): a provision of main ideas and details as appropriate to support the argument

3 Language (0.2 point): a variety of vocabulary and structures appropriate to the level of secondary gifted students

4 Presentation (0.2 point): coherence, cohesion, and style appropriate to the level of secondary gifted students

-THE END -Quy tắc làm tròn điểm:

5.1 5.0 5.2  5.25 5.3  5.25 5.4  5.5 5.5  5.5

5.6  5.5 5.7  5.75 5.8  5.75 5.9  6.0 6.0  6.0

ĐỀ SỐ 15

SỞ GIÁO DỤC VÀ ĐÀO TẠO THỪA THIÊN HUẾ

ĐỀ CHÍNH THỨC

KÌ THI TUYỂN SINH LỚP 10 CHUYÊN QUỐC HỌC Môn : TIẾNG ANH - Năm học: 2008 - 2009

(148)

( Đề thi gồm 05 trang)

-

-I GRAMMAR AND VOCABULARY ( 30 points) Part 1:Choose the best answer among A,B,C or D (10 points) It is the most beautiful building the world A around B of C in D at

2 If you come to the cinema, your friend with you A bring B get C invite D take

3 I’m sorry I missed the meeting yesterday I didn’t well A feel B fell C felt D feeling

4 We discussed the problem but we a final decision

A don’t reach B won’t reachC hadn’t reached D didn’t reach Tom, write carefully and don’t any mistakes!

A B have C give D make Sue can’t come to the party she’s broken her leg

A although B because C because of D so rush hour, the roads in the city are very busy

A While B During C Through D On Kate a great time in France and doesn’t want to come back A is having B has C had D had had

9 If you forty cigarettes a day, you would cough a lot

A smoke B smoked C would smoke D smoking 10 When I went back to my home town after many years, I was surprised that

it changed so much

A has B had C was D is

Part 2: Complete each sentence with a suitable preposition: (10 points) We’ll have to use the stairs The lift is of order Surely Peter can’t all that work his own

3 Turn right the traffic lights and you’ll see the cinema

4 This letter is full mistakes Could you write it again, please? You' re very different your brother

(149)

8 Helen is excited winning the prize

9 I am very busy my lessons and tasks today 10 Who's the woman dressed green?

Part 3: Use the word given in capitals at the end of each line to form a word that fits in the space in the same line.The first one has been done for you :(0)(10 points)

WHERE TO EAT

It’s hard to find a place to eat which satisfies the whole family (0) Perfection is not an easy thing to find when it comes to restaurants I remember my parents taking me as a child to the most (1) restaurant they could find They didn’t think it was worth spending a lot of money on someone who had no table manners!

Today the choice of restaurants is so great, however, that total

(2) is rare It’s difficult to explain exactly what I’m looking for in a restaurant It’s certainly not (3) surroundings, although I must admit that I’m usually (4) to go into a place which looks dirty or dingy In these days of fast food, it takes a lot of (5) to persuade my own children that a bowl of salad is every bit as tasty as a burger and chips! Some parents I know (6) taking their children to fast food restaurants, but I’ve always made an effort to go and see just what’s available in those places I can’t honestly say that some of the experiences were so good that they were (7) , but at least I feel I have given the fast food chains a try I would like to be able to say that we go out to dinner once a week (8) , this is not the case We either seem to be too busy, or decide that it’s more (9) to have a family meal at home, which is probably for the best! But one thing is certain, I am still as (10) about food as I ever was, and intend to remain so!

PERFECT EXPENSE

DISAPPOINT LUXURY WILLING DETERMINE LIKE

FORGET FORTUNATE SATISFY

ENTHUSIASM

(150)

Part 1: Read the passage and choose the correct answer among A, B, C or D: (15 points) HEDGEHOGS THREATEN WILDLIFE

There are so many threats to the wildlife on our planet nowadays that we are rarely surprised when we are (1) about another one (2) ., some people might find it difficult to (3) that small creatures (4) hedgehogs would be the cause, rather than the victims, of one of these threats One resident of an island which (5) .off the west coast of Scotland (6) .how this extraordinary situation has come about It appears that hedgehogs are not native to the islands They were introduced by a gardener who thought they (7) .be an effective way of controlling the slug population There are now so (8) hedgehogs on the island that they are putting many rare birds at (9) because they eat birds’ eggs The hedgehogs, it seems, are (10) , so what are the local people going to (11) it ? One group of conservationists tried to transport hedgehogs to the mainland The plan might have (12)…………if residents on the mainland hadn’t pointed out that the hedgehogs would (13)…………just as much of a threat on the mainland as on the island But, unless action is (14) …………soon, the continuing increase in the hedgehog population will (15) …………in the disappearance of certain rare species of birds

1 A mentioned B explained C told D said A Even B Although C Yet D However A believe B convince C suppose D consider

4 A so B such C like D as

5 A extends B lies C covers D stretches A described B commented C talked D meant

7 A should B can C ought D would

8 A largely B much C many D.far

9 A danger B risk C trouble D difficulties 10 A anywhere B everywhere C anything D nothing

11 A with B about C to D for

(151)

14 A made B done C taken D held

15 A result B lead C cause D turn

( Note: hedgehog: nhím)

Part 2: Read the text below and think of the word which best fits each space Use only ONE word in each space (15 points)

LOST AND FOUND

My father considered (0) himself to be a lucky man He wasn’t rich or famous, but he was lucky (1) he lost anything, he was usually (2) to find it again (3) .such example was a fountain pen, a present from my mother on (4) .… wedding anniversary We had had a family day out in the mountains My brother, (5) was about ten years old at the time, and I enjoyed going to a lake where you could hire a boat and row out across the water (6) had once been a village in the valley, but the valley had been flooded in order (7) create a reservoir for a city many miles away When the water was clear, you could still make out the top of the church spire and the walls of houses (8) we were returning home that night, my father discovered that he had lost his treasured fountain pen About two weeks later, he announced that in spite (9) the gloomy weather, we were going back to the lake for the day We were surprised, but said nothing We set (10) about nine in the morning, and as (11) as we arrived at the lake, we headed (12) the spot where we had had our picnic on the previous occasion As we were deciding whether to hire a boat for the afternoon or (13) , my father gave a shout There, lying on the ground, was a pen There was (14) doubt at all in my mind that (15) was the pen he had lost two weeks beforehand

Part 3: Read the following passage and answer the questions (5 poínts)

(152)

important than what happens during the meeting The skills used then are more professional and less procedural So no matter how well you run a meeting, it is the work that gets done after the meeting that is important

1 What is one way to run a meeting well?

A Watch how your manager run meetings B Minimize the number of participants C Let the group make decisions D Let everyone speak

2 What is the purpose of a meeting agenda?

A To keep the speakers organized B To allow free discussion

C To send to others in advance D To keep focused on improtant items How should you receive other people’s comments at a meeting?

A Try to keep others from talking B Thank them and move on

C Give them as much time as they want D Respond in detai to all comments How should people state their ideas?

A Succinctly B Quietly C Wordily D Encouragingly The author states that

A meetings should be held more frequently B all meetings should be in the morning C no one should receive credit for their work D the real work is accomplished after the meeting III WRITING: ( 35 points)

Part 1: Complete the second sentence so that it has the similar meaning to the first one, by using the word given.( Do not change the word given.) ( 10 points)

1 There was too little time to go back home for my umbrella not There ……… ………

2 It was necessary to book concert tickets well in advance be Concert tickets ……… ………

3 What interests you about this city? find

(153)

4 I regret not studying harder at school only If ……… ………

5 Could you come and collect me from the station? up Would you mind ……… ……… ?

6 Are you interested in sports ? tell

Can you ……… ………?

7 They had extinguished the fire by the time the fire brigade arrived put The fire ……… ………

8 I’m sure you were proud of your son’s success must You ……… ………

9 I started studying English three years ago for I ……… ………

10 The football players’ training is still going on finished The football players ……… ………

Part 2:Read the text below and look carefully at each line Some of the lines are correct, and some have a word which should not be there If a line is correct, put a tick () by the number If a line has a word which should not be there, write the word by the number

( 15 points) ADVENTURE IN PERU

0 I’ve just been to see a film is called Touching the Void It’s

00 about two climbers who decided to go climbing in the mountains 1 in Peru It took them two days to reach the mountain they

2 had been decided to climb because it was in such a remote place 3 They left a companion and the most of their supplies at a base camp, 4 optimistically thinking about they would be back in a couple of

5 days After a difficult climb, they reached at the top of

(154)

6 the mountain It was then that everything started to go wrong 7 Because of one climber had broken his leg, he had to be

8 lowered down the mountain by means of a rope His leg it was so 9 much painful that he couldn’t stand on it After a very exciting bit 10 of the film, which I won’t describe, he found by himself alone

11 at the bottom of a crevasse He managed to climb out and eventually 12 crawl to safety The cold was so intense that he got frostbite in his hands,

13 but he was carried on, dragging himself down a glacier and across rocks

14 When he arrived at the base camp, his friends were too amazed to see him

15 They couldn’t believe that in despite his broken leg, he had managed to reach the camp

6 ……… 7 ……… 8 ……… 9 ……… 10 …… 11 …… 12 …… 13 … … 14 …… 15 ……

Part 3: Write an essay ( at least 200 words) on the following topic: (10 points)

There have been many technological developments in the 20th century, for example in transport, telecommunications and health

What technological development you think has been the most important ? How has it changed people's lives ? Have all the changes been positive ?

Give reasons for your answer

(155)

(You can write on the back of this page) ĐÁP ÁN ĐỀ SỐ 15

SỞ GIÁO DỤC-ĐÀO TẠO KÌ THI TUYỂN SINH LỚP 10 CHUYÊN QUỐC HỌC THỪA THIÊN HUẾ Môn: TIẾNG ANH - Năm học: 2008 - 2009 - Thời gian làm : 150 phút ĐỀ CHÍNH THỨC

ANSWER KEY

I VOCABULARY & GRAMMAR: (30 points)

Part 1: (10 points)(one point for each correct answer)

Q# 10

Key C A A D D B B A B A

Part 2: (10 points) (one point for each correct answer)

Q# 10

Key out on at of from / to

for to abou t/at

(156)

Part 3: (10 points)(one point for each correct answer)

1 inexpensive disappointment luxurious unwilling determination dislike unforgettable Unfortunately satisfying 10 enthusiastic II READING: (35 points)

Part 1: (15 points) (one point for each correct answer)

Q # 10 11 12 13 14 15

Key C D A C B A D C B B B D B C A

Part 2: (15 points) (one point for each correct answer)

1 Whenever/When/ If2 able One their who

6 There to While/As/When of 10 off / out 11 soon 12 for/to/towards 13 not 14 no 15 it / this / that

Part 3: (5 points) (one point for each correct answer)

1 B D B A D

III WRITING: (35 points)

Part 1: (10 points) (one point for each correct answer)

1 There was not enough time (for me) to go back home for my umbrella Concert tickets had/needed to be booked well in advance

3 What you find interesting about this city? If only I had studied harder at school

5 Would you mind picking me up from the station? Can you tell me if you are interested in sports ?

(157)

8 You must have been proud of your son’s success

9 I have been studying / have studied English for three years 10 The football players haven’t finished their training yet

Part 2: (15 points) (one point for each correct answer)

1  been the about at  of it much 10 by 11 12 13 was 14 too

15 in

Part 3: (10 points)

ĐỀ SỐ 16

SỞ GD& ĐT NGHỆ AN KỲ THI TUYỂN SINH VÀO LỚP 10 TRƯỜNG THPT CHUYÊN PHAN BỘI CHÂU

NĂM HỌC 2011- 2012

Môn thi: TIẾNG ANH

Thời gian: 120 phút (không kể thời gian giao đề)

Điểm thi Họ tên, chữ kí giám khảo Số phách Điểm số………

………

Giám khảo 1: ……… ………

(158)

Điểm chữ……… ………

Giám khảo 2: ……… ……… ………

SECTION I: PHONETICS

I Choose the word whose underlined part is pronounced differently from that of the others.

1 A precious B preparation C pretty D repetition A architect B parachute C choir D psychology A treason B reason C season D jealous A aggressive B suggest C energy D garbage A measure B vision C pleasure D pension Your answers:

1

II Identify the word whose stressed pattern is different from that of the others. A official B literacy C inventory D stationery A diversity B severe C ecosystem D courageous A prediction B compulsory C intelligent D judgement A valuable B entertain C honesty D mountain A realize B product C mechanic D separate Your answers:

1

SECTION II: VOCABULARY AND GRAMMAR I Choose the best answer from A, B, C or D to fill in the blanks. It is recommended that he _ this course

(159)

A that is successful B what success isC that success is D what is success I could not turn in my paper on time because my word processor _

A fell down B turned down C tore down D broke down “You must always _ us the truth” the judge said to the man

A tell B confess C speak D say

5 Her father is a _ drinker He is always drinking

A strong B heavy C addictive D capable

6 Don’t set off fireworks too closely to your house, _?

A you B don’t you C will you D won’t you Trains stop here in order to _ passengers only

A pick up B get off C pull up D pull in

8 In recent years, more and more people _ for things with credit cards

A pay B are paid C are paying D have been paying He was the first person _ the fire

A discover B to discover C discovering D discovered 10 “Who was the man with you yesterday?” - “He was _.”

A the next door man B the man next to the

door

C the man next by the door D the man next door 11 John: “I didn’t pass my driving test.” -Anna: “ _!”

A Better luck next time B So poor C Congratulations D That was nice of them

12 You can _ your sister to our party tonight

A bring B carry C take D get

13 You need more exercise, so you should _ golf

(160)

14 I like that photo very much Could you make an _ for me?

A increase B enlargement C expansion D extension 15 His roses won the first in the local flower competition

A prize B award C cup D reward

Your answers:

1

6 10

11 12 13 14 15

II There are TEN mistakes in the passage Each numbered line in the right column has one Find and correct them.

Everyone wants to reduce pollution But the pollution problem is as complicated as it is serious It is complicated because of much pollution is caused by things that benefit people For example, exhaust of automobiles causes a large percent of all air pollution However, the automobile provides transportation for million of people Factories discharge much of material that pollutes air and water, but factories give employment to the large number of people Thus, to end or great reduce pollution immediately, people would have to stop using many things that benefit them Most the people not want to that, of course Nevertheless, pollution can be gradual reduced in several ways Scientists and engineers can work to find ways to less the way of pollution that such things like automobiles and factories cause Governments can pass and

Your answer:

(161)

enforce laws that request businesses and traffic to stop, or to cut down on certain polluting activities

(10) ……… ……

III Fill in each blank with the correct form of the given word.

1 He feels sad about his _ result in the examination SATISFY The _ of this factory are widely consumed in this country PRODUCE You must keep on working with him no matter how much you

AGREE Film festivals are _ divided into categories like drama,

documentary or animation

TYPICAL He enjoyed _ with friends and it cost nothing SOCIAL I have just bought a replacement for one of my favorites which

_ disappeared

MYSTERY Gift exchange is _ expression of social relations SIGNIFY I try not to remember this experience that only leaves me with

unhappy thoughts

FRIGHTEN The boy _ asked for permission to go out with his friends REPEAT 10 British Rail apologized for the _ of the 4.20 to Bath CANCEL Your answers:

1

6 10

SECTION III: READING COMPREHENSION

I Choose the best answer from A, B, C or D to fill in each numbered blank in the following passage.

NEW HUNT FOR LIFE IN SPACE

(162)

Two days (3) _, NASA scientists had shown a proof that one of Jupiter’s moons could support life, Penny announced that his telescope may be included in a European Space Agency mission The Darwin project, with a (4) _ of 500 million, is on a short list of two proposals If approved it will probably be (5) _ around 2015, its destination somewhere between Mars and Jupiter The blueprint is actually for five telescopes positioned 50 meters apart in space, slowly circling a (6) _ processing station The combined data from these telescopes would build up a full picture of a planet, picking out faint images that have never been seen before Darwin would not be able to take detailed photographs of the planets it (7) _, but Penny believes a second-generation telescope could be sent up to this He claims it is worthwhile mapping the universe around our (8) _ galaxy, even though these planets lie (9) _ our reach for the moment The European Space Agency will make its decision (10) Darwin within three years

1 A another B other C others D the other A light B lighten C lightening D lighting

3 A following B after C next D later

4 A price B schedule C charge D budget

5 A driven B fetched C launched D taken A central B centre C middle D heart A invents B searches C discovers D looks

8 A alone B same C one D own

9 A out B toward C beyond D over

10 A of B on C about D with

Your answers:

1

6 10

II Read the following passage and choose the best answer from A, B, C or D to the questions.

(163)

show a pilot how fast nearby planes are moving The basic principle of radar is exemplified by what happens when one shouts in a cave The echo of the sounds against the walls helps a person determine the size of the cave With radar, however, the waves are radio waves instead of sound waves Radio waves travel at the speed of light, about 300,000 kilometers in one second A radar set sends out a short burst of the radio waves Then it receives the echoes produced when the waves bounce off objects By determining the time it takes for the echoes to return to the radar set, a trained technician can determine the distance between the radar set and other objects The word “radar”, in fact, gets its name from the term “radio detection and ranging” “Ranging” is the term for detection of the distance between an object and the radar set Besides being of critical importance to pilots, radar is essential for air traffic control, tracking ships at sea, and for tracking weather systems and storms

1 What is the main topic of this passage?

A The nature of radar B History of radar C Alternatives to radar D Types of ranging

2 According to the passage, what can radar detect besides locations of objects?

A Shape B Speed C Size D Weight

3 Which type of waves does radar use?

A tidal B heat C sound D radio

4 The word tracking in the passage most closely means _

A sending B repairing C ranging D searching for What might be inferred from the passage about radar?

A It has improved navigational safety B It was developed from a study of sound waves C It takes the place of a radio D It gave birth to the invention of the airplane

Your answer:

1

III Fill in each numbered blank with a suitable word in the following passage.

(164)

attempt to (2) _ money for charity Mrs Naylor drove (3) _ sports car for two miles in a straight line along the runway of a disused airfield She was in radio contact with her husband Pete Naylor, (4) _ was able to give her directions and advice in this way Her amazing achievement is expected to raise about £50,000, which will be donated (5) _ a company that trains dogs for the blind Mrs Naylor has been blind (6) _ she was six years old and she still has some visual memories of the world (7) _ her She admits that not having her sight makes her life awkward at times She believes that if you are disabled in any way, you shouldn’t waste time (8) _ sorry for yourself “It makes a lot of sense to keep busy” she says “Achieving (9) _ can be a great source of self confidence” Miranda is now looking for a new challenge and (10) _ like to ride a motorbike

Your answers:

1

6 10

SECTION IV: WRITING

I Finish the second sentence so that it has the same meaning as the first one. I find his handwriting very hard to read

 I have ……… ……… ……… He got down to writing a letter as soon as he returned from his work No sooner ……….… Success depends on your hard work

 The harder ……… … “If I were you, I wouldn’t accept his marriage proposal”, said Nam to Lan Nam……….……… No matter how hard I tried, I could not open the window

(165)

II Finish the second sentence so that it has the same meaning as the first one, using the given word Do not change the given word.

1 The fridge is completely empty ……….……… …

LEFT

2 It is pointless to have that old typewriter repaired

……….……… ……

WORTH

3 Frank never pays any attention to my advice

……….……….…

NOTICE

4 John only understood very little of what the teacher said ……….……… ……

HARDLY

5 Her ability to run a company really impresses me

……….……….……

IMPRESSE D

III Write a composition on the following topic (about 150-200 words) Do you agree or disagree with the following statement?

It is better for children to grow up in the countryside than in a big city.

(166)

………

……… ……… ……… ……… ……… ……… ……… ……… ……… ……… ………

………

……… ……… ……… ……… ………

………

(167)

……… ……… ……… ……… ……… ……… ……… ……… ………

………

……… ……… ……… ……… ………

………

(168)

……… ………

ĐÁP ÁN ĐỀ SỐ 16

ĐÁP ÁN BÀI THI TIẾNG ANH VÀO 10 TRƯỜNG THPT CHUYÊN PHAN BỘI CHÂU

SECTION I: PHONETICS

I Choose the word whose underlined part is pronounced differently… (5x1=5 pts)

1 C B D A D

II Identify the word whose stressed pattern is different from … (5x1=5 pts)

1 A C D B C

SECTION II: VOCABULARY AND GRAMMAR

I Choose the best answer from A, B, C or D to fill in the gaps (15x1=15 pts)

1 C B D A B

6 C A D B 10 D

11 A 12 C 13 D 14 B 15 A

II There are TEN mistakes in the passage Find them out and correct them (10x1=10 pts)

(169)

6 greatly Most of gradually lessen 10 as

III Fill in each gap with the correct form of the given words (10x1=10 pts)

1

unsatisfactory

2 products disagree typically socializing

mysteriously

7 significant frightening repeatedly 10

cancellation

SECTION III: READING COMPREHENSION

I Choose the best answer to fill each blank in the following passage (10x1=10 pts)

1 B A D D C

6 A C D C 10 B

II Read the passage and choose the answers to the questions below (5x2=10 pts)

1 A B C D A

III Fill each blank in the following passage with a suitable word (10x1=10 pts)

1 crazier raise a who to

6 since around feeling something 10 would

(170)

I Finish the second sentence so that it has the same meaning as the first (5x1=5 pts)

1 I have difficulty (in) reading his handwriting

2 No sooner had he returned from his work than he got down to writing a letter The harder you work, the more successful you are

4 Nam advised Lan not to accept his marriage proposal Try as hard as I might, I could not open the window

II Finish the second sentence so that it has … (5x1=5 pts)

1 There is nothing leftin the fridge

2 That old typewriter is not worth repairing / It is not worth repairing that old typewriter

3 Frank never takes any notice of my advice

4 John could hardly understand what the teacher said /Hardly could John understand……

5 What I am really impressed by is her ability to run a company / I’m really impressed by her…

III Write a composition on the following topic (about 150-200 words) (15 pts)

- Layout and length

- Grammar, structures and vocabulary - Content

(171)

ĐỀ SỐ 17

SỞ GIÁO DỤC VÀ ĐÀO TẠO HÀ TĨNH

-ĐỀ CHÍNH THỨC

ĐỀ TUYỂN SINH LỚP 10 THPT CHUYÊN HÀ TĨNH

Môn: Tiếng Anh

Thời gian gian: 120 phút( không kể thời gian phát đề) ( Thí sinh làm vào tờ giấy thi phát kèm đề thi này)

Part 1: Choose the word whose underlined part is pronounced differently from that of the rest

1 A hope B stop C slope D cope

2 A technical B attention C event D talent A encounter B enter C enlarge D entrust

4 A call B count C centre D cable

5 A pure B durable C cure D pursue

Part 2: Pick out the word whose syllable is stressed differently from the others: A passage B enjoyable C moment D matches

2 A associate B together C traveler D protected A weather B Electric C absorb D potential A preference B attraction C advantage D discover A escaping B eruption C delicate D explain

Part 3: Choose the correct word or phrase underlined to complete the sentence What your sister like? She is funny and intelligent

A is B does C was D did

2 If I knew you busy, I wouldn’t disturb you

A were B had been C are D would be

3 The man towards us is an engineer

A coming B comes C is coming D to come

(172)

A would meet B would have met C had met D met

5 The company employees are on strike is closing down soon

A which B whose C that D where

6 Water boil/ is boiling/ boils/ was boling at 100oC Sunday is a holiday most people rest

A which B that C where D when

8 When she was cooking dinner, she her finger

A cuts B cut C was cutting D is cutting I have lived here since I 10

A am B have been C will be D was

10 They would be angry if you them

A aren’t B don’t C won’t D didn’t visit

11 What will you if you _a million dollars?

A win B won C had won D will win

12 If you stay up late, you _sleepy the next morning A feel B felt C would feel D will feel 13 _which attract so many visitors as Venice, Italy A Fewer B As few C There are few D Few places 14 He was _of all his money

A stolen B robbed C removed D taken

15 I _him to arrive in a fortnight

A expect B wait C hope D suspect

16 The question was difficult therefore _of students could answer it A few B a lot of C a few D a bit

17 The people in the theatre are the

(173)

18 If you live in a house which is not yours you have to pay _

A a fee B rent C fare D charge

19 The person who designs a house is the

A engineer B architect C painter D builder

20 Can you a pound note? I need some coins for the telephone box

A exchange B change C give D offer

21.What advice she gave us !

A so good B such good C a good D good 22.My dog as well as my cats twice a day

A eat B eats C has eaten D have eaten

23.When I came into the class, I realized I knew _ A them all B all them C they all D all of they 24.Do it right now, _ ?

A you B aren’t you C will you D don’t you 25.We for her for ten minutes

A wait B waited C are waiting D have been waiting 26.His father is not interested in tennis and _

A he doesn’t, either B so doesn’t he C neither does he D neither is he

27.The little girl wasted half an hour _for her picture book

A look B looking C to look D looked

28. does it take you to the washing ?

A How fast B What time C How long D How often 29.Your hair is long Are you going to ?

A have it cut B have cut it C be cutting it D have it cutting 30.Would you like tea or coffee? – I drink tea than coffee

A would like B would prefer C would rather D would wish 31.It is high time you _more attentive in class

(174)

32.He was _that we turned off the TV set immediately

A so bad singer B such bad singer C such bad a singerD so bad a singer 33.I wish you _that! It’s really annoying habit

A won’t B couldn’t C wouldn’t D don’t 34 “I’m sorry about that!” – “ _!”

A That’s right B Of course C It’s OK D You’re welcome 35.Not until 2008 _across the river

A the first bridge was built B the first building of a bridge was C built the first bridge D was the first bridge built

36.Excuse me! I’m learning my lesson _turning down your stereo a bit ? A Would you please B Would you mind

C Could you D Would you like

37 I was just _ to go out when Peter phoned

A used B planned C about D around

38 The book provides students useful tips to pass the coming exam

A of B about C on D with

39 _ he practised hard, he didn’t pass the driving test A As long asB Even though C In spite of D If

40 I felt disappointed because I was by the company I applied to A criticized B rejected C blamed D forgotten

41 If I were you, I harder to pass the examination A would work B will work C worked D work 42 We were all hungry, so she suggested dinner early

A to have B having C had D have

43 I wish I _ in London with my friends now

A am B was C were D will be

44 I find it difficult to make a good _ on this problem

(175)

45 We didn’t to the station in time to catch the train

A reach B arrive C get D lead

46 My mother is one of the women who money

A enjoys to spend B enjoys spending C enjoy spending D enjoy to spend 47 Their exports are just ours

A similar B like C as D same

48 You may put money in a small coin bank, a piggy bank at home to keep it safe A as such B such C such as D so

49 It is time you hard for the exam

A to revise B revise C revised D revising

50 I live in Dalat, is one of the most beautiful cities of Vietnam A who B where C which D that

Part 4: Give the correct form of the words in brackets to complete the following sentences

1 Nowadays laser beams can be used to treat _ Thousands of are working hard to find out remedies to swine flu

3 Only five were shorlisted for the interview Peter drives very _ He’s hardly had a minor accident

5 My father has lately _ in giving up smoking After the earthquake, very few people were found to be

_

7 Your son is very naughty He has cut himself again, and it is badly

8 The doctor tried to _ the soldier’s wound in a fierce battle

9 She is a studious pupil She always listens _ to her teachers’ lessons

(176)

10 Whenever there are food people immediately preserve food

Part 5: There is a mistake in each of following sentence find and correct it

1 He spent two hours to translate this text into English yesterday morning We are going on a trip to a countryside with our classmates next month Many of the peoples in my home village have lived in their village all their lives

4 First the examiner asked me what my name was, and where I come from Could you send me some information about a possibly course that I can take?

6 They who arrive early will get the best selection of seats

7.Fertilize farmland is one of the biggest natural resources in the Central States

8.There are many different ways of comparing the economy of one nation with those of another

9.Drug addition has resulted of many destroyed careers and expulsions from school or college

10 The company did not want to hire a man that his experiment was so limited

11 I have a very interested job because as a journalist I meet many people 12 My boss watches me closely and lets me know how I am doing and tells me if I am doing bad

13 You probably have never heard of my country where I live because it’s very far of your home

14 I would like you see some pictures of my country and I will attach them to my next report

15 I am trying since I was at school to learn a foreign language but it was difficult for me

16 On their very first evening he told his wife he was just going out for a

(177)

short walking but it was really to buy her some flowers

17 He found a shop quickly and brought the flowers but he couldn't get back to the hotel

18 I have eaten too many of those greasy bacon pies, and now I feel sickly 19 He was upset last Monday morning by her bad attitudes

20 My brother is not old enough for driving a car

Part 6:Choose the best answer to fill in each of the gaps in the text below

There’s now increasing concern about the world's energy resources, particularly about those involving fossil (1) _ In less than a hundred years we shall probably (2) _ all the present (3) _ of oil and gas The world's coal (4) should last longer but, once used, these cannot be (5) _

(178)

energy and nuclear energy as well as water and wind (7) _ (classed as (8) _ energy).Until these energy (9) are widely used, it is important for the developed countries to reduce energy (10) _ as much as possible

1 A fuels B powers C forms D energies

2 A end B complete C total D exhaust

3 A findings B productions C amounts D sources A reserves B stores C mines D contents

5 A updated B repeated C renewed D produced A traditional B alternative C surprising D revolutionary A force B strength C power D motion A repeatable B continual C renewable D continuous A goods B supplies C provisionsD materials

10.A exhaustion B destruction C waste D consumption Part 7: Fill in each numbered blank with one suitable word

George Washington was born on February 22nd, 1732 in Virginia His (1) were Augustine and Mary Washington George (2)

up on a farm in Virginia Little is (3) of his early

childhood He attended (4) irregularly from his 7th to his 15th year His (5) _ subject was mathematics He learned to be a surveyor of land when he grew up He (6) _ the army and was a leader during the American Revolution He later became the first (7) of the United States George Washington is (8) by his people the "Father of our country" The Americans (9) _his birthday on Presidents' Day in February His (10) is on the one-dollar bill

Part 8: Read the following passage and then put a circle round the letter of the correct words to complete each sentence which follows:

Keeping our teeth healthy

It’s very important to have healthy teeth Good teeth help us to chew our food They also help us to look nice

(179)

the tooth This happens after germs and bits of food have collected there Then the decay slowly spreads inside the tooth Eventually, poison goes into the blood, and we may feel quite ill

How can we keep our teeth healthy? First, we ought to visit our dentist twice a year He can fill the small holes in our teeth before they destroy the teeth He can examine our teeth to check that they are growing in the right way Unfortunately, many people wait until they have toothache before they see a dentist

Secondly, we should brush our teeth with a toothbrush and fluoride toothpaste at least twice a day once after breakfast and once before we go to bed We can also use

wooden toothpicks to clean between our teeth after a meal

Thirdly, we should eat food that is good for or teeth and our body: milk, cheese, fish, brown bread, potatoes, red rice, raw vegetables and fresh fruit Chocolate, sweets, biscuits and cakes are bad, especially when we eat them between meals They are harmful because they stick to our teeth and cause decay

1) Good teeth help us _

a) be nice b) have good eyesight c) chew our food d) be important 2) When food and germs collect in a small crack, our teeth _

a) become hard b) begin to decay c) send poison into the blood d) make us feel quite ill 3) A lot people visit a dentist only when

a) their teeth grow properly b) they have holes in their teeth c) they have toothache d) they have brushed their teeth 4) We ought to try to clean our teeth

a) once a day b) at least twice a day c) between meals d) before breakfast 5) We shouldn’t eat a lot of _

a) red rice b) fresh fruit c) fish d) chocolate 6) Sweets are harmful because they make our teeth _

a) black b) ache c) bad d) cracked

Part 9: Rewrite the following sentences, begin as shown and keep the meaning unchanged

1 The flight from Bangkok to London lasted for more than twelve hours

(180)

2 No decision on the issue has been made yet

Nothing It was such a boring film that she felt asleep

The film _ One of my favorite pastimes is watching plays

Going _ You can borrow the car but you’ve got to fill it up with petrol

Provided _ It's quite pointless to complain about the situation

There is no _ She spends too much money on clothing

If You are the worst footballer in the world

No one Susan finds it easy to make friends at her new school

Susan has no _ 10 There are a lot of vehicles on the road today

There is _ Part 10: Use the words given to make meaningful sentences so as to complete a story man/ holiday/ London/ first time

_ after/ arrive/ hotel/ he/ send/ wife/ telegram

_ in/ telegram/ he/ inform/ wife/ of/ safe/ arrival

(181)

_ next day/ he/ go/ sightseeing/ round/ London

_ he/ satisfy/ enjoy/ visit/ much/ as/ it/ one/ most beautiful cities/ world

_ he/ go/ restaurant/ have/ dinner/ after/ tour

_ he/ tired/ want/ go back/ hotel/ get/ some sleep

_ unfortunately/ he/ not remember/ name/ address/ hotel

_ 10 late that night/ wife/ receive/ strange/ telegram/ say/

“please/send/name/address/hotel/at once”

_

ĐÁP ÁN ĐỀ SỐ 17

ĐÁP ÁN 10 CHUYÊN ANH Part 1: Pronunciation

1 B stop D talent B enter C centre D pursue D large A guidance D thread D wonder 10 D palace Part 2: Multiple choice

(182)

28C How long29 A have it cut 30 C would rather 31 B were

32 D so bad a singer33 C wouldn’t 34 C It’s OK35 D was the first bridge built36 B Would you mind 37 C about 38 D with39 B Even though 40 B rejected 41 A would work42 B having 43 C were

44 B decision 45 C ge t 46 B enjoys spending 47 B like 48.C such as 49 C revised 50 C which

Part 3: Multiple choice (0,1 x 10 = 1,0 i m)

1 A fuels D exhaust D sources A reserves C renewed

6 B alternative C power C renewable B supplies 10 D consumption

Part 4: Word formation (0,1 x 10 = 1,0 i m)

1 deafness scientists3 applicants4 carefully5 succeeded alive7 bleeding8 heal9 attentively10 shortages

Part 5: Error shooting (0,1 x 20 = 2,0 i m)

1 to translate They 11 interested 16 walking

2 a countryside Fertilize 12 bad 17 brought

3 the peoples those 13 of 18 sickly

4 come of 14 see 19 attitudes

5 possibly course 10 that his 15 am 20 for driving Part 6: Gap filling (0,1 x 10 = 1,0 i m)

1 parents grew known school favorite/ best joined President called celebrate 10 picture/photo Part 7: Rewriting (0,1 x 10 = 1,0 i m)

1 It took (me) more than twelve (/12) hours to fly from Bangkok to London Nothing (on the issue) has been decided yet

(183)

4 Going to the theatre is one of my favorite pastimes/ something I really enjoy doing Provided (that) you fill the car up with petrol , you can borrow it

6 There is no point/sense/good (in) complaining about the situation

7 If I were her, I wouldn't spend too much/ I would spend less money on clothing (/ If only she wouldn't spend too much/ she would spend less money on clothing) No one (else) in the world plays football as/so badly as you (do)

9 Susan has no difficulty/difficulties in making friends at her new school 10 There is a lot of/ heavy traffic in the road today.

Part 8 : Sentence completing (0,1 x 10 = 1,0 i m)

1 A man was/went on holiday in London for the first time

2 After arriving/ he (had) arrived at the hotel, he sent his wife a telegram In the telegram he informed his wife of his/the safe arrival

4 He also gave her the name and the address of his/the hotel The next day he went sightseeing round London

6 He was satisfied and enjoyed his visit very much as it was one of the most beautiful cities in/ of the world

7 He went to a restaurant to have/ and had dinner after his/the tour He was tired and wanted to go back to the hotel to/ and get some sleep

9 Unfortunately, he couldn't remember the name and (the) address of the/his hotel 10 Late that night his wife received a strange telegram which said “Please send me the name and (the) address of my hotel at once”.

ĐỀ SỐ 18

SỞ GIÁO DỤC VÀ ĐÀO TẠO BẮC GIANG

ĐỀ THI CHÍNH THỨC ( Đề thi gồm trang)

KỲ THI TUYỂN SINH THPT CHUYÊN BẮC GIANG Năm Học: 2007 - 2008

(184)

Học sinh làm vào tờ giấy thi

Học sinh không sử dụng tài liệu kể từ điển

PartI Choose the correct answer A,B,C or D to complete the following sentences. He suggested that we……… to the theatre instead of staying at home

A should go B go C will go D A and B are correct They would buy that house if they ………….enough money

A have B have had C had D would have

3 The rules of the games ………by the referee within ninety minutes

A is observed B observe C are observed D must observe Tom as well as his two brothers…………very good at Maths

A is B are C have been D being

5 We have lived in this town ………1998

A for B since C ago D in

6 The teacher let me……home early because I felt sick

A go B going C to go D went

7 It’s not worth…………her to change her mind She will never listen to anybody A persuading B to persuade C make D to make They…………all day swimming and sunbathing at the beach

A passed B used C took D spent

9 I prefer coffee…… tea I don’t like tea

A from B than C over D to

10 Tom enjoys……strange stamps

A to collect B collect C collecting D to collecting Part II Give the correct form of the verbs in brackets

(185)

(meet already) a lot of interesting peole

2 Yesterday, I ( see) Pam, a friend of mine, at a restaurant I (not see) her for years

At first, I ( not recognise) her because she (lose) at least fifty pounds If I ( be) taller, I (can) be a policeman, but I’m too short

Part III Give the correct prepositions to complete the following sentences Write to me and tell me all your holiday in France

2 The wall of the room was made thick glass

3 She asked her sister to look ………… her baby when she went shopping yesterday My present job is not very wonderful, but I prefer it ………….what I did before Did he break the window……… accident or on purpose?

6 She apologized…………her teacher for her coming late Sorry, I haven’t written ……… you for such a long time Young children are keen……… playing in the rain At weekends, the library is crowded……… students 10 ……… the end, he succeeded in solving the problems

Part IV Give the correct form of words in brackets in the following sentences. He had no (CHOOSE)………but to make his own (DECIDE)………

2 The (DISCUSS)…………lasted hours but we didn’t reach any (AGREE)……… You should (APOLOGY)………to her for your (RUDE)………

4 We often go (BOAT) and (FISH) in the river in Summer The shop (ASSIST)………….is always (HELP)………to customers

Part V Choose the correct answer A,B,C or D that best fits each space in the followingpassage.

A trip to France

(186)

is a very large port A lot of people live in Marseilles and (4) are many interesting shops and cafes there Paul started learning French

at school two years ago and he spoke French all the time with Marie and her family (5) it was very difficult for him but soon it became (6) easier One day, Marie and her parents (7) Paul for a picnic in the mountains They climbed a big hill From the top of the hill, they had a wonderful (8) In the (9) they could see the sea Paul was very sad when it was time to go back to London and school He is already looking (10) to the next summer when Marie is going to spend her holidays with his family in England

1 A got B moved C went D transfered

2 A met B saw C took D greeted

3 A country B town C capital D village

4 A they B those C these D there

5 A At first B At beginning C At starting D At first time

6 A much B so C too D extremely

7 A brought B took C carried D got

8 A view B sight C scene D scenery

9 A space B air C distance D way

10 A through B forward C on D out

Part VI Fill each gap in the following letter with a suitable word.

My most embarrassing experience (1) when I had just left university I had just started (2) in a Liverpool secondary school One morning my alarm clock didn't (3) because I had forgotten to (4) it up I woke up at haft past eight and school began at nine I (5) washed, shaved, got dressed, jumped (6) my car and (7) to school When I arrived, the students (8) already gone into class I didn't go to the staff room, but went (9) into class After two or three (10) the students (11) laughing, and I couldn't understand (12) ! Suddenly I looked (13) and (14) I had (15) on one black shoe and one brown shoe!

(187)

In Britain, natural gas is the most popular fuel for heating homes, and for cooking It is piped all over the country from the gas fields below the North sea Natural gas is used chiefly as a direct source of energy, although it is also used in the chemical industry At the moment, the supply is plentiful, but it will be running short by the end of the century, and it will run out during the 21st century unless new supplies are found. We could without natural gas, as coal gas can be used as a substitute, and there should be enough coal to last for several centuries However, it is a form of energy which is not only cheap, but also highly efficient

1 What is natural gas used for?

 Where does it come from?

 What is gas used chiefly as? When will it run out?

 What can be used as a substitute for natural gas?

 Is natural gas expensive or cheap?

Part VIII Find one mistake and correct it in each of the following sentences.

1 My best friend is coming to Hanoi to visit me in 19th July Most of the students seemed very nervously before the final exam

(188)

9 Would you like going camping with us this Sunday? 10 When you want to go fishing this morning, I'll go with you Part IX Complete the following letter using suggested words

Dear Sir

 I/ see/ your Institute’s advertisement/ Today’s TV program.//

 I/ interested/ learn English and I/ would like/ some information/ Institute.//  I/ can speak/ a little English/ but/ read it /slowly/ writing/ bad so/ I/ want/ improve/ reading/ writing.//

 Could you/ provide/more information/ length/ courses and fees/ beginners? I/ can supply/

record of English study/ if necessary.//

 I/ look forward/ hear/ you/ soon.//

 Yours sincerely

Susan

Part X Make questions for the underlined words or phrases

(189)

 _ She often goes to work by bike

 _ They have moved to London for nearly two years

 _ I want to know more information about the English course

 _ He often goes out for a walk with his family after dinner

Part XI Rewrite the sentences in such a way that they mean almost the same as those printedbefore them.

1 She spends two hours a day practicing English

It takes……… If you don’t work hard, you won’t pass the entrance exam Unless……… We finished this task in about two hours

We spent……… People think Columbus found America before others America………

5 “ Shall we go for a ride to the countryside this Sunday” one of the boys said One of the boys………

6 I like ice – cream and my sister likes it, too

Both……… We didn’t return home until we finished the test

Not until……… I had just arrived home when my mother phoned me

No sooner……… This exercise is very difficult so we can’t it well

(190)

10 Although they were very poor, they managed to send their children to school Despite………

The end ĐÁP ÁN ĐỀ SỐ 18

HƯỚNG DÃN CHẤM ĐIỂM Total: 100 points = 10points

PartI 10points

1 D C C A B A A D D 10 C Part II 10points

1 have you learnt ( learned) – came – have you made – have already met saw – hadn’t seen – didn’t recognise – (had) lost

3 were – could Part III 10points

1 about of after to by to to on with 10 In Part IV 10points

1 choice – decision discussion – agreement apologize/ apologise – rudeness boating - fishing5 assistant – helpful

Part V 10points

1 C A B D A A B A C 10 B Part VI 15points

1 happened teaching/working ring4 wind quickly / rapidly into drove

8 had9 straight 10 minutes 11 started/began 12 why13 down14 understood15 put

Part VII 5points

(191)

3 Gas/ It is used chiefly as a direct source of energy It will run out during the 21st century.

4 Coal gas can ( Coal gas be used as a substitute for natural gas) It’s/ It is cheap

Part VIII 10 points

1 in -> on nervously -> nervous did -> was doing had had -> had

5 to help -> helping used to -> used to be done you -> don’t you

8 to talk -> talking going camping -> to go camping 10 When -> If Part IX points

Dear Sir,

1 I saw your Institute’s advertisement on Today’s TV program

2 I am (very) interested in learning English and I would like some information about your Institute

3 I can speak a little English, bur I read it ( very) slowly and my writing is bad so I want to

improve my reading and ( my) writing

4 Could you please provide more information about the length of courses and fees for beginners? I can supply my record of English study if necessary

5 I look forward / am looking forward to hearing from you soon Yours sincerely

Susan Part X points

(192)

5 When does he(often) go out for a walk with his family? Part XI 10 points

1 It takes her two hours a day to practice/ practise English Unless you work hard, you won’t pass the entrance exam We spent about two hours finishing this task

4 America is thought to have been found by Columbus before others

5 One of the boys suggested going for a ride to the countryside that Sunday Both I and my sister like ice – cream

7 Not until we returned home did we finish the test

8 No sooner had I arrived home than my mother phoned me This exercise isn’t easy enough for us to it well

10 Despite being very poor, they managed to send their children to school

Or: Despite the fact that they were very poor, they managed to send their children to school

Or: Despite their poverty, they managed to send their children to school

ĐỀ SỐ 19

KỲ THI TUYỂN SINH LỚP 10 CHUYÊN THPT_ TP.HCM NĂM HỌC 2008-2009 MƠN: TIẾNG ANH (mơn chun)(thời gian 150 phút)

I Choose the word whose underlined part is pronounced differently from that of the others:

1 A eventually B depletion C competitive D regretful A vacancy B wholesale C inflation D stagnant A education B bridges C garage D enlarge

(193)

5 A enthusiasm B fertilizer C indicator D socializing III Choose the correct answer to fill in the blank:

1 I don’t see any _ in arriving early at the theater if the show doesn’t start until o’clock

A cause B reason C aim D point The new manager’s office has _ new equipment

A many B a lot of C some of D a few I don’t _ to change my job because I like it

A risk B intend C persist D insist You thought I did wrong but the result _ my action

A agreed B approved C proved D justified Thomas has a garden which is _ mine

A double as large B semi-larger than C twice as large as D as two-time as large

6 _ “What you think of the film we’ve watched?” _ “ ” A It’s a breeze! B No kidding!

C I’ve seen better D None of your business!

7 In 1870, _, John D Rockefeller and others created the Standard Oil Company A that oil prices fluctuated B despite fluctuating oil prices C but the oil prices fluctuated D oil prices were fluctuating

8 to school by bus

A Many a student goes B Many the students go C Many of students go D Much student goes

9 Careless driving also accounts the increasing number of traffic accidents

A on B into C for D with

10 is not clear to researchers

A Why dinosaurs having become extinct B Why dinosaurs became extinct

C Did dinosaurs become extinct D Dinosaurs became extinct

11 The new manager is easy-going He is always very serious about the work A by no means B by means of C by all means D in the mean time 12 We enjoy the hams and coffee at this _ restaurant

(194)

A haven’t been invited B hadn’t been invited C not be invited D not have been invited

14 When the university suggested _, the student union protested vigorously A tuition be raised B to raise tuition

C on raising tuition D that tuition should raise

15 one after another, parallel computers perform groups of operations at the same time

A Conventional computers, by handling tasks

B Since tasks being handled by conventional computers C Whereas conventional computers handle tasks

D While tasks handled by conventional computers 16 She him of having lied to her

A threatened B blamed C criticized D accused 17 We have bought extra food _ our friends stay to have dinner

A in case B if C provided D as long as 18 Cut this cake into six _ pieces

A same B alike C like D equal

19 If only it _ differently! I had hoped to be acquitted A had been concluded B were concluded

C had concluded D concluded

20 I was in the countryside but went to school in HCMC

A brought up B taken over C made up D put off

21 Most of _ archaeologists know about prehistoric cultures is based on studies of material remains

A these B what C which D their

22 We couldn’t afford that house because it

A was over my head B paid through the nose C cost an arm and a leg D blew my own trumpet 23 the Christmas shopping season begins

A That is after Thanksgiving B After Thanksgiving it is C It is after Thanksgiving that D It is Thanksgiving that

24 New York City is America’s largest city, it is not the capital of New YorkState

A therefore B but C so D however

25 He promised her an Oxford dictionary, but disappointingly he _ on his word A turned over B stood up C went back D sat down

(195)

A for B to C as D with 27 She came a lot of problems at work

A up with B down with C in for D up against 28 They take each other like two peas

A in B after C to D for

29 Didn’t it ever to them that they would be severely punished?

A occur B happen C enter D come

30 If you want to weight, cut down on fat

A put off B come down C take over D cut off 31 By the time we get out of this traffic jam, all the guests _ home

A have gone B had gone C will go D will have gone 32 members of the class has to be responsible for his own actions

A Each of the B Every C None of D All 33 The teacher _ them the answer to the question

A explained B said C discussed D told 34 We met each other at the meeting coincidence

A by B in C for D to

35 As we’ve been discussing for hours without any solution, I’d like to my own solution to this problem

A come up B bring down C put forward D take in 36 The accident resulted some minor injuries

A to B from C in D of

37 Dave, I really appreciate _ me But I think I could manage it myself

A you to help B you helped C your helping D that you would help 38 One of the robbers was described by the witnesses _ tall and about 25

A like B by C in D as

39 I left the purse at home Can you _ without breakfast today, dear?

A go B C make D live

40 One of the solutions _ by environmentalists is to ban vehicles from the city center

A are suggested B is suggested C suggesting D suggested

41 While the boss was away, her assistant stood her and helped her solve any problems arising

A up to B in for C up against D out of

(196)

43 We could never get ready for the test at such short _

A call B notice C advice D note

44 To resist corrosion _ for today's car to prevent havoc caused by road salts, gravel and other materials

A have new coatings been developed B new coatings developing

C development of new coatings D new coatings have been developed 45 Some scientists say it is essential that mankind _ the amount of air pollution in big cities

A be reduced B reduced C reduce D will reduce 46 She couldn’t but _ there crying

A stood B standing C to stand D stand 47 where to find the key, the boy could not open the safe

A Knowing not B Knew not C Not knowing D Didn’t know 48 They received _ advice from their parents that they became successful A so good B such a good C so good an D such good 49 Computers are said to be _ for the development of mankind

A here to stay B neither here nor there C here and there D here today, gone tomorrow 50 _ “She’s the best pianist in the school.” _ “ ”

A That’s that B Never mind!

C That’s OK! D You can say that again IV Identify the mistake in each sentence:

1 Although most known for her prose works, Maya Angelou also published several collections of poetry

A most known B her prose works C also published D poetry

2 Technically speaking, astronauts can be able to visit the moon and live there briefly A Technically B can be able to C visit D briefly

3 There are many species of birds in danger of extinction, with a large number of which, 117 in all, found on oceanic islands

A There are B extinction C of which D in all If it is kept dry, a seed can still sprout up to forty years after their formation

A kept B can still sprout C up to D their

5 Some snakes have hollow teeth they are called fangs that they use to poison their victims

A Some B they are called C use D their

(197)

A The nitrogen B over C percent of D surrounding

7 Why so many students graduate weak at reading and math are questions continuing to disturb educators

A so many B weak at C are questions D.continuing

8 Although apples not grow during the cold season, apple trees must have such season in order to flourish

A Although B during C such season D to flourish

9 Dentistry is a branch of medicine that developed very dramatically in the last twenty years

A Dentistry B branch of C developed D dramatically 10 Rabbits have large front tooth, short tails, and hind legs feet adapted for running and jumping

A Rabbits B tooth C and D running and jumping

V Choose the suitable word to fill in each blank:

While (1) everyone accepts the goal of developing (2) _ in the three "R's" - reading, writing, and arithmetic - it often seems impossible to (3) _ agreement on any goal beyond that In the broadest (4) , the conflict over educational goals can be (5) _ as a conflict between conservatives and liberals, or, (6) _ they are sometimes called, essentialists and progressives

The conservatives, or essentialists, tend to identify (7) desirable education with the transmission of the cultural heritage, a no-nonsense (8) _ featuring the three R's at the elementary-school level, and academic studies or strong vocational or business courses in the secondary school They put (9) _ on the training of the mind and cultivation of the intellect

The liberals, or progressives, tend to be interested in the development of the

(10) _ child, not merely in training the child's mind or in preparing the child for adult (11) _ in a remote future They emphasize rich, meaningful school living in the present, and they view subject matter as a resource (12) total human development (13) _ as a goal in itself They not downgrade content but believe it should be (14) _ not for (15) own sake but as a means of fostering thought and inquiry

1 A most B mostly C almost D utmost A talents B actions C skills D techniques A get B accomplish C take D reach A terms B words C letters D means A viewed B thought C counted D named

(198)

7 A certain B a C the D as A subject B curriculum C object D program A significance B stress C emphasis D strength 10 A complete B whole C all-round D total 11 A living B lives C liveliness D life

12 A at B in C of D for

13 A but for B rather than C instead D better 14 A inquired B required C acquired D requested

15 A it B its C them D their

VI Read the following passage and choose the best answers to the questions: A pioneering study by Donald Appleyard made the astounding discovery that a suddenincrease in the volume of traffic through an area affects people in the way that a sudden increase in crime does Appleyard observed this by finding three blocks of houses in San Francisco that looked much alike and had the same kind of middle-class and working-class residents, with approximately the same ethnic mix The difference was that only 2,000 cars a day ran down Octavia Street (LIGHT street, in Appleyard’s terminology) while Gough Street (MEDIUM street) was used by 8,000 cars daily, and Franklin Street (HEAVY street) had around 16,000 cars a day Franklin Street often had as many cars in an hour as Octavia had in a day

Heavy traffic brought with it danger, noise, fumes and soot, directly, and trash secondarily That is, the cars didn’t bring in much trash, but when trash accumulated, residents seldom picked it up The cars, Appleyard determined, reduced the amount of territory residents felt responsible for Noise was a constant intrusion into their homes Many Franklin Street residents covered their doors and windows and spent most of their time in the rear of their houses Most families with children had already left Conditions on Octavia Street were much different Residents picked up trash They sat on their front steps and chatted with neighbors They had three times as many friends and twice as many acquaintances as the people on Franklin

On Gough Street, residents said that the old feeling of community was disappearing as traffic increased People were becoming more and more preoccupied with their own lives A number of families had recently moved and more were considering Those who were staying expressed deep regret at the destruction of their community

1 The word “astounding” in line is closest in meaning to A startling B disappointing C dubious D alternative The three streets mentioned in this passage are different in that

A they are in different cities

(199)

D the income levels of the residents vary considerably Approximately how many cars used Franklin Street daily?

A 2,000 B 8,000 C 16,000 D 20,000 All of the following are direct results of heavy traffic EXCEPT

A increased amount of trash B greater danger to residents

C more pollution D more vibrations

5 The author’s main purpose in the second paragraph is to A discuss the problem of trash disposal

B point out the disadvantage of heavy traffic C propose an alternate system of transportation D suggest ways to cope with traffic problems

6 On which street was there the most social interaction? A Octavia Street B Gough Street

C Franklin Street D There was no significance social interaction on any of the three streets

7 The word “chatted” in line 14 is closest in meaning to A joked B talked C argued D walked

8 Which of the following is NOT a statement you would expect from a resident of Gough Street?

A People on this street are unhappy because the neighborhood is deteriorating B People on this street think mostly of themselves

C People on this street have more and more space for which they feel responsible D A number of people are preparing to leave this street

9 In what order does the author present detailed discussions of the three streets? A LIGHT, MEDIUM, HEAVY B HEAVY, MEDIUM, LIGHT

C HEAVY, LIGHT, MEDIUM D LIGHT, HEAVY, MEDIUM

10 What is the writer’s attitude toward heavy traffic when he mentions the Appleyard’s study?

A neutral B favorable C critical D doubtful VII Supply each blank with one suitable word:

(200)

Many of their stories and art forms have remained (7) over time, and today, women weave clothing and carpets that have signs and symbols that are the same as (8) found in ancient temples

The different patterns and symbols on these brightly coloured (9) not only look attractive, (10) can also communicate important information such as marital (11)

Even though the majority of the Mayas are now Catholics, many of the festivals that they (12) _ part in are a mixture of the old rituals that their ancestors (13) to practise and more modern Christian elements It was perhaps the Mayas' ability to (14) _ and change that made it possible for them to survive to the (15) day

VIII Supply the appropriate forms of words in the brackets:

1 Young children should be well aware of their manner (BEHAVE)

2 Please put the books back to where you took them Don’t any of them (PLACE)

3 The are scheduled to take off at 15:00 and 15:30 (FLY)

4 I’ve never met such a strong man His energy seems (EXHAUST) Those rose bushes need protection Spray them with (INSECT)

6 Don’t you know that your good result will make your parents _ proud of you? (MEASURE)

7 Be careful You may be _ to put all your eggs in one basket (ADVICE) The boy watched the performance of the tigers, _ with amazement (BREATHE)

9 I would like to express my admiration for his _ of knowledge (PROFOUND) 10 The cancellation of the case resulted from the in court of the defendant resulted in (APPEAR)

11 The _ of the Ministry of Education announced the date of the graduation exam (SPEAK)

12 Mercedes is a very car (DEPEND)

13 There should be staff in case someone is absent and needs to be replaced (STAND)

14 You’ll be punished for all your (DO)

15 With all the modern facilities, doing housework is still very (CONSUME)

book: https://www.facebook.com/Toeic-4U-923591884393234/ epicenter

Ngày đăng: 06/03/2021, 03:29

Từ khóa liên quan

Tài liệu cùng người dùng

Tài liệu liên quan